Related Articles by Labels



Widget by Hoctro

Monday, September 8, 2008

Kanbay Placement Paper

We had kanbay on 10 th aug in MVSR eng.. college.

30 quest were logical reasoning
30 ques were maths (Arithmetics and some maths topic like geometry etc)
duration was 90 mins and cutoff is 12 from each section.
after that GD and then personal interview. and paper was 2 sets (codes both are entirely different I think one was little bit easy).

maths was not so simple it was little bit tough. i guess it is of CAT standard. so solving RS agrawal and other is not sufficient. if possible prepare form any cat book it would definitely help u its my experience.

prepare well good luck. Plz. consider my experience.
I happened to attend kanbay interview at my campus - vasavi college of engg HYD

Kanbay test consisted three rounds
first one --- written test, consisted 2 parts 1>MATHS 2> REASONING.
reasoning was dead easy & maths was easy but somewhat time consuming with many calculations. there were five sets of papers, time given was 90 mins (more than sufficient) with cuttoff 12 in each section (i fell there was upper cutoff too!)

second one ---- a GD
we were given choice, later they have decided to conduct role play. batch of 10 for abt 30 mins, many were selected.

third --- interview consisted both HR as well as TECH .it was less of tech more of HR

overall it was very easy, I personally feel kanbay has lot of requirement!
have attended on-campus of CTS at JNTU, hyd on 20/jul/05. I want to share some of my experiences with u.

PATTERN:
1)Written test 40 Questions 60 minutes
problems from probability, time&distance,progressions,4 to 6 from general english
2)Technical + Hr for 30 min.

Written test looks a little bit hard but answer those questions u r perfect first. So that time will be adjusted. Don't get nervous, nobody can attempt full paper.

Interview is so cool and every question they ask depends on ur previous answer. while giving answers see that u know everything u r saying about. Be confident while answering even it is wrong. Just go thru the 64-Q&A generally asked in interviews like why should we hire u? why do u think ur perfect for this company? Impress them with ur answers.

Kanbay Placement Paper

The selection took place in three rounds each of which was an elimination round.
1)Written Test
2)Group Discussion
3)Personal Interviews
Each section contained 30 questions.The cut-off was 12 marks in each section which was clearly announced before the exam started.1 Mark was awarded for each correct answer.1/4 Mark was deducted for each wrong answe

The questions given below were all that could be reproduced from all the 3 sets together.

Average of 5 innings is 20. If four innings are 32, 10, 40 and 12 find the 5th inning score.

Area of path around a square is 256 sq.m. Find the width of path if the side of square is 30m.

A company increases the bonus 3 times of its 50 employees. What is the % increase in bonus?

A man swims in a river 5 min upstream and 5 min downstream. Starting at A and ending at B. What is the speed of the man?

NESTIN is a jumbled word. What are the first and last letters of the correct word.

There is enough material to fence 30m linear. Also there is old fence on oneside of the plot. Find the length and breadth of the plot.

In a game of Snooker A gives B 18 points for 90, A gives C 10 points for 60. How many can C give B in a game of 70?

C is the daughter of B. h is the brother of C. F is the father of C. G is the son of F. D is the brother of F. What is the relationship between C and D?

A moves 25km North and then finds that she is movin the wrong way. She then takes a right turn and moves 2km. Then again she takes a right turn and moves 25km. How much distance does she have to travel to now reach the starting point?

Some cats are good. Some cats are black.

Gold is 19 times heavier than water. Copper is 9 times heavier than water. In what ratio should they be mixed so that the alloy is 15 times heavier than water?

A sphere of radius r is placed inside a cylinder of height 2r which just fits. What is the Volume of the empty space?

A is 3 times as old as B. B is 4 years older than C. If C is Z years old express A in terms of Z.

A is punctual. All punctual are on time.

Conclusion to be chosen from the given multiple choises.

a_ _ _baccba_ _ _aa

Srikanth has borrowed Rs 3000 at 10% C.I.. He has to repay that after 3 years in 3 equal installments. How much does he have to pay as an installment?

Given a date and day, the question was to calculate the day of the date June 12, 1979.

The weight of a Bucket full of water is 17 kg, the weight of the same bucket with half full water is 13.5 kg. What is the weight of the Bucket?

In a party attended by boys and girls where the girls outnumbered the boys Rs 280 was distributed. Each boy was given Rs 10 and each girl Rs 20. How many boys are there?

10% of an army died. 10% of the were ill. 12% of the rest met with an accident. What is the total number of the original army if 7 lakhs were finally left?

A started a business with Rs 1700 amount. B joined A after 3 months , C joined after 6 months. If the profit is Rs 1700 and they shared it in the Ratio of 2:3:5 how much did B and C invest?

A beats B by 24m. A beats C by 20m. C beats B by 1 sec. In how many seconds does A complete the race of distance 120m.

VIJAY is coded as YLMDB. STOP is coded as VWRS.
A few questions were given based on this code.

9 3 6
2 ? 1
4 7 8

20 15 10
15 10 5
10 ? 10

Which is larger Rs 35 : Rs 1.40 or Rs 48 : Rs 1.44

A started a business with Rs 1700 amount. B joined A after 3 months , C joined after 6 months. If the profit is Rs 1700 and they shared it in the Ratio of 2:3:5 how much did B and C invest?

In a group of people 600 are Non-veg and 400 are Veg. Of these 150 are both Non-Veg and Veg. In the Non-Veg 300 eat Mutton and 400 eat Chicken.(i) Find the number of people who eat only Non-Veg Mutton.(ii) Who eat only Veg.

In a wall clock the smaller needle is between 8 and 9 and the larger makes a right angle with it. Find the exact time.

If the temperature increases uniformly from 9 am to 2 pm and it increases from 21 C to 36 C. What is the temperature at noon?

Place a word in the brackets such that it makes a meaningful suffix to the first part and a meaningful prefix to the second part.
TEM (_ _ _ _) ERS

A monkey climbs 3m of a greased pole in 1 minute and then slips 1m in the next. When will he reach the top of the pole of 12 m?

The Alphabet that is 3 from the right to the letter between K and S.

Length and Breadth of a Rectangle are 100 m and 6 m. Find the area of a road around the rectangle with a width of 5m.

Find the figure with the highest Area
i)Circle with r=2.
ii)Equilateral Triangle with Side = 4.
iii)Square of diagonal =2.
iv)Triangle with a=8, b=5 and c=4.

A boat moves 10 km upstream and returns to the same point in 45minutes. The speed of the stream is 3km/hr. The speed of the boat is ?

Given an Equilateral triangle of side 10m. A cow is tied to one end. The length of the rope is 7 m. The area covered by the cow is ?

A is 7th from the left, B is 9th from the right, if they exchange A is 11th from the left. The total number of persons is?

A person moves 5 km East, turns right moves 4 km and then turns right again and moves 5 km. What is the position of the person from the starting point?

If PLEASE is coded as RMIGKI, then SLEEP is coded as ______

If TRUCK = 25 and DERIVE = 36 , What is CAPTAIN ?

A rectangular field of length 30 m and breadth 18 m. A carpet of 20 cm width is to be used to cover the field. The carpet costs 0.50ps per metre. What is the total cost of carpeting?

A person travels 20km towards North, he takes left and travels for 15km, then he takes left and travels for 20km. In what direction is the travelling now?

A train goes from A to B at 50 km/hr and comes back at 40 km/hr and hence takes an hour more time to return. What is the distance between A and B?
Q45)D/23, 17/F, I/12, 8/M, R/5

Kanbay Placement Paper 12 July 2007 AT G.I.T.A.M COE, VIZAG

KANBAY PAPER ON 12th JULY

IT'S A GOOD MORNING FOR ME, MY DEAR FRIENDS.....................

IN GENERAL KANBAY (OR) CAPEGEMINI IS CONDUCTING IT'S TEST PATTERN AS FOLLOWS:
-----1) WRITTEN TEST --> MATHEMATICS + ANALYTICAL (VERY EASY)
-----2) GROUP DISCUSSION
-----3) GENERAL INTERVEIW (TECHNICAL PARTLY & MAXIMUM OF HR)

1. WRITTEN CRITERIA:
AT 11:00 WE HAVE ATTENDED 4 WRITTEN
I FEEL THAT AS AN ENGINNERING STUDENT IT CAN BE ANSWERED WITHOUT ANY PREPARATION SINCE'''''
MATHEMATICS IS OUR INTERMEDIATE BASIS & ANALYTICAL IS OUR COMMONSENSE.........

MATHEMATICS-------->30 QUESTIONS + }-------->60 MIN & CORRECT--1M ; WRONG--->1/4 -VE MARKING;;;;;
ANALYTICAL----->30 QUESTIONS

CUTOFF---->MIN OF 10M IN EACH SECTION......
NO "OVER" CUTOFFF...........
ATTENDED : 300
PASSED : 66

I DID WELL & LUCKILY I'M ONE OF 66.....

2. GROUP DISCUSSION :
12 A BATCH ; TOTALLY 6 BATCHES.......OUR TOPIC IS "RAGGING IN CAMPUSES" IAM THE INITIATER ; FLUENT IN ENGLISH & STARTED SMARTLY BY REPLACING, THE CULTURE OF RAGGING WITH INTERACTION AMONG SR'S & JR'S.....
I LEAD THE TEAM EFFICIENTLY THROUGH OUT MY DISCUSSION BY------> RELEASING NEW POINTS WHEN DEVIATION IN TEAM OBSERVED
Like organisation of events like sports & freshers meet to have healthy interaction among them
------>made the team to run on my words & ultimately gaining trust from them to my point
------>good body language;evoking points like by frequent get-togethers WITH SR'S JR'S CAN acquire academic knowledge efficiently & faster than single one's...
------>SO,INITIATE ; COME WITH GOOD POINTS WHENEVER NECESSARY; ETTING CONSENSUS IS QUITE IMPORTANT AT THE END.... U GET SELECTED IN GD IF U FOLLOW THIS

RESULT :OUT OF 66 ;;;;;"44" ARE SELECTED since all batches done well as a team & group.......... i'm one of themmmm........

3. INTERVIEW :
ABSOLUTELY IT IS A FULL PLEDGED HR --INTERVIEW.........
I WAS CALLED & I ENTERED THE ROOOM (I WAS FROM "ECE" BACKGROUND)

----> GOOD EVENING SIR! --------->HI SRAVAN TAKE UR SEAT
----> THANQ SIR
----> TELL ME ABOUT YOURSELF ?--------> I COVERED MY RESUME WITH THE ADDITION OF MY QUALITIES
----> INTERSTING AREAS --- 8085 MP & PROGRAMMING IN 'C'....,
----> ASKED ME 2 WRITE A PROGRAM IN 8085 ASSEMBLY LANGUAGE
----> ABOUT LINKED LISTS IN "C".....
----> ABOUT FLIPFLOPS IN DCS...
----> HOW DO U REACT IF U R NOT SELECTED IN KANBAY ?
----> GAVE AN EXTRA-ORDINARY ANSWER WITH GOOD VOCABULARY....
----> AFTER MY ANSWERING TO ALL QUESTIONS HE ASKED

ONLY ONE THING THAT """ HOW CAN U GIVE UR ANSWERS IN SUCH AN EFFECTIVE WAY!!!!
I THINK U HAVE PREPARED EVERY WORD BEFORE INTERVIEW ????????"""""""
ANSWER : TO BE HONEST "YES" SIR ...

-----> THEN HIS QUESTION WAS " SO U SPEAK ALL THIS FORMALLY BECAUSE OF INTERVIEW BUT NOT REALLY????""
ANSWER: NO SIR "WORDS ARE NOT COMING FROM MIND ,,,EVOKING FROM MY "HEART".......""""""""
-----> FOR THIS HE WAS impressed VERY MUCH BY SAYING "wow what a golden word"......AFTER THAT HE ASKED MY STRENGTHS
-----> HABIT OF DOING IT NOW; VERSATILE & FLEXIBLE ; OPTIMISTIC

TOOOOOOOOOOO IMPRESSED
HE ASKED ABOUT LOCATION : MUMBAI , PUNE , HYD etc.,
I SAID THAT "'''' I WILL GO WITH THE COMPANY""""""
THAT'S MY INTERVIEW COMPLETED................

OUT OF 44 ----SELECTED -----> 30 ******** I WAS A KANBAIAN NOW**********
I ATTENDED ----> ACCENTURE ; MINDTREE ; HP-------> OUT IN GD'SSSSSSSS
----> INFOSYS ------> OUT IN INTERVIEW
I LEFT OUT ------>WIPRO & SATYAM ( i.e., I MYSELF DROPPED TO THESE COMPANIES DUE TO HAVING NO INTEREST IN THEM)

ALL MY FRIENDS RECRUITED BUT IAM WAITING FOR GOOD COMPANY
NSPITE OF THAT I HAVE ATTENDED WITH ABUNDANCE OF "CONFIDENCE " WHICH IS KEY TO SUCCESS

SOOO AT ANY TIME """""""""DON'T LOSE UR CONFIDENCE""""""""""""""""""

IVEGA Placement Paper 1

IVEGA Test

There will be 4 rnds,----written,gd,interview-tech,hr-rnd.
The written test----will be of 30 min. n 40 qns.. all r multiple choice qns.
includes eng,apt-maths,gk
gk-such as wht'z the diff bn GMT & IST..ans is +5.5 hrs..
apt-from RSAgarwal--train,time,distance,work,geometric-area,volume...
eng-from GRE syn,ant,fill in blnk..........
GD -----will be of 15-20 min abt a topic...
introduce urself and discuss abt topic....
some topics r: Love marriages vs arranged.,Indian culture is westrenizing.,
TV programs r women oriented, women equal to men,
Politician-do they need min. educational qualification..............
that's all abt 2 rnds,
Then tech intr...
Lastly HR rnd..........
After clearing all rnds u r IN.......as IVEGA emp........


Ivega was started in 1997, now the employee strength is 127, it is located in globle village , behind R V College of Engineering. it is a very small company.work is good i suppose. this recruitment is due to its merger with a france based company. i dont think that pay will be very good. regarding platforms they r presently working on .NET , ASP, JNI, JAVA.

The test is of 30 min...

it contains

1.english Verbal GRE synonyms antonyms

2.aptitude Aggarwal book

3. General knowledge

4.anlatical reasoning

Total 50 Q


Ivega conducts an aptitude test.....in which there are
50 ques to answer in 30 minutes.i think it would be
best to go thru aptitude books like r.s.
aggarwal.moreover, there are 5 rounds to get
thru....viz. aptitude test, G.D., hr interview,
technical test, C.O.O.'s interview.
anyway, all the best....



about the test.....


It was mix of all, like there were some aptitude questions, reasoning,
English grammer but surprisingly no analytical questions...for the
pattern given to me.

there were some general awareness questions like when does Olympics
started??

a man is given 8 $ per hour and he works for 8hr per day, for working
xtra hr he gets 1.5 times more than wht he usually gets. if he gets 80$
for one particular day, then how many did he work??

test duration : 30 minutes
no of questions : 50.

Topic for GD : should western dress code be cumpulsory for IT jobs.

iSoftTech Placement Paper 2

1.written test

In written it consists of

1.aptitute 25 Q's very easy

2.verbal 5 fill in the blanks,5synonyms,5 antonyms..

very easy for fill in the blanks ans is 1.d 2.d 3.d 4.a 5.d

3. Next c it contains 30 Q's plz follow the attachment be'coz all the Q's R repeated



2.G.D

For me topic is "Is s/w professionals r paying more ? It's a fact or not? "

3.Interview

Thats all be through the Q's .......

There cutoff is 50 out of 70..

C Question

1.a=5,b=3,c=a,b
d=(a,b)
printf(c,d)
ans:c=5,d=3

2.e(int n)
{
if(n>0)
{
...(not clear)
printf("%d",n);
e(--n);
}
return
}
ans:0,1,2,0

3.which has no problem with pointers

int *f1()
{
int n;
return (n)
}

int *f2()
{
int *p;
*p=3;
return p;
}

int *f3()
{
int *p;
p=malloc();
return p;
}

ans:no error

4.header file ->contains declarations.

5.sizeof operator is executed during compile time..

6.*p+=1
*p++
are these two same?
not same.

7.func(int i)
{
static int count;
count=count+i;
}
ans:1+2+3...(counts values does not go after funtion call

8.is('a'<'b') true

10.short int=16 bits

11.4 stmt. ans.int float i;

12.int num[3];
num[3]=2;

ans:first stmt deals with size
second deals with element

13.j=4
for(int i=0;i<5;i++)
{
j++;
++j;
}
output of j.
ans:14

9.char s1[20]="hello world";
s1[5]="\0";
printf("%d",strlen(s1));
printf("%%.%...(not clear)",s1);
}
ans:bad format specifier

14.brace { used in c for what ans:convention

15.parameters in c passed by ans:value.

16.when an array is passed it is by ans:pointer.

17.scanf can read ans:any data type

18.which cant be passed to subroutine.ans:preprocessor directive.

19.to get string of words..ans:gets()

20.external variables can be accesed ans:in functions which use them.


analytical:

1.cat->satc
dear->seard

sing->sings

3.1999 july 21st friday
1947 july 21st ?
ans:two days before.

4.2,12,30,56,




GD Question

HOW SOFTWARE PROFESSIONALS HELP IN THE FINANCIAL GROWTH OF A COMANY"

who is your role model premji(CEO) or Gandhiji

"it professionals part in indian economy"



This is the feed back given by my friend in venksteshwara.He has cleared the first two rounds of i-soft.plz do have a look at this.

1 analytical - some 30 questions i remember
2 verbal - 15 ques
3 technical " c " - 30 ques

analytical - 3 calendar questions., 1 relationship question(answer grand son) , it was an easy paper { only calendar q were tough for me}

verbal - 5 fill in with choices { answers i got were E,E,E,E,E)
1ST ANS - DELIVERED ON
2ND ANS BARREN
SO ON

BE PREPARED WITH HIGH FREQUENCY WORDS FROM BARRONS
ALL THE QUESTIONS WERE FROM THOSE 300 WORDS


TECHNICAL " C"


i give some of details.

apptitude test

analtical part ,

verbal are very easy.

c part also look like very easy but be careful.the first 4 questionare somewhat difficult all others are easy.all are direct questions like, compile time error etc.

if you asked him to give the topic in g.d they will give

"it professionals part in indian economy"
they give the same topic in every college.

if the group was follows the g.d rules then you follow ,otherwise you must shout like
lok shaba .
they mainly select the top scores in the apptitude.

in interview they will not ask anything.
just about you and your family background .so in interview there is no problem.

for c questions read the pointers in c book.there are some solved problems in back of every chapter it will help you lot.

don't worry about time time you have enough time.but do c questions carefully. they are very easy.

FIRST 5 QUESTIONS WERE VERY TOUGH
REMAINING WERE OK


ANYHOW JUST TRY TO ANSWER MORE QUESTIONS.DONT STICK TO ANY QUES. MOVE ON TO THE NEXT IN SUCH SITUATIONS.

iSoftTech Placement Paper 1

iSoftTech Paper

1 analytical - some 30 questions i remember
2 verbal - 15 ques
3 technical " c " - 30 ques

analytical - 3 calendar questions., 1 relationship question(answer grand son) , it was an easy paper { only calendar q were tough for me}

verbal - 5 fill in with choices { answers i got were E,E,E,E,E)
1ST ANS - DELIVERED ON
2ND ANS BARREN
SO ON

BE PREPARED WITH HIGH FREQUENCY WORDS FROM BARRONS
ALL THE QUESTIONS WERE FROM THOSE 300 WORDS


TECHNICAL " C"

FIRST 5 QUESTIONS WERE VERY TOUGH
REMAINING WERE OK


ANYHOW JUST TRY TO ANSWER MORE QUESTIONS.DONT STICK TO ANY QUES. MOVE ON TO THE NEXT IN SUCH SITUATIONS.


C Questions

1.a=5,b=3,c=a,b
d=(a,b)
printf(c,d)
ans:c=5,d=3

2.e(int n)
{
if(n>0)
{
...(not clear)
printf("%d",n);
e(--n);
}
return
}
ans:0,1,2,0

3.which has no problem with pointers

int *f1()
{
int n;
return (n)
}

int *f2()
{
int *p;
*p=3;
return p;
}

int *f3()
{
int *p;
p=malloc();
return p;
}

ans:no error

4.header file ->contains declarations.

5.sizeof operator is executed during compile time..

6.*p+=1
*p++
are these two same?
not same.

7.func(int i)
{
static int count;
count=count+i;
}
ans:1+2+3...(counts values does not go after funtion call

8.is('a'<'b') true

10.short int=16 bits

11.4 stmt. ans.int float i;

12.int num[3];
num[3]=2;

ans:first stmt deals with size
second deals with element

13.j=4
for(int i=0;i<5;i++)
{
j++;
++j;
}
output of j.
ans:14

9.char s1[20]="hello world";
s1[5]="\0";
printf("%d",strlen(s1));
printf("%%.%...(not clear)",s1);
}
ans:bad format specifier

14.brace { used in c for what ans:convention

15.parameters in c passed by ans:value.

16.when an array is passed it is by ans:pointer.

17.scanf can read ans:any data type

18.which cant be passed to subroutine.ans:preprocessor directive.

19.to get string of words..ans:gets()

20.external variables can be accesed ans:in functions which use them.


analytical

1.cat->satc
dear->seard

sing->sings

3.1999 july 21st WEDNESDAY 52= /4 13,39 === 26+39 =65/7=2
1947 july 21st MONDAY
ans:two days before.

4.2,12,30,56,

iSoftTech PATTERN

For more info on iSoftTech visit www.isofttech.com

The test cnsisted of 3 sections:
1. Aptitude -> 60 questions(1 hr)
2. Maths -> 15 questions(30 min)
3. C or Java(there is an option) -> 30 questions(1 hr)

1. Aptitude:


Started with fill in the blanks, followed by analogies, antonyms. Next came the data sufficiency & logical reasoning questions. Then there were some basic quatitative questions. Finally the aptitude test ended with some data interpretation questions.
Antonyms of:
Increduluous
Zest
Officious

Analogies:
RANK:INSIGNIA -> PEOPLE:STATUS
FATIGUE:RESTING -> OVERWEIGHT:DIETING

2.Maths:
This part was easy but somewhat tricky.
2.1 A figure with 8 poles was given. Each pole was separated by a distance of 8m. It was given that a man runs to each pole and back. Asked to find the total distance to reach the last pole? (Ans: 288)
2.2 Are there more Sundays in a millenium or inches in a kilometre. (Ans: inches)
2.3 A gambler has the "easy go easy way" policy. He doubles his money the first time and gives away $1. He trebles his money the next time and gives away $2. Finally he is left with 4 times the amount he started with. How much money he had when he started?(Ans: $2.5 check)
2.4 Are there more centimetres in a kilometre or seconds in a day? ( Ans: centimetres)
2.5 A person purchases goods at three(3) shops successively. Each time he spends 2/3 of the amount he has. If he is left with 50 paise after the last purchase, how much money he had in the beginning?
(Ans: Rs 13.5/-)
2.6 & 2.7 Two questions on age. (very easy).
2.8 My double exceeds my half by 2. What am i? (Ans 4/3)
2.9 N packets are transmitted through a network. The probability of a packet being error prone is P. What is the probability that all packets are transmitted error free? (Ans (1-P)^N check)
2.10 Y-X^2=25 is a function. What is the y intercept? (Ans 25)
2.11 A machine has N parts. The probabilty that any part fails is P. The machine fails of any of the part fails. Fine the probabilty that none of the part fails?(Ans 1-P^N).
2.12 Mexico is 13 hours behind Singapore in time. Singapore itself is 7 hours ahead of London in time. If it is 10 minutes past 4 on Tuesday morning in London what is the time and day in Mexico?
2.13 If a hen and a half lays an egg and a half, how much will half a dozen hens lay in half a dozen days?
2.14 A person deals playing cards among people. On the first chance he deals among 4 people and 3 cards are left. Next he deals amongs 2 and 3 cards are left. (Don't remember this one exactly..)
2.15

3. C test:
3.1 A question on strings and pointers.
3.2 Which function in C allocates memory and initializes it to zero?
3.3 The compiler equates x[i][j][k] to:
*(*(*(x+i)+j)+k) : no such option was given, i think this is the correct answer.
3.4 During which phase are _STDC_ and _FILE_ defined.
Options: compiling,preprocessing,coding, assembling,linking.
3.5 What is the function clock() in C used for?
3.5 A question on macros
Options: A macro can contain parameters, macro definition can contain expressions, macros are processed during the preprocessor stage, macros are processed at run-time.
3.6 What is the value of 64>>3?
3.7 Value of 0xE & ~0x9?
3.8 Value of 0xB & 0x5?
3.9 How do you redirect the output to a particular file?
3.10 Is there any inbuilt sorting function in C for sorting character arrays?
3.11 fp=fopen(file.c,"r");
What is the correct declaration for fp?( ans: FILE *fp)
3.12 What will be the output:
int x=0;
for(;;x++){
if(x==4) break;
continue;
}
printf("%d\n",x);
3.13 What is the output:
int i=100;
do
{--i;}while(i>50);
printf("%d\n",i);
3.14 & 3.15 A question on calloc()(which one would give a runtime error) and another one on pointers
although the code looks correct whats the problem with the logic).
3.16 A qustion on converting a string defined using a character pointer to lower-case
char s[100]; (Don't remember the question
char *ptr;
ptr=s;
*ptr="Hello World"
3.17 Assume 2's complement arithmetic
int m=-14;
int n=6;
int o=m%++n;
n+=m++%o
printf("%d%d%d",m,n,o);

3.18 The type returned by sizeof is:
Options: size_t,int,unsigned int

3.19 int a=1000,b=1000; Which is the shortest statement for preventing the integer overflow exception
Options: long c=long(a)*long(b), long c=a*b, long c=long(a*b),c=(long)a*b
3.20 A short int=16 bits. What is the maximum value which can be displayed using it.
3.21 struct typedef cust_record{
char name;
}CUSTOMER_REC;
CUSTOMER_REC customer[50];
How will you access the structure variable name.
(ans:customer[i]->name check)
3.22 A question on how to flush a buffer (Don't remember exactly: fflush()).
3.23 How will you read a character from the standard input.
(Ans: c=getchar(); check)

CAN'T REMEMBER OTHERS OR MAY BE I DON'T WANT TO.



"EVERY HURDLE IS HELPING ME TO REACH THE DESTINATION"
NAKUL MUKERJEE

"AFTER TRYING KEEP TRYING"

Intergraph Placement Paper 2

QUESTION PAPER CONSIST OF TWO PARTS

ANALYTICAL SKILLS : 20 QUES, 20 MIN.

C SKILLS : 30 QUES, 30 MINS.

ANALYTICAL :

------------
1. COMPLETE THE DIAGRAM :

FOUR FIG WILL BE GIVEN , YOU HAVE TO DRAW THE FINAL ONE

TRIANGLE FIG :


2. DRAW VENN DIAGRAM RELATING RHOMBUS, QUADRILATERAL & POLYGON


3.IN A GROUP OF 5 PERSONS A,B,C,D,E ONE OF THE PERSON IS ADVOGATE,
ONE IS DOCTOR, ONE BUSINESSS MAN, ONE SHOP KEEPER AND ONE IS PROFESSOR.
THREE OF THEM A,C,AND PROFESSOR PREFER PLAYING CRICKET TO FOOT BALL AND TWO
OF THEM B AND BUSINESSMAN PREFER PLAYING FOOT BALL TO CRICKET. THE SHOP
KEEPER AND B AND A ARE FRIENDS BUT TWO OF THESE
PREFER PLAYING FOOT BALL TO CRICKET. THE ADVOGATE IS C'S BROTHER AND
BOTH PLAY SAME GAME . THE DOCTOR AND E PLAY CRICKET.

(a) WHO IS ADVOGATE ?
A, B, C, D
(b) WHO IS SHOP KEEPER ?
A, B, C, D
(C) WHICH OF THE FOLLOWING GROUP INCLUDE PERSONS WHO LIKE PLAYING CRICKET
BUT DOESN'T INCLUDE PROFESSOR ?
AB,BC,CD, NONE
(d) WHO IS DOCTOR ?
A,B,C,D.

{ SAME MODEL PROBLEM WAS ASKED IN QUES PAPER BUT PROFESSIONS WILL BE DIFFERENT
SUCH AS HORTICULTURIST ,PHYSICST,JOURNALIST,ADVOCATE AND OTHER ONE. INSTEAD OF
FOOTBALL AND CRICKET THEY WILL GIVE TEA AND COFFEE }


4. THEY WILL GIVE SOME CONDITION'S AND ASKED TO FIND OUT FARTHEST CITY IN THE
WEST (EASY ONE )?


5. TRAVELLING SALES MAN PROBLEM .
SOME CONDITION WILL BE GIVEN WE HAVE TO FIND OUT THE ORDER OF STATION THE
SALES MAN MOVES
( THREE QUES'S )

6. +,-,*, /, WILL BE GIVEN DIFFERENT MEANING
EXAMPLE : TAKE + AS * AND SO ON .
THEY WILL GIVE EXPRESSION AND WE HAVE TO FIND THE VALUE OF THAT.


7. 3+5-2 =4
WHICH HAS TO BE INTERCHANGE TO GET THE RESULT ?


8. WE DON'T NO EXACT PROBLEM .
EX : 8A3B5C7D.....
A WIIL BE GIVEN + SIGN.
B WILL BE GIVEN - SIGN.
FIND THE VALUE OF EXPRESSION ?


9. FIND THE TOTAL NUMBER OF SQUARES IN 1/4 OF CHESS BOARD ?


10. 6 FACE OF A CUBE ARE PAINTED IN A MANNER ,NO 2 ADJACENT FACE HAVE SAME
COLOUR. THREE COLURS USED ARE RED BLUE GREEN. CUBE IS CUT IN TO 36 SMALLER
CUBE IN SUCH A MANNER THAT 32 CUBES ARE OF ONE SIZE AND REST OF THEM BIGGER
SIZE AND EACH BIGGER SIDE HAVE NO RED SIDE. FOLLOWING THIS
THREE QUES WILL BE ASKED .
{ IN QUES PAPER COLORS WILL BE DIFFERENT }



11. TWO LADIES ,TWO MEN SIT IN NORTH EAST WEST SOUTH POSITION OF RECTANCULAR
TABLE. USING CLUES IDENTIFY THEIR POSITION ?


12. CLOCK PROBLEM :
(ONE QUES )


13. ALL MEN ARE VERTEBRATE.
SOME MAMMALS ARE MEN.
CONCLUDE.


C SKILLS :

1. find(int x,int y)
{ return ((xcall find(a,find(a,b)) use to find
(a) maximum of a,b
(b) minimum of a,b
(c) positive difference of a,b
(d) sum of a,b

2. integer needs 2bytes , maximum value of an unsigned integer is
(a) { 2 power 16 } -1
(b) {2 power 15}-1
(c) {2 power16}
(d) {2 power 15}

3.y is of integer type then expression
3*(y-8)/9 and (y-8)/9*3 yields same value if
(a)must yields same value
(b)must yields different value
(c)may or may not yields same value
(d) none of the above

4. 5-2-3*5-2 will give 18 if
(a)- is left associative,* has precedence over -
(b) - is right associative,* has precedence over -
(c) - is right associative,- has precedence over *
(d)- is left associative,- has precedence over *

5. printf("%f", 9/5);
prints
(a) 1.8,
(b) 1.0,
(c) 2.0,
(d) none
.
6. if (a=7)
printf(" a is 7 ");
else
printf("a is not 7");
prints
(a) a is 7,
(b) a is not 7,
(c) nothing,
(d) garbage.

7. if (a>b)
if(b>c)
s1;
else s2;
s2 will be executed if
(a) a<= b,
(b) b>c,
(c) b<=c and a<=b,
(d) a>b and b<=c.

8. main()
{
inc(); ,inc(); , inc();
}
inc()
{ static int x;
printf("%d", ++x);
}
prints
(a) 012,
(b) 123,
(c) 3 consecutive unprectiable numbers
(d) 111.

9.preprocessing is done

(a) either before or at begining of compilation process
(b) after compilation before execution
(c) after loading
(d) none of the above.

10. printf("%d", sizeof(""));
prints
(a) error
(b)0
(c) garbage
(d) 1.

11.main()
{
int a=5,b=2;
printf("%d", a+++b);
}

(a) results in syntax,
(b) print 7,
(c) print 8,
(d) none,

12. process by which one bit patten in to another by bit wise operation is
(a) masking,
(b) pruning,
(c) biting,
(d) chopping,

13.value of automatic variable that is declared but not intialized
will be
(a) 0,
(b) -1,
(c) unpredictable,
(d) none,

14. int v=3, *pv=&v;
printf(" %d %d ", v,*pv);
output will be
(a) error
(b) 3 address of v,
(c) 3 3
(d) none.

15. declaration
enum cities{bethlehem,jericho,nazareth=1,jerusalem}
assian value 1 to
(a) bethlehem
(b) nazareth
(c)bethlehem & nazareth
(d)jericho & nazareth

16. #include
#include
void main()
{
char buffer[82]={80};
char *result;
printf( "input line of text, followed by carriage return :\n");
result = cgets(buffer);
printf("text=%s\n",result);
}
(a) printf("length=%d",buffer[1]);
(b) printf("length=%d",buffer[0]);
(c) printf("length=%d",buffer[81]);
(d) printf("length=%d",buffer[2]);

17. consider scanf and sscanf function , which is true

(a) no standard function called sscanf
(b) sscanf(s,...) is equivalent to scanf(...) except that
input charecter are taken from string s.
(c) sscanf is equivalent to scanf.
(d) none of above.

18. #include
main()
{
char line[80];
scanf("%[^\n]",line);
printf("%s",line);
}
what scanf do ?
(a) compilation error . illegal format string.
(b) terminates reading input into variable line.
(c) and (d) other two options.

19. problem was big so i couldn't remember . simple one.

20 . ceil(-2.8) ?
(a) 0
(b) -3.0
(c) -2.0
(d) 2

21. for( p=head; p!=null; p= p -> next)
free(p);

(a) program run smooth.
(b) compilation error.
(c) run time error.
(d) none of above.

22. int x[3][4] ={
{1,2,3},
{4,5,6},
{7,8,9}
}
(a) x[2][1] = x[2][2] =x[2][3] = 0
(b) value in fourth column is zero
(c) value in last row is zero
(d) none of above.

23. problem was big so i couldn't remember . simple one.

24. main ()
{
printf("%u" , main());
}
(a) print garbage.
(b) execution error
(c) printing of starting address of function main.
(d) infinite loop.

25 . int a, *b = &a, **c =&b;
....
....
.....
a=4;
** c= 5;

(a) doesnot change value of a
(b) assign address of c to a.
(c) assign value of b to a.
(d) assign 5 to a.

26.problem was big so i couldn't remember . simple one.

27.problem was big so i couldn't remember . simple one.

28 . answer : swapping of values .

29 . simple one.

30 . i =5;
i= (++i)/(i++);
printf( "%d" , i);
prints ,
(a) 2
(b) 5
(c) 1
(d) 6

in interview they will ask questions related to u r project and some c
fundas.


Model paper of Inetrgraph



1 question__ on analogy
1 " _______non verbal reasoning

ALL THESE ARE NOT TOUGH .AVERAGE LEVEL.
THEY HAVE GIVEN TWO TYPES OF PAPERS.HERE I AM GIVING THE PAPER WHICH
WAS WRITTEN BY MY FRIEND.


1.there are 6 (p,q,r,s...)people.each one play one game
#NAME?
clues given like:shortest one plays hokey
largest person plays ..tennis
P,Q ARE NEITHER LARGEST NOR SHORTEST
R PLAYS FOOTBALL
s is between t &r in height
question: who plays which game
who is taaller than whom
2.a cube has colors blue,red ,yellow each on two opposite sides.cube is
divided into "32 small cubes and 4 large cubes".
question:how many cubes (on 36 cubes) have blue at leat one side.
how many cubes have colors on two sides.
3.anlogy: cell: tissue::atom:?

4.aa person sold two articles for 80 /- each.with 20% profit on one
article and 20% loss on another article, what is the loss / prifit he
will gain on both.

5.five cities. city1 is west to city3.city4 is east to city 5._____etc.
which is farthest on west side.

6.one question on: -=+,*=-,/=* etc
then 10/5*45=?

7.on ,C, paper is easy.mostly on pointers,3 question on structures,5 on
fundamentals,
8.ex:define max 10

main()
{int a,b;
int *p,*q;
a=10;b=19;
p=&(a+b);
q=&max;
} Q a)error in p=&(a+b) b)error in p=&max c)error in both d) no error


important suggestions:

1. average preparation is enough to qualify foe interview.time is
enough.
2.they are particular about academic background .
3.interview is only on personal details.no question on technical subjects
4.they may change paper sets also.
5.in outside recruitment they are asking more questions on CAD.
but not in campus.
in iitd they interviewed 5students out of 21 .selected 1,waiting list 1.
6.cut off cgpa:7.5

Intergraph Placement Paper 1

Intergraph

Interview Procedure

The information on the interview is pretty sketchy but it may consist of both technical grilling and HR interview.
Written Test consists of 40 questions . Time 45minutes. out of 40 questions 20 from 'c' and 20 from analytical ability,quantitative ability, also 1 question _ to draw venn diagram

important suggestions:

1. average preparation is enough to qualify foe interview.time is enough.
2.they are particular about academic background .
3.interview is only on personal details. no question on technical subjects
4.they may change paper sets also.
5.in outside recruitment they are asking more questions on CAD. but not in campus. in iitd they interviewed 5students out of 21 .selected 1,waiting list 1.
6.cut off cgpa:7.5
7.in interview they will ask questions related to u r project and some c fundas.

Intergraph

Analytical

Q. DRAW VENN DIAGRAM RELATING RHOMBUS, QUADRILATERAL & POLYGON

Q. IN A GROUP OF 5 PERSONS A,B,C,D,E ONE OF THE PERSON IS ADVOGATE, ONE IS DOCTOR, ONE BUSINESSS MAN, ONE SHOP KEEPER AND ONE IS PROFESSOR. THREE OF THEM A,C,AND PROFESSOR PREFER PLAYING CRICKET TO FOOT BALL AND TWO OF THEM B AND USINESSMAN PREFER PLAYING FOOT BALL TO CRICKET. THE SHOP KEEPER AND B AND A ARE FRIENDS BUT TWO OF THESE PREFER PLAYING FOOT BALL TO CRICKET. THE ADVOGATE IS C'S BROTHER AND BOTH PLAY SAME GAME . THE DOCTOR AND E PLAY CRICKET.

(a) WHO IS ADVOGATE ?
A, B, C, D
(b) WHO IS SHOP KEEPER ?
A, B, C, D
(C) WHICH OF THE FOLLOWING GROUP INCLUDE PERSONS WHO LIKE PLAYING CRICKET
BUT DOESN'T INCLUDE PROFESSOR ?
AB,BC,CD, NONE
(d) WHO IS DOCTOR ?
A,B,C,D.

{ SAME MODEL PROBLEM WAS ASKED IN QUES PAPER BUT PROFESSIONS WILL BE DIFFERENT
SUCH AS HORTICULTURIST ,PHYSICST,JOURNALIST,ADVOCATE AND OTHER ONE. INSTEAD OF
FOOTBALL AND CRICKET THEY WILL GIVE TEA AND COFFEE }


Q. THEY WILL GIVE SOME CONDITION'S AND ASKED TO FIND OUT FARTHEST CITY IN THE
WEST (EASY ONE )?


Q. TRAVELLING SALES MAN PROBLEM .
SOME CONDITION WILL BE GIVEN WE HAVE TO FIND OUT THE ORDER OF STATION THE
SALES MAN MOVES
( THREE QUES'S )

Q. +,-,*, /, WILL BE GIVEN DIFFERENT MEANING
EXAMPLE : TAKE + AS * AND SO ON .
THEY WILL GIVE EXPRESSION AND WE HAVE TO FIND THE VALUE OF THAT.


Q. 3+5-2 =4
WHICH HAS TO BE INTERCHANGE TO GET THE RESULT ?


Q. WE DON'T NO EXACT PROBLEM .
EX : 8A3B5C7D.....
A WIIL BE GIVEN + SIGN.
B WILL BE GIVEN - SIGN.
FIND THE VALUE OF EXPRESSION ?


Q. FIND THE TOTAL NUMBER OF SQUARES IN 1/4 OF CHESS BOARD ?

Q. 6 FACE OF A CUBE ARE PAINTED IN A MANNER ,NO 2 ADJACENT FACE HAVE SAME
COLOUR. THREE COLURS USED ARE RED BLUE GREEN. CUBE IS CUT IN TO 36 SMALLER
CUBE IN SUCH A MANNER THAT 32 CUBES ARE OF ONE SIZE AND REST OF THEM BIGGER
SIZE AND EACH BIGGER SIDE HAVE NO RED SIDE. FOLLOWING THIS
THREE QUES WILL BE ASKED .
{ IN QUES PAPER COLORS WILL BE DIFFERENT }


Q. TWO LADIES ,TWO MEN SIT IN NORTH EAST WEST SOUTH POSITION OF RECTANCULAR
TABLE. USING CLUES IDENTIFY THEIR POSITION ?

C SKILLS

1. find(int x,int y)
{ return ((xcall find(a,find(a,b)) use to find
(a) maximum of a,b
(b) minimum of a,b
(c) positive difference of a,b
(d) sum of a,b

2. integer needs 2bytes , maximum value of an unsigned integer is
(a) { 2 power 16 } -1
(b) {2 power 15}-1
(c) {2 power16}
(d) {2 power 15}

3.y is of integer type then expression
3*(y-8)/9 and (y-8)/9*3 yields same value if
(a)must yields same value
(b)must yields different value
(c)may or may not yields same value
(d) none of the above

4. 5-2-3*5-2 will give 18 if
(a)- is left associative,* has precedence over -
(b) - is right associative,* has precedence over -
(c) - is right associative,- has precedence over *
(d)- is left associative,- has precedence over *

5. printf("%f", 9/5);
prints
(a) 1.8,
(b) 1.0,
(c) 2.0,
(d) none
.
6. if (a=7)
printf(" a is 7 ");
else
printf("a is not 7");
prints
(a) a is 7,
(b) a is not 7,
(c) nothing,
(d) garbage.

7. if (a>b)
if(b>c)
s1;
else s2;
s2 will be executed if
(a) a<= b,
(b) b>c,
(c) b<=c and a<=b,
(d) a>b and b<=c.

8. main()
{
inc(); ,inc(); , inc();
}
inc()
{ static int x;
printf("%d", ++x);
}
prints
(a) 012,
(b) 123,
(c) 3 consecutive unprectiable numbers
(d) 111.

9.preprocessing is done

(a) either before or at begining of compilation process
(b) after compilation before execution
(c) after loading
(d) none of the above.

10. printf("%d", sizeof(""));
prints
(a) error
(b)0
(c) garbage
(d) 1.

11.main()
{
int a=5,b=2;
printf("%d", a+++b);
}

(a) results in syntax,
(b) print 7,
(c) print 8,
(d) none,

12. process by which one bit patten in to another by bit wise operation is
(a) masking,
(b) pruning,
(c) biting,
(d) chopping,

13.value of automatic variable that is declared but not intialized
will be
(a) 0,
(b) -1,
(c) unpredictable,
(d) none,

14. int v=3, *pv=&v;
printf(" %d %d ", v,*pv);
output will be
(a) error
(b) 3 address of v,
(c) 3 3
(d) none.

15. declaration
enum cities{bethlehem,jericho,nazareth=1,jerusalem}
assian value 1 to
(a) bethlehem
(b) nazareth
(c)bethlehem & nazareth
(d)jericho & nazareth

16. #include
#include
void main()
{
char buffer[82]={80};
char *result;
printf( "input line of text, followed by carriage return :\n");
result = cgets(buffer);
printf("text=%s\n",result);
}
(a) printf("length=%d",buffer[1]);
(b) printf("length=%d",buffer[0]);
(c) printf("length=%d",buffer[81]);
(d) printf("length=%d",buffer[2]);

17. consider scanf and sscanf function , which is true

(a) no standard function called sscanf
(b) sscanf(s,...) is equivalent to scanf(...) except that
input charecter are taken from string s.
(c) sscanf is equivalent to scanf.
(d) none of above.

18. #include
main()
{
char line[80];
scanf("%[^\n]",line);
printf("%s",line);
}
what scanf do ?
(a) compilation error . illegal format string.
(b) terminates reading input into variable line.
(c) and (d) other two options.

19. problem was big so i couldn't remember . simple one.

20 . ceil(-2.8) ?
(a) 0
(b) -3.0
(c) -2.0
(d) 2

21. for( p=head; p!=null; p= p -> next)
free(p);

(a) program run smooth.
(b) compilation error.
(c) run time error.
(d) none of above.

22. int x[3][4] ={
{1,2,3},
{4,5,6},
{7,8,9}
}
(a) x[2][1] = x[2][2] =x[2][3] = 0
(b) value in fourth column is zero
(c) value in last row is zero
(d) none of above.

23. problem was big so i couldn't remember . simple one.

24. main ()
{
printf("%u" , main());
}
(a) print garbage.
(b) execution error
(c) printing of starting address of function main.
(d) infinite loop.

25 . int a, *b = &a, **c =&b;
....
....
.....
a=4;
** c= 5;

(a) doesnot change value of a
(b) assign address of c to a.
(c) assign value of b to a.
(d) assign 5 to a.

26.problem was big so i couldn't remember . simple one.

27.problem was big so i couldn't remember . simple one.

28 . answer : swapping of values .

29 . simple one.

30 . i =5;
i= (++i)/(i++);
printf( "%d" , i);
prints ,
(a) 2
(b) 5
(c) 1
(d) 6

Integra Qn paper- 2007

Analytical




Inetrgraph Software Analytical

1. Given a cube, with different colors on its faces, and then is cut into 64 pieces, and the questions relate to the colors of different colored small cubes.

2. A few ladies and gents sit around table in some given order and 4 questions are about their seating arrangement with some restrictions.

3. Draw a venn diagram for 3 items : white, flowers, cloth
Ans : draw 3 circles each intersecting the other , with white kept in the middle.

4. A problem related to seating arrangement of 4 people ( 2 ladies and
2 gents) with some restrictions

5. problem related to milk with water added to it for three times
Ans: 20.5 litres

6. Problem related to diagrams . Five diagrams were given and asked to
find the sixth one.

C-Skills



1. Max number of nodes in a binary tree with height 3 is 20 : Ans: False

2. 10,20,30,40,50,60 : give the order when put in a queue and in a stack
Ans : Queue : 10,20,30,40,50,60
stack : 60,50,40,30,20,10

3. Debugging is the process of finding
Ans : logical and runtime errors

4. trace the error:
void main(){
int &a;
/* some other stuff here */
}
Ans: syntax error

5. a problem with a function named 'myValue' will be given and asked to find the value of main() for an argument of 150,
Ans : 150

6. Some problems related to 'for' loops.

7. problem on conditional inclusion.

8. problem related to depth first and breadth first search (DSA subject)

9. study the syntax of malloc function

10. locate the error
int arr (20);
Ans: syntax error.



Intera Software

Interview Procedure

The information on the interview is pretty sketchy but it may consist of both technical grilling and HR interview.
Written Test Paper consists of two sections :
1. analytical (20 marks)
2. C skills (20 marks) total time 45 min

Integra Qn paper- 2007

1 question__ on analogy
1 " _______non verbal reasoning

1.there are 6 (p,q,r,s...)people.each one play one game ---tennis,football,hockey,cricket etc clues given like:shortest one plays hokey largest person plays ..tennis P,Q ARE NEITHER LARGEST NOR SHORTEST R PLAYS FOOTBALLs is between t &r in height
question: who plays which game
who is taaller than whom

2.a cube has colors blue,red ,yellow each on two opposite sides.cube is divided into "32 small cubes and 4 large cubes".
question:how many cubes (on 36 cubes) have blue at leat one side.how many cubes have colors on two sides.
3.anlogy: cell: tissue::atom:?

4.aa person sold two articles for 80 /- each.with 20% profit on one article and 20% loss on another article, what is the loss / prifit he will gain on both.

5.five cities. city1 is west to city3.city4 is east to city 5._____etc.which is farthest on west side.

6.one question on: -=+,*=-,/=* etc then 10/5*45=?

7.on ,C, paper is easy.mostly on pointers,3 question on structures,5 on fundamentals,

8.ex:define max 10
main()
{
int a,b;
int *p,*q;
a=10;b=19;
p=&(a+b);
q=&max;
} Q a)error in p=&(a+b) b)error in p=&max c)error in both d) no error


important suggestions:
1. average preparation is enough to qualify foe interview.time is enough.
2.they are particular about academic background .
3.interview is only on personal details.no question on technical subjects
4.they may change paper sets also.
5.in outside recruitment they are asking more questions on CAD.
but not in campus.
in iitd they interviewed 5students out of 21 .selected 1,waiting list 1.
6.cut off cgpa:7.5

Optimize the below 1,2,3,4 questions for time:

1)
int i;
if i=0 then i:=1;
if i=1 then i:=0;

2)
int i;
if i=0 then i:=1;
if i=1 then i:=0;
(given that i can take only two values (1,0))

3)
int i;
if i=0 then i:=1;
else if i=1 then i:=0;
(given that i can take only two values (1,0))

4)
int m,j,i,n;
for i:=1 to n do
m:=m+j*n

5) Expand the following
a) ISDN
b) CASE
c) CSMA/CD
d) OOPS
e) MIMD

6) In the following questions, answer A,B,C,D depending on when
the errors are detected?
A if no error is detected
B if semantic and syntactic checking
C if during Code genration & Symbol allocation
D run time

a) Array overbound
b) Undeclared identifier
c) stack underflow
d) Accessing an illegal memory location

7) How many page faults will occur for below sequence of pages when LRU
page replacement algorithm is used ( The memory can only have 3pages):

1,2,3,4,2,1,5,2,4 (something like that)



8) If a CPU has 20 address lines but MMU does'nt use two of them.
OS occupies 20K. No virtual memory is supported. What is the
maximum memory available for a user program?

9) For a binary tree with n nodes, How many nodes are there which
has got both a parent and a child?

10) Understand the funda of incrementing a variable using
val++ and ++val . Some programs are given for error correction.

11) Learn datagram . (Computer networks)

12) Which of the following can be zero? (only one)
a) swap space
b) physical memory
c) virtual memory


13) What is a must for multitasking?
a) Process preemption
b) Paging
c) Virtual memory
d) None of the above

14) A question on call by value,
call by name,
call by reference.

f(x,y,z)
{
y := y+1;
z := z+x;
}
main()
{
int a,b;
a := 2
b := 2;
f(a+b,a,a);
print a;
}

what is the value of a printed for three different calls in main.

15) Using the following instructions and two registers , A&B.
find out A XOR B and put the result in A
PUSH
POP
NOR These instructions operates with A & B and puts the result in
AND A

(question basically to get XOR in terms of NOR and AND)

16) True/False questions:
1) The page size should be the power of 2.
2)

17)
int i=0;
int j=0;

loop:
if(i = 0)
i++;
i++;
j++;
if(j<= 25)
goto loop
xxx:
question1 : how many times is the loop entered

A few questions of that sort. some count fundaes. Easy one )

18) Who handles page faults?
a) OS
b) MMU
c) Hardware logic
d) etc etc....

19) For which of following is it not possible to write an algorithm.

a) To find out 1026th prime number
b) To write program for NP-complete problem
c) To write program which generates true Random numbers.
etc...

20) what is the essential requirement for an real-time systems

a) pre-emption
b) virtual memory
c) paging etc...

-- Question bank collected ,compiled and edited by BPM and Baiju.

INTER GRAPH SOFTWARE

Paper consists of two sections : 1. analytical (20 marks)
2. C skills (20 marks)total time 45
Analytical:

1. Given a cube, with different colors on its faces, and then is cut
into 64 pieces, and the questions relate to the colors of different
colored small cubes.

2. A few ladies and gents sit around table in some given order and 4
questions are about their seating arrangement with some restrictions.

3. Draw a venn diagram for 3 items : white, flowers, cloth

Ans : draw 3 circles each intersecting the other , with white kept in the
middle.

4. A problem related to seating arrangement of 4 people ( 2 ladies and
2 gents) with some restrictions

5. problem related to milk with water added to it for three times
Ans: 20.5 litres

6. Problem related to diagrams . Five diagrams were given and asked to
find the sixth one.

C-Skills:

1. Max number of nodes in a binary tree with height 3 is 20 : Ans: False
2. 10,20,30,40,50,60 : give the order when put in a queue and in a stack
Ans : Queue : 10,20,30,40,50,60
stack : 60,50,40,30,20,10

3. Debugging is the process of finding
Ans : logical and runtime errors

4. trace the error:
void main(){
int &a;
/* some other stuff here */
}
Ans: syntax error

5. a problem with a function named 'myValue' will be given and asked to
find the value of main() for an argument of 150,
Ans : 150

6. Some problems related to 'for' loops.

7. problem on conditional inclusion.

8. problem related to depth first and breadth first search (DSA subject)

9. study the syntax of malloc function

10. locate the error

int arr (20);
Ans: syntax error.

INTERGRAPH

1.non verbal question: four figures are given, find 5th figure.
ans: 1st figure.
2.draw the venn diagram for flowers, clothes & white
ans:
--------------
| |
------------ -------------
| | | | | |
| | | | | |
|flower | | | |clothes |
------------ -------------
| white |
--------------
3.there are 6 boys--a,b,c,d,e,f and 5 girls -- p,q,r,s,t. a team should be formed with 6 members--and some more conditions like c and d should sit together; p won't sit with e. there are 4 q's based on this ref: puzzle test of agarwal
4.a cube is painted with blue, yellow and black on opposite faces.it is cut into 36 cubes,of which 4 cubes are bigger than others.3 q's on this.
hint:first cut cube into 3 equal pieces(not cubes).cut first piece into 16 equal cubes(4x4).cut 3rd piece into 16 equal cubes(4x4).cut the 2nd piece (middle piece) into 4 equal cubes(bigger in size than previous 32 cubes).

5.a sales executive has to visit 6 companies--p,q,r,s,t,u.other conditions like --he should visit s before u ,he should not visit r immediately
after u etc..,
3 q's on this
ref:puzzle test of agarwal

6.4 members--2 men ,2 women are palying cards.no woman should sit in front of another woman.no man should sit in front of another man. man1 sits on west side etc..,find the position of woman2.
7.a container contains 28 litres of pure milk. then 8 litres of milk is taken out and 8 litres of water is added.then another 8 litres of mixture . is taken out and another 8 litres of water is added. this process is repeated 3rd time also.find the milk-water ratio.
8.some boys are cows.some cows are dogs.etc..,which of the following are true:
4 statements are given.
9.one q on interferences.
section 2:(c-programming)
1.q on binary tree: does a binary tree of depth 3 have 20 nodes ans:no
2.write order of traversal of nodes for depth first and breadth first searches of a 15 node tree
3.debugging is done finding
a)logical errors b)run time errors c)both a and b d)none of the above ans:c
4.sum(x)=1+ 1+2+ 1+2+3+ .....x
write the condition'?'
{
for(i=0;ifor(j=0;jsum+=z
}
5.show that a variable is the largest among the 3 variables using conditional operator
a=5,b=10,c=7
ans:(a>b)?((a>c)?a:c):b
b is the largest
there are 3,4 q's on syntax/logical/runtime errors. and 2,3 q's on for loops. there are 20 q's in each section the cut off is 11 correct answers in each section

Saturday, September 6, 2008

INFOSYS PAPER ON 21st JULY

HI FRIENDS,

I AM SWATHI FROM GNITS , GOT PLACED INTO INFY , N I HOPE THIS PAGE HELPS U ALL



TEST PATTERN:

1. APTITUDE and VERBAL

2. HR

NOTE :(we were given an application form of infy we were asked to write it before the exam had started)



APTITUDE:

All questions were of REASONING type -----30 questions(R.S Aggarwal reasoning book is more than enough)

here time management is very important

One question was time consuming for me and

that was from the puzzle test so be prepared wid all 7 models of PUZZLE TEST



we had DATA INTETERPRETATION and as i couldnt solve the 1st question properly i had to solve this

so dont waste time for the questions u cannot solve instead do the one which is easier n refer(r s aggarwal quantitative aptitude for this)

mostly bar charts,tables and pi charts are given(we were given a table)



5 questions were from FIGURES

we were given 3 figures followed by a '?'

in this the 1st two figures are have some relationship b/w them n based on that we hav to pick an answer figure which matches the 3rd figure

this was an easy one



5 questions were from SYLLOGISM

this was a bit different from wht i thought

they give 5 statements naming them a,b,c,d,e

then the answers choices were like 3 were paired eg:1)abe 2)dbe 3)aeb 4)dae

n we hav to find the pair which corretly matchs it if u do r.s aggarwal n r perfect wid all rules of syllogism i dont thing this would b tough



5 questions were of DATA SUFFICIENCY type

i felt this was easy

the questions i remember were from mensuration,length of train,ages of people



thts it in this part, we were not told the cutoff but i think its better to try for atleast 20 out of 30



VERBAL:

40 questions were there

i scored 24 marks

there were 2 RCs one was of 2 pages i attempted in the last 5 min but i did it well i believe

then we had



correction of sentences

fill in the blank with correct word

around 5to 6 passages for which we had go choose the correct summary

u hav to read these very fast

time management is very very imp. here

do not waste even one min looking here and there coz tht one min costs a loooooooot



finally i was called my name for HR n i was on the top of the world

i actually got thru wipro also but the joy of getting into a dream company is just great

one should definately experience it



HR:

One important thing is tht dont think tht its just HR

u should always remember tht there is one more hurdle to pass

in my 1st interview i hav learnt a lot "wht to do n wht not to do in the HR round" so if u hav not yet been placed then first analyze why tht has happened n try to improve on that coz there r huge oppertunities in this world

some way or the other u can get thru once u improve on ur flaws

so be confident and dont be jelous abt the ones who hav already got its a very bad thing



My HR round was coool

HR: come in swathi

me: yes sir

HR: cmon take ur seat

me: thank u sir (he had asked my resume and asked me to sign on it n also at few places on the application form)

HR: so ur father works as so n so

me: yes sir

HR: so where does he work

me: said

HR: do u hav ny bros or siss

me: said

HR: whr did he do his scholling

me: said (so be prepared wid ur family members' details also)

HR: so ur hobbies r travelling n music

me: yes sir

HR: whre hav u travelled

me: said a lot of places in India n i mentioned tht i like cold places

HR: k then which was the coldest place u saw

me: shillong in winter, then comes munnar

HR: so would there b snow fall fall

me: (me sadly)no sir yet to see such a place once

HR: do u like travelling coz of ur fathers transferable job

me: no sir even otherwise i never miss a chance to travel

HR: he gave me 3 topics and asked me to speak on 1

me: spoke abt supermarket (he was impressed)

HR: is there an incident in ur life whr u hav improved or had to changed ur work culture

me: said

HR: then she asked me abt my paper presentation(it was on face detection which comes under AI)

HR: how would u explain to a 7th class kid abt AI

me: i definately had an answer for tht so i answered n i think he was happy wid my answer

HR: k swathi there r 6 matchsticks here arrange them so that "each one touches one another"

me : i could make 5 to touch each other but not the 6th one

HR: then he asked me to make 4 equlateral triangles with those sticks

me: (the answer is to make a pyramid out of it)i almost did it then he sait to leave it aside

HR: k so wht sports do u play

me: i enjoy playing badminton and these days i hav started playing TT too n i njoy tht too

HR: do u hav ny questions

me: asked 2 questions n he had answered them very patiently

HR: k then u can leave now and cal the next person and ALL the Best

me: thank u sir



and that was it i thought i did well & finally i got through so hope this has helped u all & ALL THE BEST to Alll U Freshers

INFOSYS PAPER ON 27th JULY AT KOCHI

Hi…am Salman Ali..doin my 7th sem in B.Tech,I.T at Cochin University of Science and Technology, Kochi ,Kerala. InfoSys had com to our college last Friday,27.07.07….am so happy that with the Almighty`s grace I was able to turn that day into another Good Friday..

As u must hav read de other experiences…Infy exam is very simple. All u require is som gud concentratn,time management and ofcourse good preparation.. For preparation I mainly read the experiences many others hav posted at FRESHERSWORLD.COM which helped me tremendously…So let me say…”Jai FreshersWorld!!!!”……

Then as per their advices I went for Verbal n Nonverbal by R.S Agarwal….Also for a few sections I used I.M.S CAT notes which also helped me heavily…O.k,lets start…

Written test contains 2 parts…
1.logical
2.English

The logical part consists of 6 type of Questions each having 5 q`s…The types are
1.Data interpretation
2.Data Sufficiency
3.Syllogisms
4.Pictorial Q`s
The other two might b anything from Puzzles,Blood relation Q`s, Cubes n Dices.

As for Data interpretatn,it takes gud amt of time,so it wud be best done at last. Topics like Data Sufficiency n Syllogisms I covered from I.M.S notes…Syllogisms do not come as they r seen in agarwal…they give a pack of 6 sentences from which u hav to select the appropriate 2 statements n a conclusion. answer contains 4 options lik abd,cef,eba..etc etc…its very very simple. all u hav to do is go from the answers using trial and error method…. data suff. Q`s com from any part of aptitude. so u hav to b fast in applying your brains to c if the statements r suff or not. its not necessary 2 find the answer…tak care of time…

Pictorial Q`s may b to find the odd one or to find the missing figure. Both are very well given in agarwal(verbal n nonverbal).
For us the first ques was tat card game called Flash..its simple but takes time.i had already done one of them from my I.M.S material(Logical Reasoning).Luckily for me,the same q came.he he he..the answers were (1)74 (2) 26 (3) 18 (4)16 (5){9,9,10}…………..

The final type was related to relations…as in…5 friends stayin in different societies and each of them having a different bank account…and the q`s wer lik who has a acc wher,who lives in society X…..etc etc..it simple….

All u hav to do is tak care of ur time…40 mins for 30 q`s…b in touch wth ur watch…

Now for the English part which is 35 mins for 40 q`s…pls dnt go for the passages..keep them for the end…the other q`s are correcting sentence,fill in ,concluding a para etc etc..if u r week in English try out the verbal(agarwal)..I.M.S notes also help..otherwis it shouldn’t b a problem..its simple..but time consuming..so again..tak good good care of ur time….Try answering all..dont think thers negative marking…so mak it a blast…

Then the H.R…I was de first one to attend…it was simple…jus casual talk…actually my H.R was so cool tat I forgot it was an intervw..very relaxed..jus had a natural conversantn….

And all praise to the Almighty I was one among the 25 selected…47 from 329 had passed the written….really njoyd cz I had not even told my parents abt the exam tat day..so it was a total surprise dhamaka…..yeaaaaahhh!!!!!
nothin technical was askd…prepare for the exam well…not by reading the whole of agarwaal,but by going thru the required topics…

INFOSYS PAPER ON ON 29th JULY

Hi Friends, Im KAVITA from H.B.T.I Kanpur, recently INFOSYS Technologies visited our campus. We all r well aware of this IT giant ,its one of the best company of india n i think most of us consider IT as a dream company. I GOT SELECTED IN INFOSYS n its like a dream come true. Earlier i was a harcotian n now im a Infosian.

NOW SOMETHING IMPORTANT

WRITTEN:
APPTITUDE = 30 questions (40 min)

IT WAS EASY TO CRACK, NEED TIME MANAGEMENT, STUDY R.S AGARWAL(Verbal , non - verbal)
1. INPUT: START
STATEMENT 2 :a=38, b=43, c=45
STATEMENT3 :a=b , b=c
like wise some more statements
OUTPUT: STOP

NOW questions based on it (TOTAL 5 QUESTIONS SORRY I DONT REMEMBER ALL)
if a= a+b+c then find the output at the end of the program.

2. questions on odd figure out in non- verbal(5 diagrams will be given) very easy
3. data sufficiency(5)
4.data interpretation(a table was given but prepare bar graphs, pie charts, line graphs also)
5. logical reasoning (6 person are having sunshine aries, libra etc. the one who is tall is not a libran) Make table to solve these questions.
6. Syllogism
6 sentences given n we hv to arrange in logical order.use venn diagram

ENGLISH: 35 questions(30 min)
It was a difficult one, totally C.A.T. pattern.
-2 very big passages(one on how managers manage the things, other on E-Commerce)
-sentence corrections(based on grammer, logics)
-fill in the blanks with correct word(need very strong vocabulary)
-short passages(4-5 lines) u need to find correct inference, assumption, conclusion out of 4 sentences given.

IN OUR COLLEGE 200 STUDENTS APPEARED
CLEARED WRITTEN= 31
FINALLY SELECTED=19
I WAS ONE OF THE LUCKY PERSON

INTERVIEW:
No Technical , prepare puzzles(they ask)no puzzle was asked to me but a lot to my friends.
Frankly speaking.(they check 2 things either u should hv very good communication skills or ur apptitude should be very sharp, some of my friends who r not very good at communication answered the puzzles very effectively n they are in, so dont loose hope, show ur full confidence n definitely u will b selected.)

SOME OF THE QUESTIONS ASKED:
1.What is the difference between ball pen n gel pen
2.U BELONG to uttranchal so how do u percept haldwani, nainital and kanpur.
3.IN normal vision we write 6/6 WHAT does it mean
4.puzzle- WE ALL NEED MORE = MONEY(what is the value of N ans.is 9 )
5.5 weakness with eg.
6.5 strengths with eg.
7. if after 4 months we send u to U.S n ur parents wont allow whwt will u do.
8.why want join software as u are not from C.S or I.T.
9.they ask questions from CURRENT AFFAIRS, POLITICS, G.K also.from where pratibha patil belong, New7 wonders.

ALL THE BEST TO ALL OF U
KAVITA

Infosys Paper On 11th September 2007 AT Punjab Engineering College,Chandigarh

Frankly speaking, its not such a rocket science, it is only the hard work for a day or two... and u r selected.



Process:

1) Written paper

2) Interview



------------------------------------

1) Now,written paper is divided into two parts.. mainly Aptitude and Verbal ability.

Aptitude ------ 30 Questions------ 40 minutes

Verbal ability(VA) ------ 40 Questions------ 35 minutes

---> Our "Aptitude" paper was above average level,I mean,it was a bit time consuming. Actually it was a good aptitude paper.

I selected easiest of questions first and answerd thm.. thn accordingly, I solved some more,in order of their increasing difficulty level. but frankly speaking,I was able to solve about 16-18 questions in the regulation time. Rest I made guesses( always do it,if negative marking is not thr). U will clear the cut off without mch difficulty,if u hv genuine understanding. Other option is read R.D.Sharma(i hd nt read it).



Then our "Apti" paper was taken,and "VA" Q. paper ws given to us.



---> Now,this paper consist of 2 reading comprehensions & rest Sentence making & Simple grammer.

I attempted reading comp. in the end as they r generally time consuming and less scoring. So,first do the easy part,then the difficult one. This paper was comparitively easier than previous one. No need to do extra to clear this one.



**NOTE--- Each paper hs its individual cut-off.





After 1.5 hrs,our results wer out. Out of 110 students,about 73 wer shortlisted for inerviews.



2) We hd 6 panels in total. One person in each panel. Interviews r basically of HR type. Whr u will b asked to describe urself,abt the newspaper u read,something related to ur interest,may be some puzzles(not tough) etc.

My interview ws basically the "stressed" one type. Whr I ws put under sme sort of pressure(rude behaviour etc,it depends on ur luck and panel). My hobby ws music and singing,so i ws asked abt music and even to sing a song(wch i sung well..) .

overall,it ws a typical HR interview.

If u hv confidance,u will clear it easily. Be cool during interview (sp during solving a puzzle). My interview ws a stressed 15 min HR interview. (All other panals were relatively cool and calm.)



--------------------------------------------



The interviews wer ended at abt 11 pm and our results wer out at 11.30 pm. We wer called in audi,and names of selected candidates were announced. Out of 73,abt 62 wer selected. I ws one of them.



So friends, it is not a difficult process,bt requires confidence and faith in urself.



All the best to all sitting. Meet you in Infosys.

Infosys Paper On 1 September 2007 AT P.E.C,Chandigarh

Frankly speaking, its not such a rocket science, it is only the hard work for a day or two... and u r selected.



Process:

1) Written paper

2) Interview

------------------------------------

1) Now,written paper is divided into two parts.. mainly Aptitude and Verbal ability.

Aptitude ------ 30 Questions------ 40 minutes

Verbal ability(VA) ------ 40 Questions------ 35 minutes

---> Our "Aptitude" paper was above average level,i mean,it was a bit time consuming. Actually it was a good aptitude paper.

I selected easiest of questions first and answerd thm.. thn accordingly, i solved some more,in order of their increasing difficulty level. bt frankly speaking,i was able to solve about 16-18 questions in the regulation time. Rest i made guesses( always do it,if negative marking is not thr). U will clear the cut off without mch difficulty,if u hv genuine understanding. Other option is read R.D.Sharma(i hd nt read it


Then our "Apti" paper was taken,and "VA" Q. paper ws given to us.
---> Now,this paper consist of 2 reading comprehensions & rest Sentence making & Simple grammer.

i attempted reading comp. in the end as they r generally time consuming and less scoring. So,first do the easy part,then the difficult one. This paper was comparitively easier than previous one. No need to do extra to clear this one.



**NOTE--- Each paper hs its individual cut-off.

After 1.5 hrs,our results wer out. Out of 110 students,about 73 wer shortlisted for inerviews.



2) We hd 6 panels in total. One person in each panel. Interviews r basically of HR type. Whr u will b asked to describe urself,abt the newspaper u read,something related to ur interest,may be some puzzles(not tough) etc.

My interview ws basically the "stressed" one type. Whr i ws put under sme sort of pressure(rude behaviour etc,it depends on ur luck and panel). My hobby ws music and singing,so i ws asked abt music and even to sing a song(wch i sung well..) .

overall,it ws a typical HR interview.

If u hv confidance,u will clear it easily. Be cool during interview (sp during solving a puzzle). My interview ws a stressed 15 min HR interview. (All other panals were relatively cool and calm.)

--------------------------------------------

The interviews wer ended at abt 11 pm and our results wer out at 11.30 pm. We wer called in audi,and names of selected candidates were announced. Out of 73,abt 62 wer selected. I ws one of them.

So friends, it is not a difficult process,bt requires confidence and faith in urself.


All the best to all sitting. Meet you in Infosys.

INFOSYS PAPER ON 21 OCTOBER AT DELHI

Hello friends,i attended this off campus in delhi. the paper format was the same,the very first ques was expected 2 b of cubesbut it was a difficult one.
1. a person has property consisting of cash,some gold coins & some silver bars.each gold coin costs 4000 Rs & each silver bar costs 3000/.he
gave his first daughter 25%of his property+Rs 25000
2 his second daughter 25% of property + Rs 10000 and gave his other daughters in succession 10% of property+25000 more than he gave to his eldest daughter. 5 ques were given on this data like 2 find total no of daughters etc... i did not get the ques so i skipped it.but as there is no negative marking go for some blind guesses.
5 ques were based on figures.
5 ques on data sufficiency
5 ques on a puzzle ....PQRST are 5 members of a family.3 ladies and 2 malesall ahve diff professions and 2 ladies dont play any game....some more cluesan easy one
5 ques on syllogism do practice fm venn diagrams method

the verbal sec was an easy one .if u r fm english medium u can easily b through it..2 long passages. it is always beneficial 2 start paper fm back then try passages at the end. first c the ques then search the ans u will easily find them. so i got through d written . we were called for the interview d next day. in my panel there were 2 people.they asked me 2 introduce myself. vthen asked me which paper u read and which page. asked abt the books u have read till now...asked me to tell the story of 1. gave me 1 puzzle 2 make 81 using 8 ,14 times. i tried ,my approach was ok but i didnt reach the ans.

it lasted abt 15-20 mins then they said ok best of luck. asked mr if u have any ques ..i asked was i able 2 satisfy ur queries. they said we cant ans this ques any thing else.. i said no..

the result is 2 b mailed after 4-5 weeks ..friends just wish me luck...

By,
Parul Agarval

INFOSYS PAPER ON 31st OCTOBER AT HYDERABAD

Hi every one iam v.shashikanth sharma i had completed my btech(ece) in anantapur i had applied for infy in april n i got call in oct.so friends if u had applied for infy definitly u will get call.they will inform u before 15 days.so time is sufficient for preparation.it is conducted by MERRIT TRAC .i had given my test on 31 in hyd n cleared the written, attended the hr n waiting for the result.

now coming to the pattern it consists of two sections
1.logical reasoning(consiting of genaral analytical n di question)(30 qs 40 min)
2.english (this paper is given by infy)(40 qs 35 mins) it is online examination so please take care of time.

1.reasoning section it consisted of following sub sections
1.puzzle(1-5) very simple rank of students in the class.it will b lik ."ramesh position in the class from top is 14 n frm bottom is 15 now the no of students in the class is? the ans is 28(ie..13+1+14)"lik this five question very easy
2.figures (6-10) here five figures will b given n we have to choose the next sequence it will be TRICKY BE CARE FULL
3.datasufficiency(5qs) here they had given five statments like n option will b given frm tat we have to choose the right one the best way is"frm options"

ALL RATS R MICE
ALL ANIMALS R INTELLIGENT
MICE IS INTELLIGENT
CAT IS NOT INTELLIGENT
CAT IS NOT ANIMAL
lik this 5 qs easy but time consuming
4. datainterpretation(5qs)
here they had given the pie chart n based up on tat 5 qs actually its very confusing. i had done 4 qs but try this qs in the last minutes if u had time r else just guess them.
5.puzzle(5qs)
here very lengthy puzzle ie. they had given the data of six persons abt their names n their interesting games n their cars
they had given entire data but it is very very tricky n time consuming so b fast here dont waste more time make a table abt the information u will get it.i had done 3qs n other two guesses
6.blood relations(5 qs)
this part was some what easy we can do it
N NOW THE ONE N ONLY ONE BOOK FOR THE PREPARATION IS "VERBAL N NON VERBAL "BY RS AGARWAL N FOR DATA INTERPRETATION REFER "QUANT BY RS AGARWAL"
ALL THE ABOVE TOPICS R THER IN THAT BOOKS PRACTICE THEM ONLY EXAMPLE PROBLEMS
U CAN MAKE ITVERBAL SECTION

here comes the real battle the time is very precious here 40 qs 30 min
1.2 RCS ONE IS EASY OTHER IS TOUGH (DO THEM AT LAST)(10QS)
2.CORRECTION OF SENTENCES(REFER WREN N MARTIN R CAT TIME MATERIAL V102) (10QS)
3.CHOOSE THE COORECT ALTERNATIVE FOR THE UNDER LINED PART(10QS)( IT WILL BE VERY LENGTHY, READ CARE FULLY R ELSE U WILL B IN THE TRAP ALL THE OPTION WILL B SAME)
4. FILL THE BLANK WITH THE CORRECT ALTERNATIVE ( SOME WHAT TOUGH PART)
ALL THE THING WE CAN DO FOR ENGLISH IS TO PREPARE BASIC GRAMMER N AFTER THAT OUR LUCK.
THAT S it abt written frnds the exam had completed n we WAITED for 40 min they announced the results

i had been selected in that n the interview was then itself.
i was happy to clear the written
NOW COMING TO INTERVIEW I WAS THE SECOND PERSON THEY CALLED ME N I WENT IN TO THE ROOM MY HR WAS A LADY SHE WAS VERY SHARP OBSERVING EACH N EVERY ACTION DONE BY ME. I ENTERED N GRETED HER SHE TOLD ME TO TAKE MY SEAT N GIVE MY RESUME N HR FORM THEN SHE STARTED
SHE:K SHARMA TELL ME SOME THING ABT U ( I ANSWERED)
SHE:SHE ASKED ME HOW WAS THE PAPER N CAN U REMEMBER ANY QS IN UR PAPER I TOLD THE PUZZLE N OTHER QS IN THE PAPER N VERBAL PART I EXPLAINED MY APPROACH ALSO IN EACH N EVERY SECTION
SHE:K SO U FEEL VERY EASY
ME :YES MAM
SHE:K THEN SHE GAVE ME 2 SITUATIONS
1. IF AM THE PROJECT LEADER OF THE COMPANY N 1 DAY BEFOR THE SUBMISSION OF MY PROJECT I HAD NOTICED MANY ERRORS N IT HAS BEEN DONE 2 PEOPLE IN UR PROJECT THEN HOW U WILL REACT
2.U R MOVING IN A TRAIN N U GOT STRUCK IN THE TRAIN DUE TO FLOODS ON THE TRACK
THE TRAIN WILL NOT MOVE FOR 1 DAY .SO WAT U WILL DO WITH IN THAT ONE DAY STAYING IN THE TRAIN(FRDS HERE SHE IS ASKING HOW U WILL UTILIZE UR TIME UR MOTIVE IS NOT TO ESCAPE FRM THERE)
ME : I ANSWERED THEM
SHE:GAVE ME 5 TOPICS N TOLD TO SPEAK ON ANY ONE
SHE:TELL ME WHERE U WERE INNOVATIVE IN UR LIFE
SHE:GAVE ME A LONG PASSAGE N ASKED ME TO READ IT LOUDLY N FAST AFTER THAT WITH IN FRACTION OF SECONDS SHE ASKED ME SOME QS FRM TAT .I ANSWERED THEM N EXPLAINED THE THEME OF THE PASSAGE
SHE:HOW U HAD ORGANISED THE EVENTS IN UR COLLEGE
I ANSWERED ABT MY MECHATRONICS HELD IN MY COLLEGE
SHE:ASKED IN WHICH AREA DO U WANT TO IMPROVE
ME:TOLD THINKIN ABILITY
SHE:HOW CAN U RELATE IT TO REAL LIFE
ME:TOLD MAM AFTER COMPLETION OF THIS INTERVIEW I WILL DISCUSS MY ENTIRE INTERVIW WITH MY FRNDS SENIORS N I WILL ASK THEM WEATHER IS THERE ANY FAULT IN MY INTERVIEW N I WILL NOTICE WAT I HAD DONE N LIKE THIS I WANT TO INPROVE MY SKILLS THROUGH THINKING
SHE:NARRATED A BIG STORY .THE CONTENT WAS IF U HAD GIVEN ANY WORK BY HIGHER AUTHORITIES HOW U WILL DO.REALLY FRDS HERE SHE MADE ME TO GET IN TO THE TRAP.
SHE NARRATED THE SITUATION NEARLY FOR MORE THAN 3 MIN VERY VERY FAST
ME: ANSWERED
SHE:ANY QURIES
I ASKED SOME QUERIES ABT COMPANY
SHE: TOLD ME K NICE MEETING U N ALL THE BEST FOR UR FUTURE

TATS IT FRDS IT IS MY EXPERIENCE N ALL THE TIME SHE WAS LOOKING AT ME IN A WAY TAT MY ANSWERS R WRONG BUT REALLY THE INTERVIEW WAS VERY FAST N IT WAS LIKE RAPID FIRE ROUND SHE EXPECTED ANSWERS VERY FASTLY.SO BE CARE FULL N B CAREFULL WAT U R ANSWERING DONT B IN A HURRY KEEP SMILING EVEN IF SHE SCOULDS U N DISCOURAGES U.
THEY TOLD ME WE WILL GIVE THE RESULT WITH IN 4-5 WEEKS.IAM WATING FOR THE RESULT.GOD MUST SHOW GRACE UP ON ME.U PEOPLE ALSO PRAY FOR ME.

ALL THE BEST FOR ALL THE PEOPLE WHO R TAKING INFY TEST.
THANKS
V.SHASHIKANTH SHARMA

Infosys Solved Puzzles ( 1- 400 )

Infosys Solved Puzzles ( 1- 400 )


1.There is a escalator and 2 persons move down it.A takes 50 steps and B takes 75 steps while the escalator is moving down. Given that the time taken by A to take 1 step is equal to time taken by B to take 3 steps. Find the no. of steps in the escalator while it is staionary.

Solution (not sure):
If A takes 1 step in one second, then B takes 3 steps in one second. If A takes t1 seconds to take 50 steps, then B takes 150 steps in t1 seconds.
For B, to take 150 steps he requires t1 seconds,
Then to take 75 steps he requires t1/2 seconds.
So now, s1=50, t1 = t1 & s2=75, t2=t1/2
ans= (s1*t2 ~ s2*t1) / (t1 ~ t2) which gives 100.
so 100 steps is the answer

2. If 5/2 artists make 5/2 paintings using 5/2 canvases in 5/2 days then how many artists r required to make 25 paintings using 25 canvases in 25 days?

3. If the digits of my present age are reversed then i get the age of my son.If 1 year ago my age was twice as that of my son.Find my present age.
ans. father-73, son-37

4. There are 6561 balls out of them 1 is heavy.Find the min. no. of times the balls have to be weighed for finding out the haevy ball.
ans. 8

5. If i walk with 30 miles/hr i reach 1 hour before and if i walk with 20 miles/hr i reach 1 hour late.Find the distance between 2 points and the exact time of reaching destination is 11 am then find the speed with which it walks.
ans. 120miles and 24 miles/hr

6. There r four face cards (J,Q,K,A) all of different types(diamond,club,spade,heart) and some conditions r given.find the order of cards
ans. king -> jack -> queen -> ace
heart diamond spade club

7. If A,B,C,D,E r 5 members of a family.4 of them give true statements :
1. E is my mother in law
2. C is my son in law's brother
3. B is my father's brother
4. A is my brother's wife

Who made the stmt. and what r the realtions among them
ans. E
|
A<-->B--C
|
D
<--> denotes husband-wife
-- denotes brothers

8. The product of 5 different temperatures is 12.If all of then r integers then find all the temperatures
ans. -2,-1,1,2,3

9.There r 9 cities numbered 1 to 9.From how many cities the flight can start so as to reach the city 8 either directly or indirectly such the path formed is divisible by 3.
eg. 1368-Flights goes through 1-3-6-8.

10. If i do this puzzle i find it to be hard than the last puzzle that i did before that after that..............very complex stmt.
Is that puzzle difficult,easy,can't say or depends on the no. of puzzles

11. Replace each letter by a digit. Each letter must be represented by the same digit and no beginning letter of a word can be 0.
O N E
O N E
O N E
O N E
-------
T E N
-------
Ans: 0 =1, N = 8 ,E = 2, T = 7.

12. Ann, Boobie, Cathy and Dave are at their monthly business meeting. Their occupations are author, biologist, chemist and doctor, but not necessarily in that order. Dave just told the biologist that Cathy was on her way with doughnuts. Ann is sitting across from the doctor and next to the chemist. The doctor was thinking that Boobie was a goofy name for parent's to choose,but didn't say anything. What is each person's occupation?
Ans: Since Dave spoke to the biologist and Ann sat next to the chemist and across the doctor, Cathy must be the author and Ann the biologist. The doctor didn't speak, but David did, so Bobbie is the doctor and Dave the chemist.

13. Sometime after 10:00 PM a murder took place. A witness claimed that the clock must have stopped at the time of the shooting. It was later found that the postion of both the hands were the same but their positions had interchanged. Tell the time of the shooting (both actual and claimed).
Ans: Time of shooting = 11:54 PM Claimed Time = 10:59 PM.

14. Next number in the series is 1 , 2 , 4 , 13 , 31 , 112 , ?

Ans: 224.
No number has digits more than 4. All of them are 1 , 2, 4, 8 , 16 , 32 , 64 converted to numbers in base 5.

15. Shahrukh speaks truth only in the morning and lies in the afternoon, whereas Salman speaks truth only in the afternoon. A says that B is Shahrukh. Is it morning or afternoon and who is A - Shahrukh or Salman?
Ans: Afternoon.A is Salman.

16. Two trains starting at same time, one from Bangalore to Mysore and other in opposite direction arrive at their destination 1 hr and 4 hours respectively after passing each other. How nuch faster is one train from other?
Ans: Twice.

17. There are 6 volumes of books on a rack kept in order ( ie vol.1, vol. 2 and so on ). Give the position after the following changes were noticed. All books have been changed Vol.5 was directly to the right of Vol.2 Vol.4 has Vol.6 to its left and both weren't at Vol.3's place.Vol.1 has Vol.3 on right and Vol.5 on left. An even numbered volume is at Vol.5's place Find the order in which the books are kept now.
Ans: 2, 5,1,3,6,4.

18. I bought a car with a peculiar 5 digit numbered licence plate which on reversing could still be read. On reversing value is increased by 78633.Whats the original number if all digits were different?
Ans: Only 0 1 6 8 and 9 can be read upside down. So on rearranging these digits, we get the answer as 10968.

19. The shape in the sketch below is that of a square attached to half of a similar square. Divide it into four equal pieces.
Ans: Hint : The figure can be divided into 12 equal triangles.


20) There are two balls touching each other circumferencically. The radius of the big ball is 4 times the diameter of the small all. The outer small ball rotates in anticlockwise direction circumferencically over the bigger one at the rate of 16 rev/sec. The bigger wheel also rotates anticlockwise at N rev/sec. What is 'N' for the horizontal line from the centre of small wheel always is horizontal.

21)
1 2 3 4
+ 3 4 5 5
----------
4 6 8 9
- 2 3 4 5
----------
2 3 4 4
+ 1 2 5 4
------------
3 6 9 8
------------
Q) Strike off any digit from each number in seven rows (need not be at same place) and combine the same operations with 3 digit numbers to get the same addition. After this strike off another digit from all and add all the No.s to get the same 2 digit No. perform the same process again with 1 digit No.s. Give the ' no.s in 7 rows at each stage.

22) There is a safe with a 5 digit No. The 4th digit is 4 greater thansecond digit, while 3rd digit is 3 less than 2nd digit. The 1st digit is thrice the last digit. There are 3 pairs whose sum is 11. Find the number.
Ans: 65292.

23) there are 2 guards Bal and Pal walking on the side of a wall of a wearhouse(12m X 11m) in opposite directions. They meet at a point and Bal says to Pal "See you again in the other side". After a few moments of walking Bal decides to go back for a smoke but he changes his direction again to his previous one after 10 minutes of walking in the other(opposite) direction remembering that Pal will be waiting for to meet. If Bal and Pal walk 8 and 11 feet respectively, how much distance they would have travelled before meeting again.

24. 13 kigs and 6 libs can produce 510 tors in 10 hrs, 8 kigs and 14 libs can produce 484 tors in 12 hrs.
Find the rate of production of tors for kigs and libs. Express the answer in tors/hr.

25)
xxx)xxxxx(xxx
3xx
-------
xxx
x3x
-------
xxx
3xx
-------

Q) Find the 5 digit No.
Hint: 5 is used atleast once in the calculation.

26) A fly is there 1 feet below the ceiling right across a wall length is 30m at equal distance from both the ends. There is a spider 1 feet above floor right across the long wall eqidistant from both the ends. If the width of the room is 12m and 12m, what distance is to be travelled by the spider to catch the fly, if it takes the shortest path.

27) Ramesh sit around a round table with some other men. He has one rupee more than his right person and this person in turn has 1 rupee more than the person to his right and so on, Ramesh decided to give 1 rupee to his right & he in turn 2 rupees to his right and 3 rupees to his right & so on. This process went on till a person has 'no money' to give to his right. At this time he has 4 times the money to his right person. How many men are there along with Ramesh and what is the money with poorest fellow.

28)Question related to probabilities of removing the red ball from a basket, given that two balls are removed from the basket and the other ball is red. The basket contains blue,red,yellow balls.

29)Venkat has 1boy&2daughters.The product of these children age is 72.The sum of their ages give the door numberof Venkat.Boy is elder of three.Can you tell the ages of all the three.

30)L:says all of my other 4 friends have money
M:says that P said that exact one has money
N:says that L said that precisely two have money
O:says that M said that 3 of others have money.
P:Land N said that they have money.
All are liers.
Who has money & who doesn't have?

31)Post man has a data of name surname door number, pet name of 4 families. But only one is correct for each family. There are a set of statements & questions.

33) 4 couples have a party. Depending on the set of statements, find who insulted whom and who is the host of the party.

34) 5 women given some of their heights (tall,medium,short) Hair( long, plainted), stards(Black or Brown), sari,2 medium,2-short.Tall->no sari.Plainted->medium. Answer the combinations.

35) A person has to go both Northwards & Southwards in search of a job. He decides to go by the first train he encounters.There are trains for every 15 min both southwards and northwards. First train towards south is at 6:00 A.M. and that towards North is at 6:10. If the person arrives at any random time, what is the probability that he gets into a train towards North.

36) A person has his own coach&. Whenever he goes to railway station he takes his coach. One day he was supposed to reach the railway station at 5 O'clock. But he finished his work early and reached at 3 O'clock. Then he rung up his residence and asked to send the coach immediately. He came to know that the coach has left just now to the railway station. He thought that the coach has left just now to the railway station. He thought that he should not waste his time and started moving towards his residence at the speed of 3 miles/hr. On the way, he gets the coach and reaches home at 6 o'clock. How far is his residence from railway station.

37) Radha,Geeta & Revathi went for a picnic. After a few days they forgot the date, day and month on which they went to picnic. Radha said that it was on Thursday, May 8 and Geeta said that it was Thursday May 10. Revathi said Friday Jun 8. Now one of them told all things wrongly, others one thing wrong and the last two things wrongly. If April 1st is tuesday, what is the right day, date and month?


38. There is 66x33m rectangular area. Ram is 11/8 times faster than Krishna. Both of them started walking at opposite ends and they met at some point then, Ram said "See you in the other end" Then they continued walking. After some time Ram thought he will have tea so he turned back walked back 15 meters then he changed his mind again andcontinued walking. How much Krishna has traveled by the time they meet?

39. There are 5 burglars and once went to a bakery to rob it obviously The first guy ate 1/2 of the total bread and 1/2 of the bread. The second guy ate 1/2 of the remaining and 1/2 of the bread. The third guy ,fourth guy and fifth guy did the same. After fifth guy there is no bread left out. How many bread are there?

40. All members belonging to D are members of A.
All members belonging to E are members of D.
All members belonging to C are members of both A & D.
Some members of A does not belong to D. All members belonging to D are members of E. 5 questions are there.

41. Write each statements true or false:-
1. The sum of the 1st three statements and the 2nd false statement gives the true statement.
2.The no. of true statements false statement.
3. The sum of 2nd true statement and 1st false statement gives the first true statement.
4. There are at most 3 false statements.
5.There is no two consecutive true statements.

42. There are twelve consecutive flags at an equal interval of distance. A man passes the 8th flag in 8 seconds. How many more seconds will he take to pass the remaining 4 flags?

43. A person has to cover the fixed distance through his horses. There are five horses in the cart. They ran at the full potential for the 24 hours continuously at constant speed and then two of the horses ran away to some other direction. So he reached the destination 48 hours behind the schedule. If the five horses would have run 50 miles more, then the person would have been only 24 hours late. Find the distance of the destination.

44. A boat M leaves shore A and at the same time boat B leaves shore B. They move across the river. They met at 500 yards away from A and after that they met 300 yards away from shore B without halting at shores. Find the distance between the shore A & B.
45. A person was going through train from Bombay to Pune. After every five minutes he finds a train coming from opposite direction. Velocity of trains are equal of either direction. If the person reached Pune in one hour then how many trains he saw in the journey?

46. Food grains are to be sent to city from godown. Owner wants to reach the food grains at 11 O' Clock in the city. If a truck travels at a speed of 30km/hr then he will reach the city one hour earlier. If the truck travels at a speed of 20km/h then he will reach the city one hour late. Find the distance between the godown to city. Also with which speed the truck should travel in order to reach at exactly 11 'O clock.

47. There are five persons A,B,C,D,E whose birthdays occur at the consecutive days. Birthday of A is some days or day before C & birthday of B is exactly the same days or day after E. D is two days older than E. If birth day of C is on Wednesday then find out the birthdays of other.

48. Persons say these statements.
A says either Democratic or liberal wins the elections.
B says Democratic wins.
C says neither democratic nor liberal wins the election.
Of these only one is wrong. Who wins the election?

49. Six persons A,B,C,D,E &F went to soldier cinema. There are six consecutive seats. A sits in the first seat followed by B, followed by C and so on. If A taken on of the six seats, then B should sit adjacent to A. C should sit adjacent to A or B. D should sit adjacent to A, B or C and so on. How many possibilities are there ?

50. Suppose there are four grades A, B, C, D. (A is the best and D is the worst) 4 persons Jack, Jean, Poul and Lucy wrote the final exam and made the statements like this:-
1. Jack: If I will get A then Lucy will get D.
2. Lucy: If I will get C then Jack will get D.
Jack grade is better than Poul grade.
3. Jean: If Jean doesn't get A then Jack will not get A.
4. Poul: If Jack get A, then Jean will not get B, Lucy will get C, I won't either A or B.
If all the above statements are true, then which person will get which grade?

51. Each man dances with 3 women, Each women dances with 3 men. Among each pair of men they have exactly two women in common. Find the no. of men and women.

52. A survey was taken among 100 people to find their preference of watching t.v. programmes. There are 3 channels. Given no of people who watch at least channel 1, at least channel 2,at least channel 3, no channels at all, at least channels 1 and 3, at least channels 1 and 2, at least channels 2 and 3. Find the no of people who watched all three.

53. A bird keeper has got P pigeon, M mynas and S sparrows. The keeper goes for lunch leaving his assistant to watch the birds. Suppose p=10, m=5, s=8.
a.) When the bird keeper comes back, the assistant informs that x birds have escaped. The bird keeper exclaims oh no! all my sparrows are gone. How many birds flew away.
b.) when the bird keeper come back, the assistant told him that x birds have escaped. The keeper realised that atleast 2 sparrows have escaped. What is minimum no of birds that can escape.

54. Select from the five alternatives A,B,C,D,E. At the end of each question, two conditions will be given. The choices are to filled as follows.
A: If a definite conclusion can be drawn from condition 1.
B: If a definite conclusion can be drawn from condition 2.
C: If a definite conclusion can be drawn from condition 1 and 2.
D: If a definite conclusion can be drawn from condition 1 or 2.
E: No conclusion can be drawn using both conditions.

1. Person 1 says N<5
Person 2 says n>5.
Person 3 says 3N>20
Person 4 says 3n>10
Person 5 says N<8.
What is the value of N?

55. There are N coins on a table. There are two players A&B. You can take 1or 2 coins at a time. The person who takes the last coin is the loser. A always starts first.

1. If N=7, then
a) A can always win by taking two coins in his first chance.
b) B can win only if A takes two coins in his first chance.
c) B can always win by proper play.
d) none of the above.

2. A can win by proper play if N is equal to
a) 13 b) 37 c) 22 d) 34 e) 48
Ans: E.

3. B can win by proper play if N is equal to
a) 25 b)26 c) 32 d) 41 e) none

4. if N<4, can A win by proper play always?

56. There are 4 parties A,B,C,D. There are 3 people x,y,z. X-says A or D will win. Y-says A will not win. Z-says B or D will not win. Only one of them is true. Which party won?

57. 5 persons R,S,T,U,V are contesting for a medal. Evaluation is over English, Maths, Physics, Chemistry and Hindi. Toper will get 5 marks, least will get 1 mark. No ties any where. R get 24 and won the overall medal. V gets first in Chemistry and third in Hindi, T got consistent scores in 4 subjects. Their final standings where in the alphabetical order. What was the score of S in Chemistry.

57. There are 3 types of castes, say A,B,C. A- always tells truth, B- always false, C- alternating.
X says --> Z is of C type, I am of A type.
Y says --> X is a B type.
Z says --> X is of B type.
Who is of which type?

58. Persons A and B. Person A picks a random no. from 1 to 1000. Then person B picks a random no. from 1 to 1000. What is the probability of B getting no. greater then what A has picked?

59. Three boys and three girls brought up together. Jim, Jane, Tom, Virgina, Dorthy, XXX. They marry among themselves to form three couples. Conditions are:-
i) Sum of their ages would be the same.
ii) Virgina was the oldest.
iii) Jim was dorthy's brother.
iv) Sum of ages Jane+Jim and Tom+dorthy is same.
Give the three couples.

60. X^(1/3) - X^(1/9) =60. Solve for X.

61. X Z Y+X Y Z = Y Z X.
Find the three digits.

62. Two boats start from opposite banks of river perpendicular to the shore. One is faster then the other. They meet at 720 yards from one of the ends. After reaching opposite ends they rest for 10mins each. After that they start back. This time on the return journey they meet at 400yards from the other end of the river. Calculate the width of the river.

63. Basketball Tournament organizers decided that two consecutive defeats will knock out the team. There are 51 teams participating. What is the maximum no. of matches that can be played.

64. The Master says to his grandmaster that me and my three cousins have ages in prime nos. only. Summation of our ages is 50. Grandmaster who knows the age of the master instantly tells the ages of the three cousins. Tell the ages of three cousins.( 1 is not considered as prime no.)

65. There are two families Alens and smiths. They have two children each. There names are A,B,C,D whose ages are different and ages are less then or equal to 11. The following conditions are given:-
i) A's age is three years less then his brother's age .
ii) B is eldest among the four.
iii) C is half the age of the eldest in Alens family.
iv) The difference in sum of the ages of Alens children and smiths children is same as that of five years ago.
Find the ages of all the children.

66. a,b,c,d,e are having numerical values. There are some conditions given:-
a) a=c <=== b!=e
b) Difference between a and c as same as difference between c and b as same as difference between a and d.
c) cd.
Then find a,b,c,d,e.

67. There are six cards in which it has two king cards. all cards are turned down and two cards are opened.
a) What is the possibility to get at least one king.
b) What is the possibility to get two kings.

68. There are 5 persons a,b,c,d,e and each is wearing a block or white cap on his head. A person can see the caps of the remaining four but can't see his own cap. A person wearing white says true and who wears block says false.
i) a says i see 3 whites and 1 block.
ii) b says i see 4 blocks.
iii) e says i see 4 whites.
iv) c says i see 3 blocks and 1 white.
Now find the caps weared by a,b,c,d and e.

69. There are two women, kavitha and shamili and two males shyam, aravind who are musicians. Out of these four one is a pianist, one flutist, violinist and drummer.
i) Across aravind beats pianist.
ii) Across shyam is not a flutist.
iii) Kavitha's left is a pianist.
iv) Shamili's left is not a drummer. v) Flutist and drummer are married.

70. When Arthur is as old as his father Hailey is now, he shall be 5 times as old as his son Clarke is now. By then, Clarke will be 8 times older than Arthur is now. The combined ages of Hailey and Arthur are 100 years. How old is Clarke?

71. The seven digits in this subtraction problem are 0, 1, 2, 3, 4, 5 and 6. Each letter represents the same digit whenever it occurs.
D A D C B
- E B E G
--------------------
B F E G
--------------------
What digit is represented by each letter?

72. The Jones have named their four boys after favorite relatives; their friends, the Smiths, have done the same thing with their three boys. One of the families has twin boys. From the following clues, can you determine the families of all seven children and their ages?
i) Valentine is 4 years older than his twin brothers.
ii) Winston, who is 8, and Benedict are not brothers. They are each named after a grandfather.
iii) Briscoe is two years younger than his brother Hamilton, But three years older than Dewey.
iv) Decatur is 10 years old.
v) Benedict is 3 years younger than Valentine; they are not related.
vi) The twins are named for uncles.

73. Motorboat A leaves shore P as B leaves Q; they move across the lake at a constant speed. They meet first time 600 yards from P. Each returns from the opposite shore without halting, and they meet 200 yards from. How long is the lake?

74. On the Island of imperfection there is a special road, Logic Lane, on which the houses are usually reserved for the more mathematical inhabitants. Add, Divide and Even live in three different houses on this road (which has houses numbered from 1-50). One of them is a member of the Pukka Tribe, who always tell the truth. Another is a member of the Wotta Tribe, who never tell the truth and the third is a member of the Shalla Tribe, who make statements which are alternately true and false, or false and true. They make statements as follows:-
ADD:
1. The number of my house is greater than that of Divide's.
2. My number is divisible by 4.
3. Even's number differs by 13 from that of one of the others.
DIVIDE :
1. Add's number is divisible by 12.
2. My number is 37.
3. Even's number is even.
EVEN :
1. No one's number is divisible by 10.
2. My number is 30.
3. Add's number is divisible by 3.
Find to which tribe each of them belongs, and the number of each of their houses.

75. The names of the inhabitants of Walkie Talkie Land sound strange to the visitors, and they find it difficult to pronounce them, due to their length and a few vowel sounds they contain. The Walkie Talkie guide is discussing the names of four inhabitants –
A,B,C and D. Their names each contain upto eight syllables, although none of the four names contain the same number. Two of the names contain no vowel sounds; one contains one vowel sound; and one contains two vowel sounds. From the Guide's statements below, determine the number of syllables and vowel sounds in each of the four Walkie Talkie names:-
i) The one whose name contains two vowel sounds is not A.
ii) C's name does not contain more than one vowel sound or fewer than seven syllables.
iii) The name with seven syllables does not contain exactly one vowel sound.
iv) B and C do not have names with the same number of vowel sounds.
v) Neither the name with five syllables nor the name with seven syllables contains more than one vowel sound.
vi) Neither the name with six syllables, nor the B's name, contains two vowel sounds.

76. Two identical twins have a very unusual characteristic. One tells nothing but lies on Mondays, Wednesdays and Fridays, and tells nothing but the truth all other days. The other tells nothing but lies on Tuesdays, Thursdays and Saturdays, and tells nothing but the truth all other days. On Sundays both children speak the truth.

77. According to the information presented, which of the following conversations will be impossible.
a)Twin A : "Today you are a lier"
Twin B : "You are telling the truth"
b)Twin A : "Today you are a lier"
Twin B : "Today I am a truth teller"
c)Twin A : "Tommorow I shall be a lier"
Twin B : "That's correct"
d)Twin A : "Tommorow you will be a lier"
Twin B : "Today you are a truthteller"
e)Twin A : "Yesterday we were both truthtellers"
Twin B : "You are lying".

78. Assume that the twins followed a different set of rules, so that on a given day both told only the truth while next day both only lied, alternating days of truth telling and lying. Under these rules,which of the following conversations would be possible?
a) Twin A : "Today you are a lier"
Twin B : "That is correct"
b) Twin A : "Today you are a lier"
Twin B : "That is not so"
c) Twin A : "Tommorow we will be liers"
Twin B : "Yesterday we were truthtellers"
d) Twin A : "Tommorow we will be liers"
Twin B : "You are 1 year older than I am"
e) Twin A : "We always tell the truth"
Twin B : "We some times tell the truth".

79. If the twins are heard saying the following on the same day, which choice presents a correct statement ?
Twin A : "It is Sunday Today"
Twin B : "Yesterday was Sunday"
Twin A : "it is summer season now"
a) it is a summer sunday.
b) it is a summer monday.
c) it is Monday but not summer.
d) it is Sunday but not summer.
e) it is impossible to determine whether it is Sunday or Monday.

80. In the month of october in a year has exactly four mondays and four fridays, find what day of week wiil be on the 20th of November of that year.
Ans: 20th November was a wednesday.

81. Six persons A,B,C,D,E & F went to solider cinima. There are six conseutive seats. A sits in one of the seats followed by B, followed by C and soon. If a taken one of the six seats , then B should sit adjacent to A. C should sit adjacent A or B. D should sit adjacent to A, B,or C and soon. How many possibilities are there?
Ans: 32 ways.

83. In mathematica country 1,2,3,4....,8,9 are nine cities. Cities which form a no. that is divisible by 3 are connected by air planes. (e.g. cities 1 & 2 form no. 12 which divisible by 3 then 1 is connected to city 2). Find the total no. of ways you can go to 8 if you are allowed to break the journies.
Ans: 5.

84. ABCDE are sisters. Each of them gives 4 gifts and each receives 4 gifts No two sisters give the same combination ( e.g. if A gives 4 gifts to B then no other sisters can give four to other one.)
(i) B gives four to A.
(ii) C gives 3 to E.
How much did A,B,C,E give to D?
Ans: Donor no of gifts A 1 B - C 1 D 2

85. There are some bulbs,which are numbered from 1 to 100.all the bulbs are in on conditions. The following operations are performed:-
1. Those bulbs number which are divisible by 2 are switched OFF.
2. Those bulbs numbered which are divisible by 3 are switched ON (which are already OFF) and OFF bulbs are switched ON.
3. Similarly bulbs numbers divisible by 4 are either switched ON or OFF depending upon there previous condition.

4. This procedure is adopted till 100th bulb.
At the end there were how many bulbs which were in ON condition?
Ans: 10 ( only perfect squares ).

86. There are different numbers related with A,B,C,D,E.such that, AB*CD=EEE. E*CD-AB=CC.
Find AB*D.
Ans: BE.

87. Find the total no of 10 digits whose sum is 4.

88.Four musician problem(refer GRE BARRONS).

89.GRE BARRONS problem --> Problem number 25 to 28 page no. 4.

90. A, B, C are 3 girls and there are 770 Apples. For every 4 Apples, A takes,B takes 3. For ever 6 Apples, C takes 7 Apples? Ans: 261:145:303.

91) T, U, V are 3 friends digging groups in fields. If T & U can complete i groove in 4 days &, U & V can complete 1 groove in 3 days & V & T can complete in 2 days. Find how many days each takes to complete 1 groove individually. Ans: 24 days.

92) 4 mathematician has x apples. If he arranges them in rows of 3 one will be left. The same is the case with 5,7,9 apples. But when he arranged them in rows of 11, non will be left. Find the no. of apples.C h e ta na S
Ans: 946. (Hint: 11*6 11*11 11*16 11*21 =2E......11*76 =3D946).

93) H starts running after T reaches 1/5th they must when H reach 1/6th, if H wants win at what speed H should be run? Note: One circle is there, you show this type of problem.

94) There are 4 mothers, 4 daughters and the colour of their dresses, and they are aged 1, 2, 3 & 4. Details of the dresses are given & then it asked about the remaining dresses.

95) There are 5 levels of dolls and each of different colors & condition are given. Note: This type of problem also refer.

96) 5 student A, B, C, D, E. One student knows 5 languages. Like that up to one langauge. Conditions:-
*) Spanish is most popular langauge.
*) 3 persons knows Porchigese.
*) B & C normally speak English, but when D gathered, they switched to Spanish because that is only common between the three.
*) Only langauge common between A, B, E is French.
*) Only langauge common between C & E is Italian.

97. An escalator is descending at constant speed. A walks down and takes 50 steps to reach the bottom. B runs down and takes 90 steps in the same time as A takes 10 steps. How many steps are visible when the escalator is not operating. ANS. 150.

98. Every day a cyclist meets a train at a particular crossing. The road is straignt before the crossing and both are travelling in the same direction. Cyclist travels with a speed of 10 Kmph. One day the cyclist comes late by 25 min. and meets the train 5km before the crossing. What is the speed of the train.
60 kmph.

99. Five persons muckerjee, misra, iyer, patil and sharma, all take then first or middle names in the full names. There are 4 persons having first or middle name of kumar, 3 persons with mohan, 2 persons with dev and 1 anil.
-- Either mukherjee and patil have a first or middle name of dev or misra and iyer have their first or middle name of dev.
-- Of mukherkjee and misre, either both of them have a first or middle name of mohan or neither have a first or middle name of mohan.
-- Either iyer of sharma has a first or middle name of kumar but not both.
Who has the first or middle name of anil?
Today is Mukherjee.

101. Two turns have vertain peculiar characteristics. One of them always lies on Monday, Wednesday, Friday. The other always lies on Tuesdays, thursdays and saturdays. On the other days they tell the truth. You are given a conversation.
Person A -- Today is sunday and my name is anil.
Person B -- Today is tuesday and my name is Bill. What is today?
Today is tuesday.

102) Which of the following statements can be deduced from the information presented?
i) If it is Sunday, the twins will both say so.
ii) If it is not Sunday, one twin will give the correct day and the other will lie about everything.
iii) On any given day, only one twin will give his correct name.
a) i only.
b) i and ii only.
c) i and iii only.
d) ii and iii only.
e) i,ii and iii.

103) If the twins are heard saying the following on the same day, which choice presents a correct statement?
Twin A : "It is Sunday Today".
Twin B : "Yesterday was Sunday".
Twin A : "it is summer season now".

a) It is a summer sunday.
b) It is a summer monday.
c) It is Monday but not summer.
d) It is Sunday but not summer.
e) It is impossible to determine whether it is Sunday or Monday.

104. Logical reasoning tactics practice puzzle poetry.
1) Henny, Axie, Amie are friends. Conditions:-
a) Herry or Axies is the oldest.
b)If Axie is the oldest, Amie is the youngest.
Who is the youngest & who is the oldest?
Ans: Amie is the youngest, Axie is oldest.

105. There is a robery and four persons are suspected out of them one is
actual thief, these are the sentences said by each one of them!
A says D had done
B says A had done
C says i dddnt done
D B lied when he said that i am thief

Out of these only one man is true remaining are false
ans C is thef, D is true!

106 How many four digit numbers divisible by four can be formed using
1, 2, 3, 4; repetitions are not allowed! ans 6

107 A vender solds two things at same cost 12 RS with one item at 25%profit and other at 20%loss,by this transaction he made profit or loss by how much? Ans loss,60paise

108. Conversation between two employees is as follows:-
EMPLOYEE-1: Hello! Now your experience is twice the my experience.
EMPLOYEE-2: Exactly two times.
EMPLOYEE-1: But at the last meet, you said that your experience is thrice of my experience.
EMPLOYEE-2: That is when we met at 2 years back, your experience is thrice that of yours.
What is the experience of two employess with the company?
Ans: EMPLOYEE-1: 4 years EMPLOYEE-2: 8 years.


109 ther are four persons A,B,C,D and for languages english ,french,german,italian.
conditions
1 only one language is spoken by more than two men
2 A dont know english
3 a man can speak either french or german but not both

4 all man cannot spek in a group(no common language)
5 A can mediate when B and C want to speak with each other
6 each men can speak two languages
ans
A french italian
B english french
C german italian
D german italian

110. There are 3 women ,they having three jewells, named diamond emerald, ruby
3 women A,B,C 3 thiefs D,E,F each they had taken one jewel from each of the women
following conditions
one who had taken diamond is the bachelor and most dangerous
D 's brother in law E who is less dangerous than the thief who had stolen emerald
(this is the key from this e had stolen ruby)
D did nt stolen from B
one more condition is there

111. there were three suspects for a robbery that happend in a bank, tommy,
joy and bruceEach of them were saying that I haven't done anything and the
other two has done it.police found that tommy was lying .who is the thief.
3M (MARKS).

112. Joe started from bombay towards pune and her friend julie in opposite
direction.they meet at a point .distance travelled by joe was 1.8 miles
more than that of julie.after spending some both started there way.
joe reaches in 2 hours while julie in 3.5 hours.Assuming both were travelling
with constant speed.Wath is the distance between the two cities.

113. there were five hunters A,B,C,D,E and five animals A,B,C,D,E. Hunter
having the same name with the animal didn't kill it. Each hunter has
missed some animal.
A animal was hunt by the hunter whose name matches with animal hunt by
hunter B.
C animal was hunt by the hunter whose name matches with animal hunt by
hunter D.
E has hunt C and missed D .find out animals hunted by A,B,C. 6m.

114. A boy picks up the phone and asks "Who are you?". The voice from the other side answers "I am your mother's mother-in-law". What is the relation of the boy with the fellow speaking at the other end.

115. Imagine a rectangle. Its length = 2*width. A square of 1 inch is cut on all corners so that the remaining portion forms a box when folded. The volume of the box is _____ cubic inches. Find the original dimensions of the box.

116. 2 persons are doing part time job in a company say A and B. THe company is open for all the 7 days of the week. 'A' works every second day. 'B' works every 3rd day. If 'A'works on first june and 'B' works on second june. Find out the date on which both 'A' and 'B' will work together.

117. Consider a pile of Diamonds on a table. A thief enters and steals 1/2 of th e total quanity and then again 2 extra from the remaining. After some time a second thief enters and steals 1/2 of the remaining+2. Then 3rd thief enters and steals 1/2 of the remaining+2. Then 4th thief enters and steals 1/2 of the remaining+2. When the 5th one enters he finds 1 diamond on the table. Find out the total no. of diamonds originally on the table before the 1st thief entered.

118. Imagine 4 persons A,B,C,D. (It is a strength determining game). A found it hard, but could pull 'C' and 'D' to his side. AC and BD pairs on opposite sides found themselves equally balanced. When A and B exchanged thier positions to form pairs AD and BC, BC pair could win and pull AD to thier side. Order the 4 persons in Ascending order according to thier strengths.

119. Consider a beauty contest. 3 persons participate. Their names are Attractive, Delectable, Fascinating. They are from 3 tribes Pukkas, Wottas, Summas. Pukkas - Always speak truth. Wottas - Always speak lies. Summas - Speak truth and lies alternatively. Each of the 3 persons make 2 statements. The person who speaks truth is the least beautiful. From the statements they give and the character of the 3 tribal types, find out which person belongs to which tribe. Also find out the persons in the Ascending order of their beauty.

120. There are 5 positions-Clerk, Buyer, Cashier, Manager, Floorwalker. There are 5 persons- Mrs.Allen, Mrs.CLark, Twain, Ewing, Bernett. Conditions:
1. Clerk and cashier lunch time 11.30.to12.30.
2. Others 12.30 to 1.30.
3. Mrs.Allen and Bernett play durind lunch time.
4. Clerk and cashier share Bachlor rooms.
5. Ewing and Twain are not in good terms because one day when Twain retuned early from lunch he saw Ewing already sitting for lunch and reported about him to the manager. Find out which person holds which post.

121. There are 8 courses to be handled by faculty in 2 semesters. 4 in 1st semester and 4 in 2nd semester. The candiadates hired for the post are k, l, m, n, o. The courses are Malvino, Shakespeare, Joyce, Chauncer........... Some conditions will be given like,
1. L and N handle Shakespeare and Malvino.
2. M and O handle Malvino and Joyce.

122. A family I know has several children. Each boy in this family has as many sisters as brothers but each girl has twice as many brothers as sisters. How many brothers
and sisters are there?
ans: 4 boys and 3 girls.

123. No. of animals is 11 more than the no. of birds. If the no. of birds were
the no. of animals and no. of animals were the no. of birds( ie., interchanging no.s
of animals and birds.), the total no. of legs get reduced by one fifth (1/5).
How many no. of birds and animals were there?
ans: birds:11,animals:22

124. In a soap company a soap is manufactured with 11 parts.For making one soap you will get 1 part as scrap. At the end of the day u have 251 such scraps. From that how many
soaps can be manufactured? ans: 22 + 2+ 1 = 25.

125. 2 * * |
3 * * | No. 7 does not occur in this
---------------- |
5 * * | multiplication.
* 4 * |
* * 3 | Find the product.
---------------- |
* * * * * |
------------- |
--------------------------------------------------
ans 2 8 1
3 2 2
-----
5 6 2
5 6 2 0
8 4 3 0 0
---------
9 0 4 8 2
---------

126. There is a 5digit no. 3 pairs of sum is eleven each.Last digit is 3 times the first one.
3 rd digit is 3 less than the second. 4 th digit is 4 more than the second one.
Find the digit.
ans : 25296.

127. There are five thieves, each loot a bakery one after the other such that
the first one takes 1/2 of the total no. of the breads plus 1/2 of a bread.
Similarly 2nd, 3rd,4th and 5fth also did the same. After the fifth one no.
of breads remained are 3. Initially how many breads were there?
ans : 31.


128.There are some chicken in a poultry. They are fed with corn One sack of corn
will come for 9 days.The farmer decides to sell some chicken and wanted to hold
12 chicken with him. He cuts the feed by 10% and sack of corn comes for 30 days.
So initially how many chicken are there?

129.Two people X & Y walk on the wall of a godown in opposite direction.
They meet at a point on one side and then go ahead. X after walking for some time,
walks in opposite direction for 15 mtrs.Then again he turns back and walks
in the original direction. What distance did Y walk before they met again,
if X walks 11 mtrs by the time Y walks
8 mtrs.

130. Problem from SAKUNTALA DEVI 'PUZZLES TO PUZZLE U'.
Problem no: 23( Walking back to happiness.)


131. Find a five digit number subject to following conditions:-
a. It contains 2 prime no digits.
b. 3rd digit is the lagest.
c. 1st digit = (3 rd digit - 1).
c. Sum of 4th digit and 5th digit is less than 1st digit.
d. Value of the 5th digit lies between the value of 1st digit and 2nd digit, 5th digit is one half of the 4th digit.
Ans: 71842

132. 1, 2 , 3, 4 digits are available. How many number of 4 digited numbers which are divisible by 4. (There should be no repetition of digits in the numbers) can be formed?
Ans: 6 numbers.

133. A hill of 440 yards is there. Two competitors JACK and JILL go up the hill, first JACK reaches the topmost and immediatly starts back and meet JILL 20 yards from the topmost point. Finally JACK reaches the starting point 0.5 minutes earler than JILL. Speed while coming down is 1.5 times the speed of going up. Find the time taken by JACK for whole journey (880 yards)?
Ans: 6.3 minutes.

134. A merchant in the last day sells 2 lamps for Rs.12 price. He finds that he has got 25 % gain on one and 20% lost on the other. Did he loose or gain overall? If so how much?
Ans: 60 paise Loss.

135. 4 persons are there caledd JOHN, JACOB, PITER, and WILLIAMS. 4 laungages are there named ENGLISH, ITALIAN, GERMAN, FRENCH. Conditions:-
a. There is no common language for all.
b. Except one language, no language is spoken by more than two.
c. One can know either German or FRENCH but not both.
d. John can't speak ENGLISH But John can act as interpreter between JACOB and PITER. Jacob knows GERMAN but he can tailk with WILLIAM who doesn't know a word of GERMAN.
e. No common language between JOHN, PITER, and WILLIAMS.
Which two languages does each person speaks?

Hint: ITALIAN IS SPOKEN BY THREE PERSONS (This hint is given in Question paper)

136. 5 couples are there.
MEN: L, M, N, O, P.
WOMEN: S, T, U, V, W.
10 seats are in one row. Odd numberd seats are reserved for MEN only Like that many conditions are there. This problem is from GRE-BARRONS BOOK.


137. I participated in a race.1/5th of those who are before me are equal to 5/6th of those behind me. What were the total number of contestants in the race?

138. Find the 3 digit number. Third digit is square root of first digit. Second digit is sum of first and third digits. Find the number.

139. This problem is of time and work type. Some A and some B are able to produce so many tors in so many hours.(for example 10 A and 20 B are able to produce 30 tors per hour). Like this one more sentence was given. We have to find out the rate of working of A and B in tors/hour.

140. A and B play a game of dice between them. The dice consists of colors on their faces instead of numbers. A wins if both dice show same color. B wins if both dice show different colors. One dice consists of 1 red and 5 blue. What must be the color in the faces of other dice.(i.e how many blue and how many red?). Chances of winning for A and B are even.

141. A girl has 55 marbles. She arranges them in n rows. The nth row consists of n marbles, the (n-1)th row consists of (n-1) marbles and so on. What are the number of marbles in nth row?

142. This question is of analogy type. Some sentences regarding tastes of people to poetry are given like all who like A's Poem, like the poems of B. Like this 7 or 8 sentences were given. Questions were based on this.

143. This question is also of analogy type. Four persons are there A,B,C,D. Each of the four persons own either P,Q,R,S. 10 sentences using if clause were given. We have to find out which belongs to whom.

144) Every station in N railroad issues every other station's ticket. Some stations are added. Now they have to issue 46 more tickets. Give the No. of stations after and before added.

145) There was a race between 3 people. Me, Doug and Anne. When I take 21 steps the distance covered is equal to Doug's 24 steps and Anne's 28 steps. I take 6 steps to every 7 steps of Doug and 8 steps of Anne. Who won the race?

146)How many bearers will an explorer need for a 6 day march if each man can carry the food stuff of one man for 4 days.

147) Consider the following statements:
Albert: Dave did it.
Dave: Tony did it.
Gug: I did not do it.
Tony: Dave lied when he said that i did it.
(a)If only one out of all above statements is true, who did it?
(b)If only one out of all above statements is false, who did it?

148) A contribution of Rs. 500 was raisedfrom 500 people. The fee was as follows:
Men: Rs.3.00 each
Women: Rs. 2.00 each
Childern: 0.48 each
If number of women is more than number of men, how many childern are there?

149) Alice and Liu had some berries. The total of Alice's berries and square of number of berries with Liu is 62. The total of Liu's berries and square of number of berries with Alice is 176. How many berries does each of them have?


150) A rope ladder was left down from a ship. 12 steps of the ladder were exposed at 10:00 am. The queen who was going to visit the ship, said she would visit at 1:00 pm as she would have to climb lesser number of steps then. The tide in the sea increases from morning to afternoon at the rate of 1.2 meters per hour. The distance between any 2 steps of the ladder is 0.4 mts. How many steps will the queen have to climb?

151) 5 hunters Doe, Deer, Hare, Boarand Row kill 5 animals. Each hunter kills an animal that does not correspond to his name. Also each hunter misses a different animal which again does not correspond to his name.
a) The Deer is killed by the hunter, known by the name of the animal killed by Boar.
b) Doe is killed by the hunter, known by name of animal missed by Hare.
c) The Deer was disappointed to kill only a Hare and missed the Roe.

152) A local forecast service has accuracy of 2/3 says No rain , and Meteriological service having accuracy of 4/3 says Rain. if Preference is as no rain what is the chance of rain?


153) a) 10 1 9 2 8 3 7 4 6 5 5 6 4 7 3 8 2 _ _
b) 2 4 16 512 _
Write the next elements in the series.

154) A Man is sitting in the last coach of train could not find a seat, so he starts walking to the front coach ,he walks for 5 min and reaches front coach. Not finding a seat he walks back to last coach and when he reaches there, train had completed 5 miles. What is the speed of the train.

155) The Old car of Mary requires tyres to be changed after each 24000 km. If she wants to go for 42000 km journey then how many minimum number of tyres she will need.

156) A coin is so unbalanced that it may come both heads in 2 tosses as it may come tails in a single toss. What is the probabality of getting a head in a single toss.

157) A pen, pencil and eraser together cost $1.00. if 2E2P, and 3P>4E then what a single pen will cost?

158) A local forecast service has accuracy of 2/3 says No rain , and Meteriological service having accuracy of 4/3 says Rain. if Preference is as no rain what is the chance of rain?

159) Sherlock holmes thrwated the plan to kidnapp Mrs mary when they were questioned Mercy and his two associated shipy and rany.when they were telling the story one of them told one thing wrong and other true, the other told both true, and the last told both false. examining the following tell the roles played by each
Mercy:: 1) i wrote the ransome note
2) shipy broke into the window
rany 1) shipy wrote the ransome note
2) mercy ran away with the lady
shipy 1)i broke into the window
2)rany wrote the ran some note.

160) Tom asked kim did you like the stamps? She said yes ,me and rob too liked them. Kim again said that rob got 3 more than he would have got, if i would have kept 2 more than, what he got. Tom asked how many u gave Rob? She replied 2 more than what I got. Tell, how many stamps each rob and kim got?

161) The virgo club members used to meet every week to play cards. Each time they used to seat around a round table and for their memory they used all the possiblecombinations of postions each for a single time only. Can you tell for how many times they met?
Essays Asked
If you are given a chance to change a thing in you hometown, what would you change? Give examples why you want to do so.
Television is creating a communication gap among young generation.


162. A person needs 6 steps to cover a distance of one slab. If he increases his foot length (step length) by 3 inches he needs only 5 steps to cover the slabs length. What is the length of the each slab.
Ans: 31 inches.

163. There are 19 red balls and one black ball. Ten balls are put in one jar and the remaining 10 are put in another jar. What is the possibility that the black is in the right jar.
Ans: 1/2.

164. There is one lily in the pond on 1st june. There are two in the pond on 2nd june . There are four on 3rd june and so on. The pond is full with lilies by the end of the june.
(i) On which date the pond is half full?
Ans: 29th. --the june has 30 days).
(ii) If we start with 2 lilies on 1st june when will be the pond be full with lilies.
Ans: 29th June.

165. A lorry starts from Banglore to Mysore at 6.00 A.M, 7.00 A.M, 8.00 am.....10 pm. Similarly one another starts from Mysore to Banglore at 6.00 am,7.00 am, 8.00 am.....10.00pm. A lorry takes 9 hours to travel from Banglore to Mysore and vice versa.
(i) A lorry which has started at 6.00 am will cross how many lorries. Ans: 10.
(ii) A lorry which had started at 6.00pm will cross how many lorries. Ans: 14.

166. A person meets a train at a railway station coming daily at a particular time . One day he is late by 25 minutes, and he meets the train 5 k.m. before the station. If his speed is 12 kmph, what is the speed of the train.
Ans: 60 kmph. Refer--Shakuntala Devi Book.

167. A theif steals half the total no of loaves of bread plus 1/2 loaf from a backery. A second theif steals half the remaing no of loaves plus 1/2 loaf and so on. After the 5th theif has stolen there are no more loaves left in the backery. What was the total no of loaves did the backery have at the biggining.
Ans: 31.

168. A gardener plants 100 meters towards east, next 100 meters towards north,next 100 meters towards west. 98 meters towards east, 96 meters towards north and 96 meters towards west, 94 meters towards south. and 94 meters towards east and so on. If a person walks between the trees what is the total distance travelled by him before he reaches the center.
Ans: |---------------| | | | | | | | --------|- | ---------------------| -.


169. There are four women and 3 men. They play bridge one night. Find widow among them. Rules:
(i) wife and husband are never partners.
(ii) Wife and husand never play more than one game. One night they played four games as follows:-
1. ------ + ------ vs ------- + ---------
2. ------ + ------ vs ------- + ---------
3. ------ + --*--- vs ------- + ---------
4. ---*-- + ------ vs ------- + ---------
the woman are marked * above.
Ans: Refer Problem 21. Mind Teasers by Summers.

170. From a vessel, 1/3rd of the liquid evaporates on the first day. On the second day 3/4th of the remaining liquid evaporates. What fraction of the volume is present at the end of the second day.
Ans: 50%.

171. There is a 4 inch cube painted on all sides. This is cut down into of 1 inch cubes. What is the no of cubes which have no pointed sides?
Ans: 8.

172. Sam and Mala have a conversation. Sam says I am certainly not over 40. Mala says I am 38 and you are atleast 5 years older than me. Now, Sam says you are atleast 39. All the statements by the two are false. How old are they really?
Ans: Mala = 38 yrs; Sam = 41 yrs.

173. Ram Singh goes to his office in the city, every day from his suburban house. His driver Gangaram drops him at the railway station in the morning and picks him up in the evening. Every evening Ram Singh reaches the station at 5 O' Clock. Gangaram also reaches at the same time. One day Ram Singh started early from his office and came to the station at 4 O' Clock. Not wanting to wait for the car he starts walking home. Mangaram starts at normal time, picks him up on the way and takes him back house, half an hour early. How much time did Ram Singh walk?

174. In a railway station, there are two trains going. One in the harbour line and one in the main line, each having a frequency of 10 minutes. The main line service starts at 5 o'clock and the harbour line starts at 5.02A.M. A man goes to the station every day to catch the first train that comes. What is the probability of the man catching the first train?
Ans: 0.8.

175. A family X went for a vacation. Unfortunately it rained for 13 days when they were there. But whenever it rained in the mornings, they had clear afternoons and vice versa. In all they enjoyed 11 mornings and 12 afternoons. How many days did they stay there totally?
Ans: 18.

176. Albert and Fernandes have two leg swimming race. Both start from opposite ends of the pool. On the first leg, the boys pass each other at 18 m from the deep end of the pool. During the second leg they pass at 10 m from the shallow end of the pool. Both go at constant speed but one of them is faster. Each boy rests for 4 seconds at the end of the first leg. What is the length of the pool?

177. Each alphabet stands for one digit in the following multiplication.
T H I S
x I S
---------
X F X X
X X U X
------------
X X N X X
------------
What is the maximum value T can take?


178. If 1/4 of the time from midnight plus 1/2 of the time from now to midnight is the present time, then What is the present time? 2. In a 10 digit number, if the 1st digit number is the number of ones,2nd digit number is the number of twos, and ... so on. 10th digit is the number of zeroes, then find the number.

179. A train blows a siren one hour after starting from the station. After that it travels at 3/5th of its speed it reaches the next station 2 hours behind schedule. If it had a problem 50 miles farther from the previous case,it would have reached 40 minutes sooner. Find the distance between the two stations.

180. An army 50 miles long marches at a constant rate. A courier standing at the rear moves forward and delivers the message to the first person and then turns back and reaches the rear of the army as the army completes 50 miles. Find the distance travelled by the courier.


181. Olympic race : 4 contestants : Alan,charlie, Darren ,Brain. There are two races and average is taken to decide the winner. One person comes at the same position in both the race. Charlie always come before Darren. Brian comes first once. Alan comes third atleast once. Find the positions. Alan never comes last. Charlie & Darren comes 2nd atleast once.


182) There are 6561 number of balls in a bag. Out of which one is heavy ball. In how many minimum number of weighing you can find the heavy ball.
Ans: 8.

183) The profit made by a company in one year is enough to give 6% return on all shares. But as the preffered shares get on return of 7.5%, so the ordinary shares got on return of 5%. If the value of preferd shares is Rs 4,000000, then what is the value of ordinary shares?
Ans: Rs. 6,000000.

184) There were 50 players playing a game among themselves. Each player is out of the game when lose 3 matches. What is the number of matches should be played in order to get the winner.

185) A & B two places. C & D are two people. C started from A and D started from B. When they meet each other in the way C traveled 18 m more than D. Then C takes 13 and half a minute and D takes 24 minutes to reach the other end. What was the distance between A & B.
Ans: 126.

186) I have been hearing a girl singing a song for last two score. Song: If seven times five and three times seven is added to my age it would be as far above six nines and four as the difference between twice of my age and a score. Given-A score is 20 yrs.

187) A tourist wants to go from A to B. There are four ways to do this:-
1. To take a wagon. The wagon stops for half an hour at a station in between a & b and then goes to b.
2. To walk to B. If he leavs A at the same time the wagon leaves, he will be between by the wagon by 1 mile to reach B.
3. To walk from A at the same time the wagon leaves from A. He will arrive at the mid station at the time when the wagon is prepared to leave. He can take the wagon from there. This will take shortest time.
4. To go on upto the mid station & to walk from there. He will reach at B 15 minutes before the wagon.
What is the distance between A & B?.

188) In a train there is one brakeman, conductor, engineer & fireman. Their names are Art, John, Tom & Pete given in this order or in reverse order. You have to tell the occupation of the four, w.r.t. these conditions:-
1. Brakeman has no relatives.
2. John is older than art.
3. Engineer & fireman are brothers.
4. John is pete's nephew.
5. Fireman is not conductor's uncle.
6. Conductor is not engineer's uncle.
Ans: Pete & Tom are brothers. Tom--Father and John is his son.
Art--Brakeman.
John--Conductor.
Tom--Engineer.
Pete--Fireman.

189) There is a 18 strong building and 4 people live in it. They are dentist, lawyer, accountant, architect. Dentist floor is 5 times the lawyer's floor. Account is below dentist. If archetect moves two floors up he will be midway between dentist and account. If architect moves to midway of the building (9th floor) then he will be middle of dentist & lawyer. Ground floor can be ignored i.e. floor 0.
Ans: Dentist 15. Accountant 13. Archetect 12. Lawyer 3.

190) 4 ladis, Mrs Margarat, Mrs Price, Mrs Winter & Mrs Ellen went for marketing. Each went for 2 shops only. Their surnames are lorret, torrey, doris and marshall. One went to a hardwares shop. Two went to bank. Two went to buchers. All but dorris went to grocery etc. Who went where?

191). A software engineer starts from home at 3 pm for evening walk. He walks at a speed of 4 kmph on level ground and then at a speed of 3 kmph on the uphill and then down the hill at a speed of 6 kmph to the level ground and then at a speed of 4 kmph to the home at 9 pm. What is the distance on one way?

192). A bag contains certain number of files. Each file is numbered with one digit of 0 to 9. Suppose the person want to get the number between 1 to 2000 (or 7000 check ). How many minimum number of files should be present in the bag.
.
193). a + b + c +d = d + e + f + g = g + h + i =17.
If a = 4, what are the values of d and g. Each letter taken only one of the digit from 1 to 9.
Ans: a = 4 ,b = 2, c =6, d = 5, e = 3, f = 8, g = 1, h = 7, i = 9.

194. A frog jumps 3 ft comes back 2ft in a day. In how many day it will come out of 30ft deep well?
Ans: 28 day.

195. A-B=C
D/E=F
G+H=I
C.F=I
Ans: A=9, B=5, C=4, F=2.

196. When the actual time pass 1 hr, wall clock is 10 min behind it. When 1 hr is shown by wall clock, table clock shows 10 min ahead of 1 hr. When table clock shows 1 hr, the alarm clock goes 5 min behind it. When alarm clock goes 1 hr, wrist watch is 5 min ahead of it. Assuming that all clocks are correct with actual time at 12 noon, what will be time shown by wrist watch after 6 hr?
Ans: 5:47:32.5 (n X 60 )50/60 X 70/60 X 55/60 X 65/60.

197. A software engineer just returned from US, has eaten too much fat & put a lot of weight. Every sunday he starts walking 4 km/hr on level ground, then up at 3 km\hr, then back down hill at 6km\hr, then again on level ground at 4km\hr till he reaches his destination. If he returned home at 9 p.m., what distance did he covered?
Ans: 24 km.

198. Answer the questions from facts:-
The members of certain tribe are divided into 3 casts abhor, dravid amp; magar.
1. An abhor woman can't marry dravid man.
2. A magar woman can't marry a dravid man.
3. A son takes the caste of his father and a daughter takes caste of her mother.
5. All marriages except those mentioned, are not permitted.
6. There are no children born out of a wedlock.

199 There are 2 scales of temp A & B. It was given A varies from 14 to 133 and B varies from 36 to 87. Find the temperature, when temperature of A is equal to temp of B.
Ans: 52.5.
Let t=mx+c, c=-70 => m=51/119
a=a.51/119 -70 => a=52.5.


201. There are 4 married couples, out of which, 3 poeple in a group is needed. But there should not be his or her spouse in the group. How many groups are possible?
Ans: 32.

202. In the 4 digits 1,2,3,4, how many 4 digited numbers are possible which are divisible by 4? Repeatations are allowed.
Ans: 64.

203. Two men are going along a track of rail in the opposite direction. One goods train crossed the first person in 20 sec. After 10 min the train crossed the other person who is comming in opposite direction in 18 sec. After the train has passed, when the two persons will meet?
Ans: Approx. 72 min, check it once.

204. The no. of children, adults. The no. of adults the no. of boys. The no. of boys no. of girls. The no. of girls no. of family. Conditions:-
1. No family is without a child.
2. Every girl has at least one brother and sister.
Ans: c > a > b > g > f; 9 6 5 4 3.

205. There are 4 boys - Anand, Anandya, Madan and Murali with nic-names perich, zomie, drummy and madeena not in the same order. Some conditions.
Ans: Anand : Perich
Anandya : Drummy
Madan : Zombie
Murali : Madeena

206. There are 2 diamonds, 1 spade and 1 club and 1 ace and also 1 king, 1 jack and 1 ace are arranged in a straight line.
1. The king is at third place.
2. The left of jack is a heart and its right is king.
3. No two red colours are in consecutive.
4. The queens are separated by two cards.
Write the order of which suits (hearts ,clubs)and names (jacks queens etc.) are aranged?

207. Write each statement as true or false.
8 Marks
1. The sum of the first three statements and the second false statement gives the true statement.
2. The no. of true statements > No. of false statements.
3. The sum of second true statement and first false statement gives the first true statement.
4. There are atmost 3 false statements.
5. There are no two consequtive true statements.

208. There are 3 piles each contains 10, 15, & 20 stones. There are A, B, C, D, F, G and H persons. One man can catch upto four stones from any pile. The last man who takes will win. If first A starts next B and so on, who will win?
Ans: May be F.

209. In a certain department store the position of Buyer, Cashier, Clerk, Floorwalkar & Manager are held, though not necessarily respectively, by Evans, Ames, Conroy, Davis amp; Buyer. The cashier & the manager were roommates in college. The Buyer is bachelor, Evans & Miss Ames have only business contacts with each other. Mrs. Conroy was greatly dosappointed when her husband told her that the manager had refuged to give him a raise. Davis is going to be the best man when the clerk & the cashier are married. What position does each person held?

210. In a four team foot-ball tournament, all the teams played each Other in three rounds of matches as shown in the Table - A. Some of The results of the tournament are shown in the
Table - B. Using the Clues given below, please fill in the blank columns in the result Table - B (Goals for & Goals Against?).
Note : Two points for win, one point for draw & zero points for defeat are awarded.
Clues :
1. East zone won the tournament despite scoring one less goal than the runners-up.
2. North zone scored an odd number of goals in their first round Game.
3. South zone, who failed to score in their final match, were beaten by a two-goal margin in the first round.
4. East zone lost their match aginst west zone.
5. All four teams scored goals in the second round matches.
6. West zone scored the same number of goals against east zone as North zone scored aginst them.


211. East zone scored four goals in round two match.
Table - A (Matches Played)
Round 1
North zone vs South zone
West zone vs east zone.
Round 2
South zone vs West zone
East zone vs North zone
Round 3
South zone vs East zone
West zone vs North zone.
Table - B (Results)
Played Won Draw Lost Golas For Goals Against Points
East Zone 3 - - - ? 3 4
North Zone 3 - - - ? ? 4
West Zone 3 - - - 4 3 3
South Zone 3 - - - 2 5 1


212. In certain community, there are thousand married couples. Two thirds of the husbands who are taller than their wives are also heavier and three quarters of the husbands who are heavier than their wives are also taller. If there are 120 wives who are taller and heavier than their husbands, how many husbands are taller and heavier than their wives?

213. Both the Guptas and Sinhas have two young sons, whose ages are under Eleven. The names of the boys, whose ages rounded off to the nearest year are all different, are Rajesh, Praveen, Lalith and Prathap. Taking the ages of the boys only to the nearest year, the following statements are true:-
Rajesh is three years younger than his brother is.
Praveen is the oldest.
Prathap is 5 years older than the younger Sinha's boy.
Lalith is half as old as one of the Guptha's boys.
The total ages of the boys in each family differ by the same amount today as they did five years ago.

214. A long Division Problem:-
xx)xxxxxxxxx(xxxxxxx
xx
------
xxx
xx
------
xxx
xx
-------
xx
xx
-------
xxx
xxx
-------
-------
In the complete solution, there are four 5's. Find the missing digits.

215. Following services are operated by Asian airlines between the two are located in different countries with different time zones. As it is normally done, the time shown is the local time - viz IST & TST.
Regular Flight Supersonic Flight
Arrive Alexandria 17:10 TST 15:40 TST
Depart Alexandria 20:50 TST 22:50 TST
Arrived Rampur 23:40 IST
Is the arrival time of supersonic flight into Rampur from Alexandria same as the Arrival time of the Regular flight, assuming each Service - Regular and Supersonic maintains its own constant speed of flight.


216) A, B, C, D, E related. Four of them made these statements each:-
i) C is my son-in-law's brother.
ii) B is my father's brother.
iii) E is my mother-in-law.
iv) A is my brother's wife.
Who made these statements?


217) A ship is away from the shore by 180 miles. A plane is travelling at 10 times speed of the ship. How long from the shore will they meet?

218) A clock showing 6 o'clock takes 30 secs to strike 6 times. How long will it take to strike 12 at midnight?
Ans: 66 seconds.

219) Only boys aged > 16 wear coats. Boys aged > 15 go to watch football. Some more statements are given. What can be said about those who are watching football?

220) There are 3 societies A, B amp; C having some tractors each. A Gives B and C as many tractors as they already have. After some days B gives A and C as many tractors as they have. After some days C gives A and B as many tractors as they have. Finally each has 24 tractors. What is the original No.of tractors each had in the beginning?
Ans: A - 39. B - 21. C - 12.


221. BE * BE = ACB.
A, B, C, E are non zero numbers. Find B, E.
Ans: B=1 E=9.

222. A, B, C, D, E are having numerical values. There are some conditions given:-
a) A=C <===> B!=E
b) Difference between A and C as same as difference between C and B as same as difference between A and D.
c) C < A and C > D,
Then Find A, B, C, D, E.

223. There are six cards, in which, it has two king cards. All cards are turned down and two cards are opened.
a) What is the possibility to get at least one king?
b) What is the possibility to get two kings?

224. A person went to a shop and asked for change for 1.15 paise, but he said that he could not only give change for one rupee but also for 50p, 25p, 10p and 5p. What were the coins he had?
Ans: 1-->50p 4--->10p 1--->25p.

225. There are 3 nurses and they work altogether only once in a week. No nurse is called to work for 3 consecutive days.
Nurse 1 is off on tueseday, thursday and sunday. Nurse 2 is off on saturday. Nurse 3 is off on thursday, sunday.
No two nurses are off more than once a week. Find the day on which all the 3 nurses were on work.

226. There are 5 persons A, B, C, D, E and each is wearing a block or white cap on his head. A person can see the caps of the remaining 4 but can't see his own cap. A person wearing white says true and who wears black says false.
i) A says I see 3 whites and 1 black.
ii) B says I see 4 blacks.
iii) E says I see 4 whites.
iv) C says I see 3 blacks and 1 white.
Now Find the caps weared by A, B, C, D and E.

227. There are two women, Kavitha and Shamili and two males Shyam and Aravind, who are musicians. Out of these four one is a Pianist, one Flutist, Violinist and Drummer.
i) Across Aravind beats Pianist.
ii) Across Shyam is not a Flutist.
iii) Kavitha's left is a Pianist.
iv) Shamili's left is not a Drummer.
v> Flutist and Drummer are married.

228. 1/3 rd of the contents of a container evaporated on the 1st day. 3/4 th of the remaining contents of the container evaporated the second day. What part of the contents of the container are left at the end of the second day?

229. A man covered 28 steps in 30 seconds but he decided to move fast and covered 34 steps in 18 seconds. How many steps are there on the escalator when stationary?

1. Person1: Most of us are satch J.
Person2: Most of us are jute S.
Person3: Two of us are satch J.
Person4: Three of us are jute J.
Person5: I am satch J we have to find who is satch and who is jute.
Ans: S: Satch J:Jute.

2. Four persons are there to cross a bridge they have one torch light.
Person A can cross in 1 min.
Person B can cross in 2 min.
Person C can cross in 5 min.
Person D can cross in 10 min.
They have to cross bridge with in 17 min. At a time only two persons can cross.
I. A & B ----> 2.
II. A <---- 1.
III. C & D ----> 10.
IV. B <---- 2.
V A & B ----> 2
______________
Total 17.

230. What is the maximum number of slices can you obtain by cutting a cake with only 4 cuts?
Ans: 16.

231. Three are three boxes. In first box, two white balls. In second box, 2 black balls. In third box, 1 white & 1 black ball. The lables on the boxes are not correct. Then you have to open one box and to find the colour of the balls in all boxes.
Ans: Open the box labled black & white. If white balls are there then the box labled with white balls contain black balls and labled with black balls contain one black and one white ball and vice versa, if two black balls are there.

232. There is a 4 inch cube painted on all sides. This is cut into number of 1 inch cubes. What is the number of cubes which have no painted sides?

233. Sam and Mala have a conversation. Sam says I am certainly not over 40. Mala says I am 38 and you are atleast 5 years older than me. Now Sam says you are atleast 39. All the statements by the two are false. How old are they realy?

234. Ram singh goes to his office in the city every day from his suburban house. His driver Mangaram drops him at the railway station in the morning and picks him up in the evening. Every evening Ram singh reaches the station at 5 o'clock. Mangaram also reaches at the same time. One day Ram singh started early from his office and came to the station at 4 o'clock. Not wanting to wait for the car he starts walking home. Mangaram starts at normal time, picks him up on the way and takes him back house, half an hour early. How much time did Ram singh walked?

235. Some people went for vaction. Unfortunately it rained for 13 days when they were there. But whenever it rained in the morning, they had clean afternood and vice versa. In all they enjoyed 11 morning and 12 afternoons. How many days did they stay there totally?


236. Geoffrey, Hallmann and Molly attend and interview and give three different statements each.
To make it a little complex. Out of the three statements made by each one, one is false.
Geoffrey says: I am 22 - false
Hallmann is elder than me by one year.
Molly is 25

Hallmann says: I am not the youngest -
Geoffrey is 2 years younger than me
Molly is younger than me by one year - false

Molly says: Geoffrey is 23
I am a year younger than Geoffrey
(I don't remember this statement but it is false.)

Answer:
Goeffrey is 22 as Goef states,
And Goef== 23 as Molly states.
Both cannot be true.
Either one has to be true or either one has to be false.
Start deriving from this point and u get,

Geoffrey Hallmann Molly
Age: 23 25 22


237. There is a cube, which has to be inscribed with the following pair
of numbers on opposite sides.1 and 6, 2 and 4,3 and 5.
How many different ways can it be done?

238. There's an electric wire running 1 km from the side of a building.
The number of poles in between them is placed in an interval of
distance between each other.
If one pole is removed then the distance between each pole becomes 1 2/3 meters.
Find out how many poles were kept.

239. The time taken to travel in train from Town A to Town B is 5 hours.
There are trains starting from both towns at an interval of 1 hour.
How many trains meet in 1 trip?
Ans : 10 trains check it as trains come from both sides every hour.

240. Shadow went to an Isle where the natives lie and the visitors speak truth.
Shadow saw a salesman and wanted to know whether he was a native or a visitor.
He did not pose a question directly but asked him indirect instead.
Shadow saw a woman and asked the salesman,"Is that a NATIVE or
VISITOR?" .For which the salesman replied," She is a visitor".
Is the salesman a Native or a Visitor?
Ans :
Since Shadow himself saw him/her as a woman and asked the salesman.
The Salesman replied ,"SHE " by which he speaks truth and is a Visitor.
Salesman is a Visitor.


241. Three friends divided some bullets equally.
After all of them shot 4 bullets the total number of bullets remaining is equal to the bullets each had after division.Find the original number divided.

Ans: 18 (2 marks)

Initially . x x x
Now x-4 x-4 x-4
Equation is 3x-12 = x



242. There are 3 societies A, B, C.
A lent cars to B and C as many as they had already.
After some time B gave as many tractors to A and C as many as they have.
After sometime c did the same thing. At the end of this transaction each one of them had 24.
Find the cars each orginally had.
Ans: A had 39 cars, B had 21 cars & C had 12 cars

243. The Bulls, Pacers, Lakers and Jazz ran for a contest.
Anup, Sujit, John made the following statements regarding results.
Anup said either Bulls or Jazz will definitely win
Sujit said he is confident that Bulls will not win
John said he is confident that neither Jazz nor Lakers will win
When the result cameit was found that only one of the above three had made a correct statement.
Who has made the correct statement and who has won the contest.
Ans: Sujith; Lakers

244. Five people A ,B ,C ,D ,E are related to each other.
Four of them make one true statement each as follows.

(i) B is my father's brother.
(ii) E is my mother-in-law.
(iii)C is my son-in-law's brother
(iv)A is my brother's wife.

Ans: (i) D (ii) B (iii) E (iv) C

245. Some statements are given below:

L says all of my other four friends have money
M says that P said that exactly one among them has money
N says that L said that precisely two among them have money
O says that M said that three of the others have money
P, L and N said that they have money
All the above statement are false..
Who has money & who doesn't have any money?

246. Fifty minutes ago if it was four times as many minutes past three o'clock,how many minutes is it to six o'clock?
Ans: Twenty six minutes.

247. A hotel has 10 storeys.Which floor is above the floor below the floor, below the floor above the floor, below the floor above the fifth.
Ans: The sixth floor.

248. Seven members sat around a table for three days for a conference.
The member's names were Abhishek, Amol, Ankur, Anurag,Bhuwan ,Vasu and Vikram.
The meetings were chaired by Vikram.
On the first evening members sat around the table alphabetically.
On the following two nights, Vikram arranged the seatings so that he could have Abhishek as near to him as possible and abesent minded Vasu as far away as he could.http://www.ChetanaS.com
On no evening did any person have sitting next to him a person who had previously been his neighbour. How did Vikram manage to seat everybody to the best advantage on the second and third evenings?

Ans:
Second evening:Vikram,Ankur,Abhishek,Amol,Vasu,Anurag and Bhuwan.
Third evening :Vikram,Anurag,Abhishek,Vasu,Bhuwan,Ankur,Amol.


249. Two trains start from stations A and B spaced 50 kms apart at the same time and speed.
As the trains start, a bird flies from one train towards the other and on reaching the second train, it flies back to the
first train.This is repeated till the trains collide.
If the speed of the trains is 25 km/h and that of the bird is 100km/h.
How much did the bird travel till the collision.
Ans: 100 kms.

250. Four prisoners escape from a prison.
The prisoners, Mr East, Mr West, Mr South, Mr North head towards different directions after escaping.The following information of their escape was supplied:

The escape routes were The North Road, South Road, East Road and West Road.
None of the prisoners took the road which was their namesake.
Mr.East did not take the South Road
Mr.West did not the South Road.
The West Road was not taken by Mr.East
What road did each of the prisoners take to make their escape?

Ans: Mr.East took the North Road
Mr.West took the East Road
Mr.North took the South Road
Mr.South took the West Road.

251. Complete the series:
5, 20, 24, 6, 2, 8, ?

Ans: 12 (as 5*4=20, 20+4=24, 24/4=6, 6-4=2, 2*4=8, 8+4=12).

252) A soldier looses his way in a thick jungle. At random he walks from his camp but mathematically in an interesting fashion. First he walks one mile East then half mile to North. Then 1/4 mile to West, then 1/8 mile to South and so on making a loop.
Finally how far he is from his camp and in which direction.

Ans: Distance travelled in north and south directions
1/2 - 1/8 + 1/32 - 1/128 + 1/512 - and so on
= 1/2/((1-(-1/4))
Similarly in east and west directions
1- 1/4 + 1/16 - 1/64 + 1/256 - and so on
= 1/(( 1- ( - 1/4))
Add both the answers


253) How can 1000000000 be written as a product of two factors neither of them containing zeros
Ans: 2 power 9 x 5 power 9

254) Conversation between two mathematcians:
First : I have three childern. The product of their ages is 36.
If you sum their ages, it is exactly same as my neighbour's door number on my left.
The second mathematician verfies the door number and says that it is not sufficient.
Then the first says " Ok one more clue is that my youngest is really the youngest". Immmediately the second mathematician answers . Can you answer the question asked by the first mathematician?
What are the childeren ages? Ans 1,6 and 6

255) Light glows for every 13 seconds . How many times did it glow between 1:57:58 and 3:20:47 am.
Ans : 383 + 1 = 384

256) 500 men are arranged in an array of 10 rows and 50 columns according to their heights.
Tallest among each row of all are asked to fall out.
And the shortest among them is A.
Similarly after resuming that to their original podsitions that the shortest among each column are asked to fall out.
And the tallest among them is B .
Now who is taller among A and B ?
Ans A

257) A person with some money spends1/3 for cloths, 1/5 of the remaining for food and 1/4 of the remaining for travel.
He is left with Rs 100/- . How much did he have with him in the begining ?
Ans: Rs 250/-

258) There are six boxes containing 5 , 7 , 14 , 16 , 18 , 29 balls of either red or blue in colour.
Some boxes contain only red balls and others contain only blue.One sales man sold one box
out of them and then he says " I have the same number of red balls left out as that of blue ".
Which box is the one he solds out ?
Ans: Total no of balls = 89 and (89-29 /2) = 60/2 = 30
and also 14 + 16 = 5 + 7 + 18 = 30

260) Grass in lawn grows equally thick and in a uniform rate.
It takes 24 days for 70 cows and 60 days for 30 cows to eat the whole of the grass.
How many cows are needed to eat the grass in 96 days.?
Ans : 20
g - grass at the beginning
r - rate at which grass grows, per day
y - rate at which one cow eats grass, per day
n - no of cows to eat the grass in 96 days
g + 24*r = 70 * 24 * y
g + 60*r = 30 * 60 * y
g + 96*r = n * 96 * y
Solving, n = 20.

261)There r some bees in a garden..1/5th of them went to a particular
flower,1/3rd went to another flower,3 times the difference of the
above two went to third flower..n one was remaining n it was roaming
around..how many bees were there? (3 marks)
Ans:15

262)there was a community in which there were 1000 couples.In that
2/3rd of men who r taller r also heavier n 3/4th of the men who r
heavier r also taller n there were 120 women who were both heavier n
taller than men.So how many men r both taller n heavier than men?

263)A man drives with constant speed..n he after some time he sees a
milestone with 2-digits..then he travels for an hr n sees the same 2
digits in reverse order..n then after an hr he sees that the
milestone has the same 2 digits with a 0 between them..so whats the
man speed?
Ans:45km/hr

264)There were 2 systems A n B.14 degrees in A is equivalent to 36 in
system B.and 133 in A is equivalent to 87 in B.now what is the
temperature where they both r equal?
Ans:51.25 (conversion A=(7/3)B-70)

265) X Y Z X Y Z
A B + A B -
------------ -------------
C D E F B G A

find X,Y,Z,G

266)A women buys some shoestrips n then 4 times of that she buys
packet pins n then 8 times of shoestrips she buys handkerchiefs..n
she has a bill of Rs3.24..n she pays for each article as many paise
as there r articles(of tht particular item).Now whats the number of
handkerchiefs?
Ans:16

267)Ms.Anitha got her salary n she spent half of it in shopping n gave
1RS to a beggar.After that 1/2 of the remaining money she spends in
a hotel n she gives Rs.2/- as a tip to waiter.n then 1/2 of the
remaining she spends again n she gives 3 RS as charity..n after that
finally she is left with Rs 1/- when she comes out.Whats the actual
money she had?
Rs 42/- (3 marks)


268. A person is cycling in a circular track.At some point he notices
that 1/5 of people in front of him and 5/6 of people together
condtitute the total no. of cyclists.Find the total no. of cyclists.
Ans:31

269.Trains leave from New York to Washington every hour on the
hour(1:00,2:00....).Trains leave from Washington to New York every
hour on the hour and half hour(1:00,1:30,2:0,2:30....).It takes a train 5
hrs to complete its journey from Washington to New York as well as
from new york to washington.A train leaves from new york to washington. Find
out how many trains it will meet before it reaches washington.
Ans:19 or 21(not sure).

270) A pen, pencil and eraser together cost $1.00. if 2E2P, and P>4E then what a single pen will cost?

271. In a class there are less than 500 students . when it is divided by 3 it gives a whole number. similarly when it is divided by 4,5 or 7 gives a whole number.find the no. of students in the class.
ans: 420

272. Uncle reuben and aunt cynthia came to town to shop reuben bought a suit and hat for $15
cynthia paid as much as for her hat as reuben did for his suit then she spent the rest of their
money for a new dress on the way home cynthia called reuben's attention to the fact that his hat
cost $1 more than her dress then she added if we had divided our hat money differently so that
we bought different hats mine costing 1 and 1/2 time cost if yours then we each would have spent
the same amount of money in that case said uncle reuben "how much would my hat have cost"
Ans : Uncles hat costs $6.4 (total money was $29)


273. Four family names are given and their frends name are given but not in order.( u hav to find
which frend belongs to which family) Each frend prepares salad using 3 different fruits . they are
given apple, cherry, grape, banana., no two frens uses the same combination.
Various conditions are given and 4 ques asked.
Which fruit did mandy didn't use?
Who is flures friend?
Name the fruits common btwn Erica and stacy?

274.Here is a simple mathematical puzzle set by Longfellow in his own flowery, poetical language.
If 1/5th of a hive of bees flew to the badamba flower, 1/3rd flew to the slandbara, 3 times the
difference of these two numbers flew to an arbour, ad one bee continued to fly about, attracted on each side by the fragrant ketaki and malati, what was the total number of bees?
Ans: 15

275. (don't remember the exact question)a man while sorting files picks up file number one,misses
one, picks up file no. 2,mises two files,and so on...after that he found that he picked up 5% of the
files. How many files were there?
Ans:39

276. A man walks at 4 km/hr on plain, then at 3 km/hr uphill and then returns through the same road at 6 km/hr downhill and at 4 km/hr on the plain. It takes altogether 6 hours. So what distance he covered in one way?
Ans: 12 km
Solution: Let plain road = x km
And hill road = y km
? x/4 + y/3 + y/6 + x/4 = 6
? x/2 + y/2 = 6
? x + y =12

277. There are some men and some lodges for which the
following conditions hold true –
i) Each lodge is represented by exactly 3 men .
ii) Each man is associated with exactly 2 lodges.
iii) Any pair of lodge has only one man in common.
How many men and how many lodges were there?

Ans: 6 men and 4 lodges.
Solution:
i) L1 is represented by M1, M2, M3. L2 is represented by M3, M4, M5. L3 is represented by M1, M4, M6. L4 is represented by M2, M5, M6.
ii) M1 is associated with L1,L3. M2 is associated with L1,L4. M3 is associated with L1,L2. M4 is associated with L2,L3. M5 is associated with L2,L4. M6 is associated with L3,L4.
iii) Common between L1 and L2 is only M3. Common between L1 and L3 is only M1. Common between L1 and L4 is only M2. Common between L2 and L3 is only M4. Common between L2 and L4 is only M5. Common between L3 and L4 is only M6.
i.e all the given conditions are satisfied.

278. A person sells 2 items for Rs. 12 each. For one he
profits 25% and for the other he losses 20%. Altogether did he loss or gain? And by how much?

Ans: He losses by 60 paise.
Solution: Cost price of the item for which he losses = Rs.
12*100/80 = Rs. 15.
Cost price of the item for which he Gains = Rs.
12*100/125 = Rs 9.6.
? Total cost price is Rs. 24.6 and total sell price is
Rs. 24.
So altogether he losses by Rs. 0.6.

279. My father’s age was x in the year x². I am obviously talking about 20th century. In which year was my father born?

Ans: In the year 1892.
Solution: x = 44 as 44² = 1936 and the square of any other
integer will not fall in between 1900 and 2000. i.e my
father was 44 in the year 1936. ? my father was
born in the year 1892.

280.A man was going by cycle. After going 2/3rd of total distance the cycle broke down and he had to complete the journey on foot. At the end he found that he walked twice as long as he was on cycle. How many times the speed of the cycle is as the speed of walking?

Ans: 4 times.
Solution: Let the distance be z km, speed of cycle be x km/hr
and walking speed be y km/hr.
Then he covered 2z/3 km by cycle in 2z/3x hr and
covered z/3 km on foot in z/3y hr.
? according to the question 2*2z/3x = z/3y ? x/y = 4.

281.In a badminton tournament a team is eliminated from the tournament if it losses 2 games. If there are 51 teams then what is the maximum number of games required to select the champion?

Ans: 101
Solution: To eliminate the 50 teams 50*2 = 100 games are
required. And the champion team may lose in at
most 1 game. ? Max. no. of games required = 101.

282. There are 3 tribes in a Island. Sororean who always speak truth, Nororean who always speak false and Midorean who speak truth and false alternately in either order. From the statements given by A,B and C (who belong to 3 different tribes), three persons from the island, identify who belongs to which tribe.

A : C is Sororean.
B is Midorean.

B : A is Nororean.
C is Midorean.

C : A is Midorean.
I am Sororean.

Ans:
A B C
Midorean Nororean Sororean
Solution: Think logically.

283. There are 5 persons A,B,C,D,E . All of them have different occupations and none of them are of same age. There professions are Consultant, Planner, Engineer, Nutritionist and Technician definitely not in the same order. From the following conditions determine whose profession is what?

i) The consultant is the oldest among them.
ii) The Technician is not younger than the Planner and the Planner is not younger than the Nutritionist.
iii) D is not as old as A and also not as young as B, who is not as old as the Engineer and also not as young as C.
iv) C is not the youngest among them.

Ans: Names are given in descending order of their ages.
Consultant Engineer Technician Planner Nutritionist
A D B C E
Solution: Think logically.

284. 4 persons, Watts, Roger, O’neil and Smith, were eating in a restaurant while Smith was killed by poisoning. During investigation the other 3 persons gave the following statements. They gave one false statement each.

Watts : i) I didn’t do it.
ii) O’neil sat beside me.
iii) We had our usual waiter.

Roger : i) Smith was across the table from me.
ii) We had a new waiter.
iii) The waiter didn’t do it.

O’neil : i) Roger didn’t do it.
ii) The waiter has poisoned Smith.
iii) Watts lied when he said that we had our usual
waiter.

One among these three persons and the waiter murdered Smith. Who is the murderer?

Ans : O’neil is the murderer.

Solution: Think logically.
I have given the false statements in red and italics.

285 There is a 4 inch cube painted on all sides. This is cut down into of 1 inch cubes. What is the no of
cubes which have no pointed sides.
Ans: 8 [(n-2)*(n-2)*(n-2)] where n is side length of the cube

286. Find the values of each of the alphabets.
N O O N
S O O N
+ M O O N
J U N E
Ans: 9326

287. If a clock takes 7seconds to strike 7, how long will the same clock take to strike 10?

Ans: The clock strikes for the first time at the start and takes 7 seconds for 6
intervals-thus for one interval time taken=7/6. Therefore, for 10 seconds there are 9 intervals and
time taken is 9*7/6=10 and 1/2 seconds.

288. Fifty minutes ago if it was four times as many
minutes past three o'clock, how many minutes is it to
six o'clock?
Ans: Twenty six minutes.

289. Everyday in his business a merchant had to weigh
amounts from 1 kg to 121kgs, to the nearest kg.
What are the minimum number of weight required
and how heavy should they be?
Ans: .The minimum number is 5 and they
should weigh 1,3,9,27 and 81kgs.


290. Ram, Shyam and Gumnaam are friends. Ram is a widower and lives alone and his sister
takes care of him. Shyam is a bachelor and his niece cooks his food
and looks after his house.Gumnaam is married to Gita and lives in large
house in the same town.Gita gives the idea that all of them could stay
together in the house and share monthly expensesequally.
During their first month of livingtogether, each person contributed Rs.25. At the end of
the month, it was found that Rs 92 was the expense sothe remaining amount was distributed equally among
everyone. The distribution was such that everyone received a whole number of Rupees. How much did each
person receive?Ans. Rs 2 (Hint: Ram's sister,Shyam's niece and Gumnaam's wife are the same person)

291. Sam and Mala have a conversation.

• Sam says I am certainly not over 40
• Mala says I am 38 and you are at least 5 years olderthan me
• Now Sam says you are at least 39
All the statements by the two are false. How old are they really?
Ans: Mala = 38 yrs, Sam = 41 yrs.

292. Grass in lawn grows equally thick and in a uniform rate. It takes 24 days for 70 cows and 60 days
for 30 cows to eat the whole of the grass. How many cows are needed to eat the grass in 96 days.?
Ans : 20
[Hint: g - grass at the beginning r -rate at which grass grows, per day y - rate at which
one cow eats grass, per day n - no of cows to eat the grass in 96 days
g + 24*r = 70 * 24* y g + 60*r= 30 * 60 * y g + 96*r = n * 96* y, Solving, n = 20. ]

293. Three criminals were arrested for shop lifting. However, when interrogated only one told the truth in
both his statements, while the other two each told one true statement and one lie. The statements were:
• ALBERT :(a) Chander passed the merchandise. (b)Bruce created the diversion.
• BRUCE :(a) Albert passed the merchandise. (b)I created the diversion.
• CLIVE :(a) I took the goods out of the shop. (b) Bruce passed them over.

Ans: Albert passed the goods. Bruce
created the diversion. Clive took the goods out of the
shop.

294. There N stations on a railroad. After adding X stations on the rail route 46 additional tickets have
to be printed. Find N and X.

Ans. x=2 and N=11 ( Let initially,N(N-1) = t; After adding, (N+X)(N+X-1) = t+46;Trail and error method )

295. Complete the Table given below:
Three football teams are there. Given below is
the group table. Fill in the x's
Played Won Lost Draw Goals For Goals Against
A 2 2 x x x 1
B 2 x x 1 2 4
C 2 x x x 3 7

Ans: The filled table is given below

Played Won Lost Draw Goals For Goals Against
A 2 2 0 0 7 1
B 2 0 1 1 2 4
C 2 0 1 1 3 7

297. Four prisoners escape from a prison. The prisoners, Mr. East, Mr. West, Mr. South, Mr. North
head towards different directions after escaping. The following information of their escape was supplied:
• The escape routes were The North Road, South Road,East Road and West Road.
• None of the prisoners took the road which was theirnamesake.
• Mr. East did not take the South Road
• Mr. West did not the South Road.
• The West Road was not taken by Mr. East
What road did each of the prisoners take to make
their escape?

Ans: Mr. East took the North Road
Mr. West took the East Road
Mr. North took the South Road
Mr. South took the West Road.

298. A hotel has two wings, the east wing and the west wing. Some east wing rooms but not all have an ocean
view.All west wing rooms have a harbor view. The charge for all rooms is identical, except as follows :

• Extra charge for all harbor view rooms on or above
the 3rd floor
• Extra charge for all ocean view rooms except those
without balcony
• Extra charge for some harbor rooms on the first two
floor & some east wing rooms without ocean view but
having kitchen facilities.

Which of the following cannot be determined on the
basis of the information given:
I. Whether there are any rooms without a
balcony for which an extra charge is imposed.
II. Whether any room without a kitchen or
a view involves an extra charge.
III. Whether two extra charges are imposed
for any room.
(A) I only (B) II only
(C) III only (D) II and III only
(E) I, II and III
Ans: (A)


299. A ship went on a voyage. After it had traveled 180 miles a plane started with 10 times the speed of the
ship. Find the distance when they meet from starting point.

Ans: 200miles. ( Distance traveled by plane = 1/10 distance traveled by ship + 180 )

301. Father's age is three years more than three times the son's age. After three years, father's age will be
ten years more than twice the son's age. What is the father's present age?
Ans: 33 years.

302. Light glows for every 13 seconds . How many times did it glow between 1:57:58 and 3:20:47 am.

Ans : 383 + 1 = 384


303. There are 20 poles with a constant distance between each pole. A car takes 24 second to reach the
12th pole.How much will it take to reach the last pole.
Ans: 41.45 seconds (Let thedistance between two poles = x, Hence 11x:24::19x:? )

304. A man collects cigarette stubs and makes one full cigarette with every 8 stubs. If he gets 64 stubs how
many full cigarettes can he smoke. Ans: 8+1=9

305. The minute and the hour hand of a watch meet every 65 minutes. How much does the watch lose or gain time
and by how much?
Ans: Gains; 5/11 minutes

306. A survey was taken among 100 people to find their preference of watching T. V. programs. There are 3
channels. Given the no of people who watch
• at least channel 1
• at least channel 2
• at least channel 3
• no channels at all
• at least channels 1and 3
• at least channels 1 and 2
• at least channels 2 and 3
Find the no of people who watched all three.

307. Some statements are given below:

• L says all of my other four friends have money
• M says that P said that exactly one among them has
money
• N says that L said that precisely two among them
have money
• O says that M said that three of the others have
money
• P, L and N said that they have money
All the above statement are false. Who has money & who doesn't have any money?

308. 500 men are arranged in an array of 10 rows and 50 columns according to their heights. Tallest among each
row of all are asked to fall out. And the shortest among them is A. Similarly after resuming that to
their original positions that the shortest among each column are asked to fall out. And the tallest among
them is B. Now who is taller among A and B ?
Ans. A

309. Mr. Mathurs jewels have been stolen from his bank
locker. The bank has lockers of 12 people which are
arranged in an array of 3 rows and 4 columns like:
1 2 3 4
5 6 7 8
9 10 11 12
• The locker belonging to JONES was to the right of
BLACK'S locker and directly above MILLAR'S.
• BOOTH'S locker was directly above MILLAR'S.
• SMITH'S locker was also above GRAY's (though not directly).
• GREEN'S locker was directly below SMITH'S.
• WILSON'S locker was between that of DAVIS and BOOTH.
• MILLAR'S locker was on the bottom row directly to
the right of HERD'S.
• WHITE'S locker was on the bottom right hand corner in the same column as BOOTH'S.
Which box belonged to Mr. Mathurs?

Ans: Box number 9 belongs to Mr.Mathurs.


310. Problem based on sets. 100 ppl. 85 are married, 70 have phone, 75 have house, 60 have car. find ppl having house, car, phone and r married?? easy set problem. i am not sure abt figures but find total ppl who do not have these things
and subtract from 100 to get ppl having all these.
ans. 10


311. village and town. in between a hill. a person travels on cycle 8 km uphill and 24 downhill to reach town continuously in 2hrs 50 min. then he comes to village in 4 hrs and 30 min. find his speed uphill and downhill.

ans. uphill 6
downhill 16

312. find wizard's age. his age is 3 times son's age, his father's age is 40 more than twice his age. total of their ages is 1240.
ans. 360

313. there r some steps. i come down 7 steps then see a man at bottom. then he comes up and i go down at same speed(my speed). when 4 steps r remaining for me i find tht man has reached the top.
for my single step downwards he took 2 steps up. find total steps.
ans. 22 steps.

314) One person went to market to purchase three varieties of chocolates. He had purchased 20 items with 20 cents. Fudges are available at 4 cents each, chaco bars are available at 4 for a penny and gum pints are available at 2 a penny. How many of each item he had purchased.

315) There is a five digit number, where the third number is one higher than the sum of first and second digits. Fourth digit is twice of fifth and third digit is twice the fourth. Second digit is five more than the first digit. What is the number

316) A cube, which is painted red on all its sides, is cut into 27 cubes with three straight cuts. Now how many cubes have

i) No red face
ii)One red face
iii)Two red faces
iv)Three red faces

317) It is a typical Tap problem. There are two taps, which are used to fill the tank and one tap to empty the tank. First tap fill the tank in 10 min., while the second takes quarter of an hour to fill the tank if both are operated independently. Third tap is capable of emptying the tank in seven and a half minutes. If all the taps are opened simultaneously (when the tank is empty) how long (if ever) will it take for the tank to get filled completely?

318) Ten years before, one is seven years more than the "half the age" of other. Now the bride’s age is 30 years. And also one is nine-tenths the age of other. What is groom’s present age.

319) Two guys are tossing coin with a bet of $1 for each game. After some tosses., one guy earned $3 while the other won three times. How many games do they play.

320) There is one 8 marks analytical question. Which is the easiest puzzle I had ever seen. There are in total eight members in the group. Five members are to be selected from that group. They had given three conditions. We have to answer four multiple-choice questions which are framed based on the above three conditions. In almost all the questions, answer can be found out by eliminating the answers from the choices. Going thru the GRE Barrons analytical section will help. Get the gist of it., rather than trying to solve more and more.

321) There are four friends, who are speaking about the pizza, which one of them ate before. Each one will say one statement and we were given that one of them is lying. We have to find out who ate the pizza. Easy one.., no need to worry abt. this.

322) All the above nine puzzles are very easy when compared to this one. This is difficult not because of logic but because of its grammer. The sentence structure is too complicate. This question reminded me of puzzle 127 of sakunthala devi’s Puzzles to Puzzle U.

323. there is a 1 km long wire plces on x poles . if the no of poles is reduced by 1 then he distance of wire between each poles
increases 1 2/3.how many poles are there intially.\

324. clark ,jones,mason, smith are 4 ppl. there are 4 professions druggists,grocer,butcher,policeman.find out who is who?

1)clark and jones are neighbours and they drive each other to work.
2)jones earns more than mason
3) the police man earns more than the druggists and i "think" the grocer.
4) the policeman does not have a druggists as a neighbour.
5) the butcher walks to work
6)policeman does not meet the grocer until he arrests him for commiting an offence.

325. A and B write a test
A says " i got a third of the ques. wrong"
B says " i got 5 wrong"
together they got three quarters of the questions correct.
how many did a get correct. (5 m)

326. If a die has 1,6 and 3,4 and 2,5 opposite each other how many such dies can be made.

327. There are three boxes , In one box Two white balls , In two box 2 black balls In three box 1 white &1 black
The lables on the boxes are not correct.Then you have to open one box and to find the colour of the balls in all boxes.

Solution: Open the box labled black& white If white balls are there then the box labled with white balls
contain black balls and labled with black balls contain one black and one white ball and viceversa
if two black ballsare there.


328). there are containing 5 , 7 , 14 , 16 , 18 , 29 balls of either red or blue in colour.
Some boxes contain only red balls and others contain only blue . One sales man sold one
box out of them and then he says " I have the same number of red balls left out as that of blue ".
Which box is the one he solds out ?
Ans : total no of balls = 89 and (89-29 /2 = 60/2 = 30
and also 14 + 16 = 5 + 7 + 18 = 30

329). A chain is broken into three pieces of equal lenths containing 3 links each. It is taken to a backsmith to join into a single continuous one . How many links are to to be opened to make it ? Ans : 2.

330) when the actual time pass 1hr wall clock is 10 min behind it when 1 hr is shown by wall clock, table clock shows 10 min ahead of it when table clock shows 1 hr the alarm clock goes 5min behind it, when alarm clock goes 1 hr wrist watch is 5 min ahead of it assuming that all clocks are correct with actual time at 12 noon what will be time shown by wrist watch after 6 hr
ans---5:47:32.5
(n X 60 )50/60 X 70/60 X 55/60 X 65/60

331. complete the following
a. $ * * $ @ * ? ?
# @ @ # # $ ? ? some what simillar like this...but not clear.
b. 1 , 3 , 7 , 13 , 21 , __ , 43 Ans : 31
c. 1, 3, 9, __ , 16900

332. A girl took part in a (some) game with many others in a circular closed circuit. After pedaling for several minutes, he found that 1/3th of the cyclists ahead of her and 3/4th of the cyclists behind him together formed the total no. of participants. How many were participating in the race?

333. OF all pets i have, except 2 all are rabbits
OF all pets i have, except 2 all are fish
OF all pets i have, except 2 all are cats
How many rabbits, fish and cats are there?

334. given
carpenter + painter = 1100
painter + electrician = 3200
electrician + plumber = 5100
plumber + mason = 2200
mason + labour = 3000
labour + painter = 1100
find every person's cash? (i dont know the exact amount mentioned above.)

335. out of 30 questions, the three persons A,B & C answered 45 correct answers,
B answered 55% of A, B and C together answered 25 % more of what A answered.
Find how many answers each answered?

336. Jim,Bud and sam were rounded up by the police yesterday. because one
of them was suspected of having robbed the local bank. The three suspects made
the following statements under intensive questioning.

Jim: I'm innocent
Bud: I'm innocent
Sam: Bud is the guilty one.
If only one of the statements turned out to be true, who robbed the bank?


337) There are two containers on a table. A and B . A is half full of
wine, while B, which is twice A's size,
is onequarter full of wine . Both containers are filled with water
and the contents are poured into a
third container C. What portion of container C's mixture is wine ?

338) A wall clock loses 10 minutes every 1 hour. In 1 hour by the wall
clock , a table clock gets 10 minutes ahead of it. In 1 hour by the table clock an alarm clock falls 5
minutes behind it. In 1 hour of the alarm clock, a wristwatch gets 5 minutes ahead it. At noon, all 4
timepieces were set correctly. To the nearest minutes, what time will the wrist show when the correct time is 6
p.m. on the same day ?

339) "You see," said Mrs.Murphy,"Paddy is now one and one-third times as
old as he was when he took to drink, and little Jimmy, who was forty months old when paddy took to
drink is now two years more than half as old as I was when Paddy took to drink , so when little Jimmy is as old as Paddy was when he took to drink.our three ages combined will amount to just one hundred years" How old is little Jimmy?

340)Both the Allens and the Smiths have two young sons under eleven. The name of the boys whose ages rounded off to the nearest year are all different are Arthur, Bert, Carl and David . Taking the ages of the boys only to the nearest year ,
the following staements are true

* Arthur is three years younger than his brother
* Bert is the oldest
* Carl is half as old as one of the allen boys
* David is five years older than the younger smith boy
* the total ages of the boys in each family differ by the same amount
today as they did five years ago

How old is each boy and what is each boys family name.

341) In a certain organization there are either men eligible to serve on a eligible to serve on a newly established commitee of four. The selection of the members is not an easy matter , however for there are jealousies and attachements among the candidates which prevents a free choice of four committeemen, if you were the president of the organization could you select a committee of four satisfying all these whims?

* Ames will serve with anybody
* Brown won't serve unless Clayton serves
* Clayton wont serve with Evans
* Davis wont serve without hughes
* evans will serve with anybody
* French wont serve with Davis unless Grant serves too, and wont
serve with Clayton unless Davis also
serves
* Grant wont serve with both Brown and Clayton and wont serve with
either Ames or Evens
* Hughes wont serve unless either Brown or French serves and wont
serve with Clayton unless Grant serves too and wont serve with both
Ames and Evans


342) An artist has exactly seven paintings --- ,T,U,V,W,X,Y, and Z -- from which she must choose exactly five
to be in an exhibit. Any combination is acceptable provided it meets the following conditions:

* If T is chosen , X cannot be chosen
* If U is chosen , Y must also be chosen
* If V is chosen , X must also be chosen

1) Which one of hte following is an aceptable combination of
paintings for inclusion in the exhibit?

A. T,U,V,X,Y
B. T,U,V,Y,Z
C. T,W,X,Y,Z
D. U,V,W,Y,Z
E. U,V,W,Z,Y

2) If painting T is chosen to be among the paintings included int he
exhibit which one of the following cannot be chosen to be among the
paintings included in the exhibit?

A. U
B. V
C. W
D. Y
E. Z

3)Which one of the following substitutions can the artist always make
without violating restrictions affecting the combination of paintings
given that the painting mentioned first was not, and the painting
mentioned first was not, and the painting mentioned second was,
originally going to be chosen ?

A. T replaces V
B. U replaces Y
C. V replaces X
D. W replaces Y
E. Z replaces W

4) If the artist chooses painting V to be included among the
paintings in the exhibit, which one of the following must be true of
that combination of paintings?

A. T is not chosen
B. Y is not chosen
C. U is chosen
D. W is chosen
E. Z is chosen

343) Yesterday my mother asked me to buy some stamps. Stamps are available in 2 paise,7paise,10paise,15paise and 20paise denominations. For three types of stamps I was asked to buy five of each. For the other two types of stamps. I was asked to buy six of each. Unfortunately I forgot which I was supposed to buy five of and which to buy six of
Luckly my mother had given me the exact money required to buy the stamps , Rs. 3.00 and the shopkeeper was able to give me the correct stamps. Which stamps did I buy?

344)Farmer Jones sold a pair of cows for Rs. 210 , On one he made a profit of ten percent and on the other he lost ten percent. Altogether he made a profit of five percent. How many did each cow originally cost him?

345.Meera was playing with her brother using 55 blocks.She gets bored playing and starts arranging the blocks
such that the no. of blocks in each row is one less than that in the lower row. Find how many were there
in the bottom most row?

346.Rahul took part in a cycling game with many others in a circular closed circuit. After pedaling for
several minutes, he found that 1/5th of the cyclists ahead of him and 5/6th of the cyclists behind him
together formed the total no. of participants. How many were participating in the race?

347.Tom wants to catch a hare. He is standing 250 yards south from the hare. The hare starts moving due east.
Tom, instead of moving in the northeast direction,moves in such a way that at every instant,
he is goingtowards the hare. If speed of tom is one and one-third times that of the hare, find the distance each
traveled before he caught the hare.

348.Two people are playing with a pair of dies. Instead of numbers, the dies have different colors on their
sides. The first person wins if the same color appears on both the dies and the second person wins if the
colors are different. The odds of their winning are equal. If the first dice has 5 red sides and 1 blue
side, find the color(s) on the second one.

349.A company's director said during the board meeting: " The company's income from roads will be
sufficient to pay 6% of the entire stock issue, but since we are paying 7.5% interest on the preferred
stock of Rs.4, 000,000 we are able to pay only 5% of the common stock". Find the value of the common stock.

350. Mr. ANYMAN left ANYTOWN by car to attend a wedding at ANYCITY. He had been driving for exactly two hours
when the car got punctured. It took his driver exactly ten minutes to change the wheel. In order to play safe
they covered the remaining distance at a speed of 30 mph. consequently, Mr. ANYMAN was at wedding half an-
hour behind schedule. Had the car got the puncture only 30 miles later , I would have been only FIFTEEN
minutes late he told the driver . How Far is ANYCITY from ANYTOWN.
Ans: 120 miles

351. Alpha, Beta , gamma, delta and epsilon are friends and have birthdays on consecutive days though may not
be in order. Gamma is as many days old to Alpha as Beta is younger to Epsilon. Delta is two days older
then Epsilon. Gamma’s Birthday is on Wednesday. Tell whose birthday is when.

Ans:
Alpha: Friday
Beta : Saturday
Gamma: Wednesday
Delta: Tuesday
Epsilon: Thursday

352.The quarter of the time from midnight to present time added to the half of the time from the present to
midnight gives the present time. What is the present time?

Ans: 9hrs past 36 minutes AM

353. A man is going to a wedding party. He travels for 2hrs when he gets a puncture. Changing tyres takes 10mins.
The rest of the journey he travels at 30 miles/hr. He reaches 30mins behind schedule. He thinks to himself that
if the puncture had occurred 30miles later, he would have been only 15mins late.
Find the total distance traveled by the man

354. After world war II three departments did as follows First department gave some tanks to 2nd &3rd departments equal to
the number they are having. Then 2nd department gave some tanks to 1st & 3rd departments equal to
the number they are having. Then 3rd department gave some tanks to 2nd &1st departments equal to
the number they are having. Then each department has 24 tanks. Find the initial number of tanks of each department?

Ans ; A-39 B-21 C-12

355. A girl 'A' told to her friend about the size and color of a snake she has seen
in the beach. It is one of the colors brown/black/green and one of the sizes 35/45/55.

If it were not green or if it were not of length 35 it is 55.
If it were not black or if it were not of length 45 it is 55.
If it were not black or if it were not of length 35 it is 55.

a) What is the color of the snake?
b) What is the length of the snake?

Ans: a) brown b) 55

356.A man was on his way to a marriage in a car with a constant speed. After 2 hours one of the tier is punctured
and it took 10 minutes to replace it. After that they traveled with a speed of 30 miles/hr and reached the marriage
30 minutes late to the scheduled time. The driver told that they would be late by 15 minutes only if the 10 minutes
was not waste. Find the distance between the two towns?


357. Three clocks where set to true time. First run with the exact time. Second slows one minute/day. Third gains one minute/day. After how many days they will show true time.

358. There were some containers of quantity 1, 3, 4, 5, 6, 12, 15, 22, 24, 38 liters. Each was filled with some liquid except one. The liquids are milk, water and oil. Quantity of each was like this. Water = 2* milk oil = 2* water. Find out which container was empty and containers filled with milk and oil.

359. Two travelers, one with 64 barrels of wine, other with 20 barrels of wine. They don't have enough money to pay duty for the same. First traveler pays 40 francs and gives his 5 barrels, Second traveler gives his 2 barrels but gets 40 francs in exchange. What's value of each barrel, and duty for each barrel?

Ans: Value of each barrel-120 francs, Duty on each-10 francs

360. What is Ann's relation with her husband's mother's only daughter-in-law's sister's husband?

Ans: Brother-in-law

361. Some guy holding a glass of wine in his hand looking around in the room says, "This is same as it was four years ago, how old are your two kids now?" Other guy says "Three now, Pam had one more in the meanwhile." Pam says, "If you multiply their ages, answer is 96 and if you add the ages of first two kids, addition is same as our house number." The first guy says, "You are very smart but that doesn't tell me their ages." Pam says, "It's very simple, just think." What are the ages of three kids?

Ans: 8, 6, 2


362. A motor cyclist participant of a race says "We drove with the speed of 10 miles an hour one way, but while returning because of less traffic we drove on the same route with 15 miles per hour." What was their average speed in the whole journey?

Ans: 12 miles per hour


363. Given following sequence, find the next term in the series:

(i) 0, 2, 4, 6, 8, 12, 12, 20, 16, ____ Ans: 12

(ii) 3, 6, 13, 26, 33, 66, ___ Ans: 53

364. Three customers want haircut and a shave. In a saloon, two barbers operate at same speed. They take quarter of an hour for the haircut and 5 mins for the shave. How quickly can they finish the haircut and shave of these three customers?

Ans: 30 minutes

365. A shopkeeper likes to arrange and rearrange his collection of stamps. He arranges them sometimes in pair, sometimes in bundle of three, sometimes in bundle of fours, occasionally in bundle of fives and sixes. Every time he's left with one stamp in hand after arrangement in bundles. But if he arranges in the bundle of seven, he's not left with any stamp. How many stamps does a shopkeeper have?

Ans: 301

366. Three different types of objects in a bucket. How many times does one need to select object from the bucket to get atleast 3 objects of the same type?

Ans: 7


367). A stamp collector has the habit to arrange or rearrange the stamps accordingly. while doing this he some times keeps the stamps in pairs, or in group of 3 or in 4 or in or in 6 and realises that in any case he is left with 1 stamp and when he arranges them in groups of 7 no stamps remain. what is the number of stamps he has?

368). amy while walkin down the street with her daughter, meets her husband's mother's only duaghter in law's sister's husband. how is the related to her? C het anaS

369). there are 3 custoners who wants to take a hair cut and shave. there are 2 barbers who takes one quarter of an hour for a hair cut, and 5 minutes for a shave. both the barbers want to finish off and go quickly to their homes. in what time can do it.

370). we travelled to a place at the rate of 10 miles per hour and offcourse returned the same way, but owing to less traffic at the rate of 15 miles per hour.what was our relative speed.

371). there are 3 types of apples in a box. what is the number of apples we should take so that we end up with 3 apples of one kind.

372). a).3,6,13,26,33,66,_ b).0, 1,2,13 ,6,33 ,12, 63,20, 103,_


373. Each alphabet A,B.. Z is a constant.A=1,B=2,C=3^2,D=4^9 n so on.Each letter is assigned a value -the position of that letter raised to the value of preceding alphabet.(C = 3 ^B,D= 4 ^ C n so on)
Compute the numerical value of (X-A)(X-B)(X-C)....(X-Y)(X-Z).

374. Mr. T has a wrong weighing pan.One arm is lengthier than other.1
kilogram on left balances 8 melons on right.1 kilogram on right
balances 2 melons on left.If all melons are equal in weight,what is
the weight of a single melon?

375. In a game of dice, 2 dice thrown at each turn.The score at each
turn is taken as the product of number on 2 dices.there were five
turns(rolls).second roll is 1 less than first n (like this ... the
relationship between third n second,third n fourth,fourth n fifth )
were given.Find the score in first roll,second roll,third roll,fourth
roll.(jus giving an idea abt question and don know the exact
relationships).

PURPLE:these are always poisonous if red are.
YELLOW:six months in a year they are safe.
GREEN: always safe to consume if purple are poisonous.
RED:six months in a year poisonous.
The colors are colors of mushrooms available.AT this time
which one is safe to consume?


G: I am 22
M = G + 2
H = G - 1
H: I am not the youngest.
difference between M & H is 3
G is 25.

M: M is younger than G.
G = 23.
H = G+2.
Mr.G,Mr.M,Mr.H made only one statement false.

Find the ages of all three.
(I am not sure abt the statements jus chk it out with others)


376 Matrix problem repeated from prev paper
there are 3 males A,B,C and 3 females X,Y,W. they played 18 games of
golf altogether.
1. A scored 94
2. X scored 106
3. Y scored 102.
4. Z scored 100.
5. B and C scored 96 and 98 and don't no who's score what??
6. A's wife beats C's wife.
7. there are two couples whose sum of scores is same.
Determine who's wife is who and scores of B and C.


377. A girl had several dollars with her. she went out for shopping and spent half of them in shopping mall,being generous she had given 1 dollar to the beggar. After that she wqent for lunch and spent the half of the remaining and gave 2 dollars as tip to the waiter.Then she went to watch a movie and spent the half of remaining dollars and
gave autorikshaw-wala 3 dollars. This left her with only 1 dollar.How many dollars did she had with her at the beggining.

378. A person says that his son is 5 times as old as his daughter and his wife is 5 times older than his son and he is twice the age of his wife . The sum total of all the ages equals the age of the grand mother who celebrated her 81st birthday today.
How old was his son?

379. A bargain hunter bought some plates for $ 1.30 from asale on saturday,where price 2 was marked off at each article .On monday she went to return them at regular prices,and bought some cups and saucers from that much amount of money only.
the normal price of plate were equal to the price of 'one cup and one saucer'.In total she bought 16 items more than previous. saucers were only of 3 cents hence she brought 10 saucers more tahn the cups,How many cups and saucers she bought and at what price?

380. A jeweller prepared a window display each displaying 3 of the 7 jems at a time . They were methyst,opal,sapphire,emerald, ruby and garnet.Displayed according to the following conditions:-
1 A sholud always be displayed on the left window and D on the right.
2 Ruby should never come with any of D or G.
3.E should always be with S.
then some 4 questions were asked on this. easy
#1 which combination is appropriate?
AOS;ADR;AES(ans).
#2 which condition is correct in the right window?
#3 Ruby can be displayed with following other two?
#4 S can be displayed with the following other two on left side window?

381. Racing competition.Participants were from 3 tribes
Sonorean-always says truth
Midorean-alternatively says T and F ,not with any particular start.
Nororean-always False
A says-1. C obstructed me at the last moment ,which caused me to losethe race.
2. C always speak true
3. c is the winner.
B says - 1. A is the winner.
2. c says false always.
C says- 1.B won the Race
2. I didn't caused any obstruction to A at the last time.
IDentify the tribes of each.

382. A boss tells 1/6 th of his life in child hood,1/12 of his in youth and 1/7 of his in bachelor,five years after his election
a son was born whom was died four years ago at half his final age. find the boss age.
. ANS.:74 ths solution a/6+a/12+a/7 +5......

383. Two thieves went to the museum to stole the diamonds first thief stole half of them and while going he took another two and left. Second, third and fourth did the same and there was zero diamonds at the end. How many diamonds initially at the beginning?

Similar to Q. No. 193 Stolen Mangoes from Shakuntala Devi - More Puzzles Ans: 79

384. A, B, C are the husbands and D, E, F are their wives not in that order. They are playing the Golf following these conditions. D, E, F and B scores are as follows 106,102,100 and 94.A and C scores are 98 and 96 not in that order as their names are not displayed. Two couples get the same score. B wife beat the A wife list out the wives names and the scores they got.

Ans:
Hus Wife Score Total
A F 98 102 200 100 198
B D 94 106 200 106 200
C E 96 100 96 102 198

385.A women with dollar bills goto the shopping he spent half of the money she had for shopping as she was so kind she gave one dollar to the beggar.she went to the hotel and spent half of the remaning and she gave 2 dollars to the waiter,the she buy some goods with half of the remaining and she gave 3 dollars to the receptionist. how much money she had in the begining?
ans:42
come in the reverse order 1+3=4and twice of it 8+2=10 and twice of it 20+1=21 and twice of it 42.

386.A conducter in the bus ask the man how old the boy is.the man replied that my son is five times older than my daughter and my wife is five times older than my son and i am twice older to my wife and our ages summed upto my granmother whose age is 81 years.can u tell me the son age?
ans: 5 years
x+5x+25X+50X=81
81X=81
x=1 therefore son age is 5 years old

387.find out who is oldest and who is youngest from the following statements...
a)either A or B r the oldest
b)either C is the oldest or B is the youngest.
ans: A is the oldest and B is the Youngest

388.one boy tells three ppl to guess mule color..
number one says its not blk
number two says its eiterh brown or grey
number 3 says its brown.
..the boy then says one is atleast lying ans atleast telling truth...finf mule color
ans grey?

389. 2 men take turns walking and riding one horse that they share...walking speed
4km/hr..riding speed 12km/hr..one rides for some time and ties horse for the other walking fellow and continues walking......they keep going on like this alternately ..find time that the horse rests

390. 7 ppl have holidays on 7 diff days...and they give conditions like A's hol is 3 days
before B etc etc...we have to find the days on which they take holiday
5.6 subjects 3 ppl teach 2 each...some crazy conditions and we gotta find the subject for
each..i cant really recall clearly so why confuse u guys


391. A Lady (say L) is a philanthropist. she goes to a restaurent,
orders food and pays half the amount she has and another doller to a
waiter as tip. she then goes to a mall, does some purchases and pays
half the amount left and another 2 dollers to a begger outside. At
last she goes to a book store, takes some books and pays half the
amount left and another 3 dollers to a begger outside. she then
checks that she only had a doller left to her. How much money she had
initially? ANs. 42 $

392. Three couples are playing golf together. The men are E, B & T while women are M G & H. they play 8 rounds.
M, G, H & E score 106, 102, 100, 94 respectively. while B & T scored either 96 or 98 as it was unresolved due
to error at scoreboard. When finally resolved they found that two of the couples scored same. It is given that-
E's wife scored greater than B's wife. Who's whose wife & how much the men scored each?

393. A lady buys some plates in 130$ with 2$ off every item. she then returns the plates for the same amount next day to exchange them for some cups & saucers. Each saucer costs only 3$ each and the no. of saucers is 10 more than cups. Altogether she takes 16 more items than before. If she had to buy only cups, how many of them she could have
been taken home on the first day?

394. Impressed by admiration of the boy by stranger, the father
said "My son is five times as old as my daughter and my wife is five
times as old as my son. I'm double the age of my wife and my
grandmother is as old as the sum of ages of all of us and she is
celebrating her 81st birthday." what is the age of the boy?

ANS. 5 yrs

395. A, B & C participate in a race & one of them wins. They belong to
three communities-M, N,
O. O always speak the truth, N always lie and M alternate. Each of A,
B & C belongs to one community.

A SAYS:
1. I would have won the race if C had not interfered me at the last movement.
2. C always speaks truth.
3. C is the winner.

B SAYS:
1. A wins the race.
2. C is not a 'N'.

C SAYS:
1. I hadn't interfered with A at the last movement.
2. B wins the race.

Tell, who's who?


396. Harry is a friend of Axy and Amy. Two statements are given about
them.
1. if one of Axy and Amy is oldest then another is youngest.
2. Either Harry is the oldest or Amy is the youngest.
Who is the oldest?
ANS. Axy

397.sons age is 5 times daughters.mother is five times son.father is 5 times wife.
Total of all age is Grandpas who is celebrating 81st B`day.
Ans:-5 yrs

398) One woman buys plates worth 1.30$ at 2cent discount each plate.Then she exchanged the plates
for sausers & bowl where one bowl & one sausers costs equal to one plate.no of sausers which costs 3 cent
is10 more than bowl. no of saucers & bowl is 16 more than no of plates.
Ans:-10Plates

399. "One-sixth of my life", said my boss, "I spent as a child, next one-twelfth as an old boy, one-seventh & 5
more years in politics & socialization. This brought me upto when Jimmy born. Jimmy was elected for the
governer four years ago, when he was half my present age."
How old is my boss?

ANS.: 84 yrs.
(However, I overlooked that 'one-seventh' part, & got
the answer 36 - a wrong answer.)
*******************************************************J
= (B/2) + 4
(B/6) + (B/12) + (B/7) + 5 + J = B
=> B = 84
*******************************************************

401.A Couple decided to travel a north country side .so they decide to travel a minimum amount on car the first day and the second and subsequent day a distance of 20 miles .If they travel a total amount of 1080 miles. Find he distance traveled on the 4th day and the 9 day.

402. A card board of 34 * 14 has to be attached to a wooden box and a total of 35 pins are to be used on the each side of the card box. Find the total number of pins used .

403. During a Pizza buffet where A eats more times 2.4 than B, and B eats 6 times
less than C.find the leat number of times all the three has to eat.

404. Last Year my cousin came to my place and we played a game where the loosing one has to give one choclate to the person who won the game .At the end of the vacation,i.e the day my cousin was leaving she counted number of games that i won an she won.At last she gave me a total of 8 choclates even though she won about 12 games.
Find the mumber of games that we played.

405. A tree on first day grows 1/2 of its size second day 1/3rd of its size on the previous day similarly than 1/4th and so on.u have to calculate after how many days the tree will be 100 times of its original size.
ans -198 days

406. three person are there let A B C one always tell truth one always lie and one sometimes tell truth
they are standing in straight line the first person who is seniormost and always tells truth tells in middle A is standing the middle one says C is in the third position the last one says B is in second position ie middle

407. A,B,C,Dand E are juniors and F,G,H,I are seniors you have to make three groups each containg three person such that in each group one senior is there and some other conditions which i cant recall exactly but was like that if this person will be in group than this cant be in the same group .

408. there are 100 teams in a football knockout tournament how many mathces should be held to get the winner answer is 99 ie one less than the no of teams bcoz in every match on team goes out

409.A car traveling with uniform speed. There r 15 poles. A car travel from 1 to 10th pole in 10 seconds. the poles are equally spaced. then how many seconds it takes to reach the 15th pole? (4)

410. A boy jump to the river from the bridge. He swim opposite direction of the stream.
After 1000 yards he noted that his hat was fallen at the bridge. The he goes to take the hat. He didn’t change his speed.What is the velocity of stream? (4)

411. Uncles A bought a Hat & Suit for Rs.15.Aunty B bought a Suit with as much as cost of uncle’s Hat. Then reaming cost
he bought Dresses. Dresses cost is one rupee more than his hat’s cost. Then she told him his hat’s hat is 1 and ½ cost of
her hat. They spend equal amount on their purchase .
a. Then uncle said how much is the cost of Hat.
b. How much they spend altogether ? (4)


412. There are 17 brown ties,13 red ties, 9 green ties, 5 blue ties and 2 white ties.Then a man takes a tie.
so, how many times he at least take tie to get the 2 ties In same colour? (6)

413. Mrs. Barbinger bought some plates on Saturday for $1.30, when everything was being sold two cents below the regular price. She exchanged those plates on Monday, at their regular price, for cups & saucers.
Cost of one plate equals cost of one plate & one saucer. She returned home with 16 more articles than before. Since, saucers cost only 3 cents each, she bought 10 more saucers than cups.
The puzzle is, how many cups could she have bought on Saturday, for $1.30?

ANS.: 13 cups
*******************************************************
On Monday:
cup = 12 cents, saucer = 3 cents, plate = 15 cents
On Saturday:
cup = 10 cents, saucer = 1 cent, plate = 13 cents
*******************************************************

414. Mr and Mrs ABC purchase suit and hats for 15 Rs. then from remaining money Mrs. ABC purchase A dress. She shaid " My dress cost is more than 1 Rs from your hat's cost.
she also added "if we divide our money and then purchase and cost of my hat is 3 and 1/2 times yours hat then we had spend equal money"
a. " if that " condition fallows what is price of his hat?
b. Total amount spend ?
ans. hat= 6 Rs , Total = 22 Rs. ( not sure)

415. My rack contains 8 Red colour ties, 13 violate colour ties,10 Blue colour ties, 5 Pink colour ties, 4 green colour ties. If electricity gone and i want at least two ties of same colour then how many ties i should take out from my rack?
Ans : 6 ties.

416. Two trains leaving from two station 50 miles away from each other with costant speed of 60 miles per hour, approaches towards each other on diffrent tracks. if lenght of each train is 1/6 mile. when they meet How much time they need to pass each other totally?
ANS : 10 sec. ( not sure)

417. All handsome, fair skinned, muscular, lean, employed, and rich men are tall.
All handsome men are fair skinned.
Some muscular men are handsome.
Some muscular men are not fair skinned.
All lean men are muscular.
No lean man is handsome.
No fair skinned man who is not handsome is rich.
All tall men who are neither fair skinned nor muscular are employed.

1. pramod is not fair skinned. Which of the following must be true ?
a) pramod is employed
b) if pramod is muscular, he is neither handsome nor lean
c) if pramod is tall, he is employed or muscular.
d) if pramod is not employed, he is muscular.
e) if pramod is tall, he may be muscular or handsome, but not both.
2. which must be false if the information given is true ?
a) no lean men are fair skinned.
b) some fair skinned are lean.
c) some rich men are both fair skinned and muscular.
d) some tall men are neither fair skinned nor employed
e) some rich men are lean
3. which of the following can be deduced from the information given ?
a) all rich men are handsome
b) some rich men are handsome
c) some rich men are employed
d) some rich men are muscular
e) all rich men are handsome, muscular, or employed
4. which cannot be shown to be true or false on the basis of the information given ?
I. No fair skinned or muscular man is employed
II. Some muscular men are fair skinned but not handsome
III. No fair skinned man both handsome and lean

a) I only b) II only c) III only d) I and II e) II and III

418 In Mulund, the shoe store is closed every Monday, the boutique is closed every Tuesday, the grocery store is closed every Thursday and the bank is open only on Monday, Wednesday and Friday. Everything is closed on Sunday.
One day A, B, C and D went shopping together, each with a different place to go. They made the following statements:

A D and I wanted to go earlier in the week but there wasn’t day when we could both take care of our errands.
B I did not want to come today but tomorrow I will not be able to do what I want to do.
C I could have gone yesterday or the day before just as well as today.
D Either yesterday or tomorrow would have suited me.

Which place did each person visit ?

419 The Novice hockey tournaments are on for beginners. Just three teams are in the league, and each plays the other two teams just once. Only part of the information appears in the result chart, which is given below.
Team Games Won Lost Tied Goals For Goals against
A 2 1 0
B 2 1 1 2
C 2

The scoring pattern in the tournament is as follows:

Two points are awarded to the winning team. In case of a tie, both teams are awarded one point, so the total points in the standings should always equal the total number of games played ( since each game played is counted as one for each of the two participating teams). Of course, total goals scored for and goals scored against must be the same, since every goal scored for one team is scored against another.
The games are played in the following order: Game 1: A Vs B; Game 2: A Vs C; Game B Vs C
Can you determine the score of each of the above games ?


420 A recent murder case centered around the six men, clam, flip, gront, herm, mast, and walt. In one order or another these man were the victim, the murderer, the witness, the police, the judge, and the hangman. The facts of the case were simple. The victim had died instantly from the effect of gunshot wound inflicted a shot. After a lengthy trial the murderer was convicted, sentenced to death, and hanged.

V Mast knew both the victim and the murderer.
v In court the judge asked clam his account of the shooting.
V Walt was the last of the six to see flip alive.
V The police testified that he picked up gront near the place where the body was found.
V Herm and walt never met.
What role did each of the following play in this melodrama ?
a) Murderer
b) Victim
c) Judge
d) Witness

421.A alone can do a work in 6 days B alone can do in 8 days with help of c they finished the work in 3 days.If the agreed sum is 640 what is the share of c.(refer r.s agarwal)

422) A boy goes to school from his house.on one fourth oh his way to school,
he crosses a machinery station. And on one third of his way to school, he crosses a Railway station. He crossed the machinery station at 7:30 and he crosses the Railway station at 7:35. When does he leave the house & when does he reach the school ? (5M)

423.A drives a car four times a lap 10,20 30,60 kmph what is the average speed.

424.speed of boat in still water 10 km,if speed up stream is 24 km and speed down stream is 16 what is speed of the river.

425.If grand father age is sum all the three grand childern whos age r in equal interval what is the age of the grand father?

426.In a grass field if 40 cow could eat for 40 days.The same grass field can feed 30 cows for 60 days.how long
will it feed 20 cows?

427) An Eraser, Pencil, Notebook together costs $1.00. Notebook costs more than the cost of 2 Pencils. 3 Pencil costs more than 4 Erasers. 3 Erasers costs more than a Notebook.
How much does a pencil costs? (5M)

428) Four persons A,B,C,D were there. All were of different weights. All Four gave a
statement.Among the four statements only the person who is lightest in weight of all
others gave a true statement.

A Says : B is heavier than D.
B Says : A is heavier than C.
C Says : I am heavier than D.
D Says : C is heavier than B.
Find the lightest & List the persons in ascending order according to their weights. (5M)

428. A man was travelling to a place 30 miles away from starting point. he was speeding at 60 miles/hr. but when he came back, his car got breakdown and half an hour was wasted in reparing that. altogether he took 1 hr for return journey. Find the avg. speed of the whole journey.

429.I'M NOT VERY SURE REGARDING THIS QUESTION.
A detective was assigned to generate a code using 4 digits, so that no one could break it. he knew that if the code starts with 0,5 or 7 it will be cracked. so how many numbers can be formed using 4 digits.

430. A cow was standing on a bridge, 5feet away from the middle of the bridge. suddenly a lightning express with 90 miles/hr was coming towards the bridge from nearest end of the cow.seeeing this the cow ran towards the express and managed to escape when the train is one feet away from the bridge. if it would have ran to opposite direction(ie away from train) it would have been hit the train one ft away from the end of the bridge. Calculate the length of bridge.

431. there are 3 towns attacked by 3 dragons-x,y,z. Number of days x attack a town is equal to number of days y attacking another town. Number of days x attack is equal to half the square root of number of days z attacking a town.number of days y attacking the town is twice the square root of z.calculate how much days the curse of each dragon be.

432. A town have a population of 500000 and 42% of males and 28% of females are married to same town. find the total number of males

433. A and B came back home after their exam and their father asked them about the test.
A replied-- 1/3rd of my answers were wrong
B replied-- 5 of my answers were wrong but together we got 3/4 of answers right.
How many questions were there for the exam?

434) We are given 100 pieces of a puzzle. If fixing two components together is counted as 1 move ( a component can be one piece or an already fixed set of pieces), how many moves do we need to fix the entire puzzle.
Ans: 18

435) This problem has appeared b4. 4 kids from 2 families Gupta and sharma... their names are Praveen, Pra... blah blah...and some relation... which family and what are their ages..
Ans: Sinhas - 11(praveen)
Sinhas - 5(lalit)
Gupta - 10(pratap)
Gupta - 7 (rajesh)

436) This was one hell of a long problem - But it has appeared before.. Two girlz after one guy ( sue, swe and sam i think)..Sue first asks his house no... he replies with two statements (second is false) so she goes to the wrong house... blah blah.. same thing happens with the other gal
What was the house number of sue and sam
Ans: 20 and 24

437) A complex statement - about an aeroplane comming late. "The boy says if it was 6 hours later, the waiting time would be 1/5th of the time if the plane had come 2 hours earlier instead. the plane is supposed to come at midnight
Ans 11.00 a.m

438) There are 4 statements and 4 guys ( Dave, Gus, someone else and one more someone else) - sorry my memory is rotten! Anyway now these guys make 4 statements abt. who commited the crime. We need to find out who did it if (i) all but one are saying false and (ii) all but one are saying true
Ans: (i)archie( ii) tony

439) A kid goes to a bank with a checque... Comes back and says he spent 35 cents .. his mom says you have brought back twice what u ver supposed to get. What happened is the novice bank clerk gave dollars for cents and vice-versa(quote silly) anway... What was the actual amount??
Ans: don't know...


440. John had decided to divide his RS.1000/- for his four children according to their ages. The elder child should be a RS.20/- extra for each than his younger child . What will be the share of Mahesh whois the youngest?[3 marks]

441.One side of the measuring arm was longer than the other side .If 3 pyramid width was placed in longer side it is equal to 2 cube width in the shorter arm.If 8 pyramid width was placed in longer arm then it is equal to 6 cube width. Let 1 pyramid width is equal to 10kg.What is the wgt of cube width?

442. A,B,C,D are four girls who have 1,2,3,4 apples with them respectively.If E have apples equal to his sister,F have twice the apples as his sister,G have thrice the apples than his sister and I have four times the apples than my sister.All together we have 32 apples . A,B,C,D are the sisters of whom and whom?

443.Andy,Brian,Cedric,Dave are architects ,barber, case worker and dentist but not in the order.
*Architect will have the letter ‘r’ in his name.
*Atleast one of the person should have coincidence in the first letter of their name and their occupation but not all [eg: andy-architect]
* Barber and dentist share their name by only one letter
What is the occupation of each person?


444.There are 100 bulbs connected to 100 switches
• 1 to 100 all the switches are put ON.
• Only even numbers of switches are used ie, ON means OFF and viceversa
• Similarly odd numbers of switches are done
• Switch number which is divisible by 3 are done similarly
• Switch number which are divisible by 4 are done similarly
This process is done upto 100 divisibles.
When do all the bulbs are in ON and OFF condition?[8marks]

445. A man wrote his “will’ accordingly the money was also shared between his car driver,5 sons and 5 daughters.First he gave one rupee to his car driver,remaining 1/5 of the money is given to his 1st son .again he gave one to his car driver and remaining 1/5 of the money is given to his 2nd son, continuing the process until 5 sons are completed .After that he remaining money is divided between his 5 daughters. What was total amount of money?

446. 4 people identified a criminal and their statements are:
# A: Eyes was blue ,height was tall and he wore a hat & a vest.
# B: Eyes was dark , height wa short and he wore a hat & a vest
# C: Eyes was green ,height was medium and he wore hat & a tie.
# D: Eyes was grey ,height was tall and he wore a rain coat and a hat.
Everyone said only one correct identify other two was untrue . How can be the criminal identified?

4470. A wall clock was slow by 10minutes . According to the wall clock,a table clock was 10 minutes ahead of it . According to the table clock an alarm clock was 5 minutes behind and according to the alarm clock wrst watch was 5 minutes fast.Atnoon all the clocks were adjusted .What will be the time at 6 P.M wrist watch?

448.Professor Kittredge’s literature seminar includes students with varied tastes
in poetry.All those in the seminar who enjoy the poetry of browing also enjoy
the poetry of eliot.Those who enjoy the poetry of eliot despise the poetry of coleridge.
Some of those who enjoy the poetry of eliot also enjoy the poetry of Auden.
All of those who enjoy the poetry of Coleridge also enjoy the poetry of Donne.
Some of those who enjoy the poetry of Auden also despise the poetry of Coleridge.
All of those who enjoy the poetry of Donne also enjoy the poetry of Frost.

i.
Miss Garfield enjoys the poetry of Donne.Which of the following must be true?
(A) she may or may not enjoy the poetry of coleridge.
(B) She does not enjoy the poetry of Browing.
(C) She does not enjoy the poetry of eliot.
(D) She enjoys the poetry of Coleridge.

ii.
Mr.Huxtable enjoys the poetry of Browing.He may also enjoy any of the following
poets except
(A) Auden.
(B) Coleridge
(C) Donne
(D) Eliot
(E) Frost

iii.
Miss Inaguchi enjoys the poetry of Coleridge.Which of the following must be false?
(A) she does not enjoy the poetry of Auden.
(B) She enjoys the poetry of Donne.
(C) She enjoys the poetry of Frost.
(D) She does not enjoy the poetry of Browning.
(E) She may enjoy the poetry of Eliot.

iv.
Based on the information provided,which of the following statements concerning
the members of the seminar must be true?
(A) All the those who enjoy the poetry of eliot also enjoy the poetry of Browning.
(B) None of those who despise the poetry of Frost enjoy the poetry of Auden.
(C) Some of those who enjoy the poetry of Auden despise the poetry of Coleridge.
(D) None of those who enjoy the poetry of Browning despise the poetry of Donne.
(E) Some of those who enjoys the poetry of Frost despise the poetry of Donne.


a.Rimmie wears a hat only if goerge wears a tie.
b.George wears a scarf only if Johnie wears a tie.
c.Vickie wears a googles only if Rimmie wears a hat.
d,e,f,g,h

some what like this there are 8 statements,from that we have to determine who wears what.


449. 8 Kigs and 14 Ligs can do 510 tors of work in10days. 13 Kigs and 6 Ligs
can do 484 tors of work in 12 days.
Then find work done by Kigs and Ligs individually in tors/hr?

450.There is a 3 digited number. 3rd number is the square root of the 1st digit.
2nd digit is the sum of 1st and 3rd.And that number is divisible by 2,3,6,7.
What is that number?

451. A boy is playing a game. He took totally 55 blocks and kept like placing
some x number on the ground,next one less than that above those blocks
like that till the topmost one is one,like:

x
x x
x x x
x x x x
x x x x x
x x x x x x
x x x x x x x
x x x x x x x x
x x x x x x x x x
x x x x x x x x x x
(They didn’t gave this pictuire,but my explanation is not clear,that’s I gave u this picture)
the question is how many blocks are there at the base level?

452. There r 100 nations competing for a world-cup.
The board decided to make Knock-out series.
How many matches to be played for deciding the world champion?

453.There is log weighing 30kgs. The log having twice thickness and twice short as first one will weigh howmuch ??

454. there ia truck which should reach some place at 11`o clock , if it travels with 30 mph it reaches i
hour before , if it travles with 20 mph it reaches 1 hour late. what is the distance it must be travlled
and what is the speed it must maintain to rech at exact time?
ans: 120 miles and 24 mph

455.There is a square cabbage patch.He told his sister that i have a larger patch than last year and hence 211 more cabbages this year. Then how many cabbages I have this year.?
Ans:106*106=11236

456. there are two colcks one runs 1min/hrs faster and other 1min/hr slower
when will the two clocks have time time difference of 1 hr :
ans : 30hrs

457. i take a taxi whose no is 3 digit no. it is not divisible by 2,3,5,7
but divisible by 11 it is the smallest no possible:
ans : 121




458. A man brought some watermelons to town and sold them. he sold 1/2 more than 1/2 of what he brought and e was left with one melon. how many melons did he bring to Town? ans: 3

459. When u reverse the digits of age of father u will get the age of son. one year ago the age of father was twice that of son's age. what are the current ages of father and son?
ans: 73 & 37

460. There will be four friends ,one is doctor, one is lawyer... they are having four cars one ferrari, corvette .... there were some conditions and we were suppose to find what is profession of each one and what car they own. this was the one for 8 marks.

461. There is a circular ring in which there are 12 black mice and one white mice . a cat walks circularly in the ring and eats every 12th mice. where should the cat start so that the white mice is the last one to be eaten by cat?
ans: if the cat moves circularly it has to start from the 11th mice (clockwise) w.r.t. to white one.

462. A farmer grows four types of crops say W,X,Y & Z . two conditions were given:
1. If the farmer grows crop W in a year then also grows X that year
2. If the farmer grows crop Z one year then he never grows crop Y next year
There were 6 choices among which we were suppose to find one correct one which does not violate the two conditions the choices were some thing like:( first pair denotes crops grown first year and second after semicolon represents crops grown next year) W,X ; X,Z (this one is a valid one)
Ans: I don’t remember the choices but the ans was option C

463. In a class there are less than 500 students . when it is divided by 3 it gives a whole number. Similarly when it is divided by 4,5 or 7 gives a whole number. find the no. of students in the class ans: 420

464. There are three types of birds A,B & C . A costs 5pounds, B costs 3 Pounds and C costs 1/3 of a pound. find the no. of A,B &C such that u will get 100 birds for 100 pounds. (I think we were suppose to find 3 answers since there were 3 rows in the answer) ans: A : 4 B : 18 C: 78

465. There are 5 persons who have won top five places in an event in Olympics . one of them asks all the five regarding thier positions, they reply as
a: "i am not the last"
b: "c is in third place"
c: "E is behind A"
d: "B is in first place"
e: "D is not the first"
The persons who have won gold and silver have lied find the positions in order(format: name of first, name of second,..)
ans: B,D,E,A,C
466. A coffee seller has two types of coffee Brand A costing 5 bits per pound and Brand B costing 3 bits per pound. he mixes two brands to get a 40 pound mixture. he sold this at 6 bits per pound. the seller gets a profit of 33 1/2 percent. how much he has used Brand A in the mixture? ans: 30 pounds

467. You are given with two identical iron bars. one of them is magnetized and the other is not. u are suppose to find which one is magnetized. u are not suppose to use any other thing.
my ans: first time i told that i will suspend the bars freely. but then they told me that i am not suppose to use any external help. i took some time and then realized that a magnetic bar in the middle repels towards the end . i told that place one of the bars horizontal & then move the other one perpendicularly to it . if it repels towards any of ends then the horizontal one is magnetized if it attracts then is not. They were convinced

468: What is the result of (x-a)*(x-b)*(x-c)*...........*(x-y)*(x-z) ?
my ans: i told that since there is a term (x-x) the answer is zero. They told ok. finally they asked me whether i have any questions to them i asked them what is the duration of training and what is field which i will be working on if i get selected.
Anyone looking for infi solve Shakuntala devi's 2 books, George summers and Ravi narula this would me more than enough. most important thing is develop ur logical analysis skill and try to remember the approach rather than answers. don’t panic in the interview just be cool and confident u will definitely get through. Best of luck for all

469) a) 10 1 9 2 8 3 7 4 6 5 5 6 4 7 3 8 2 _ _
b) 2 4 16 512 _
Write the next elements in the series.